ip developed reviewer

165
Intellectual Property Law Course Outline FEU Institute of Law MBA JD Program 2007-2008 By: Atty. Anthony D. Bengzon I. Preliminary Matters A. What is Intellectual Property? (Sec. 4.1, IP Code) Section 4. Definitions. - 4.1. The term "intellectual property rights" consists of: 1. Copyright and related rights – e.g., Books, Music, Films (criminal case) 2. Trademarks and Service Marks – e.g.: Rolex, Nokia, Coca Cola 3. Geographic Indications – e.g.: Napa Valley; Parmigiano- Reggiano 4. Industrial Designs – e.g.: toothbrush, pen, cellular phone 5. Patents – e.g.: Stents, Viagra (utility model- civil case) 6. Lay-Out Designs (Topographies) of Integrated Circuits (kind of industrial design) 7. Protection of Undisclosed Information (Trade Secrets) (no law yet) see: AirPhil vs Penzwell GR No. 172835 December 13, 2007 B. The Intellectual Property Office 1. Bureau of Patents (Sec. 8) Section 8. The Bureau of Patents. - The Bureau of Patents shall have the following functions: 8.1. Search and examination of patent applications and the grant of patents; 8.2. Registration of utility models , industrial designs , and integrated circuits ; and 8.3. Conduct studies and researches in the field of patents in order to assist the Director General in formulating policies on the administration and examination of patents. 2. Bureau of Trademarks (Sec. 9) Section 9. The Bureau of Trademarks. - The Bureau of Trademarks 1

Upload: alyasah-mulok-andig

Post on 25-Oct-2014

108 views

Category:

Documents


3 download

TRANSCRIPT

Page 1: IP Developed Reviewer

Intellectual Property Law Course OutlineFEU Institute of Law

MBA JD Program 2007-2008

By: Atty. Anthony D. Bengzon

I. Preliminary Matters

A. What is Intellectual Property? (Sec. 4.1, IP Code)Section 4. Definitions. - 4.1. The term "intellectual property rights" consists of:

1. Copyright and related rights – e.g., Books, Music, Films (criminal case)2. Trademarks and Service Marks – e.g.: Rolex, Nokia, Coca Cola3. Geographic Indications – e.g.: Napa Valley; Parmigiano-Reggiano4. Industrial Designs – e.g.: toothbrush, pen, cellular phone 5. Patents – e.g.: Stents, Viagra (utility model- civil case)6. Lay-Out Designs (Topographies) of Integrated Circuits (kind of industrial design)7. Protection of Undisclosed Information (Trade Secrets) (no law yet) see: AirPhil vs

Penzwell GR No. 172835 December 13, 2007

B. The Intellectual Property Office

1. Bureau of Patents (Sec. 8)Section 8. The Bureau of Patents. - The Bureau of Patents shall have the following functions:8.1. Search and examination of patent applications and the grant of patents;8.2. Registration of utility models, industrial designs, and integrated circuits; and8.3. Conduct studies and researches in the field of patents in order to assist the

Director General in formulating policies on the administration and examination of patents.

2. Bureau of Trademarks (Sec. 9)Section 9. The Bureau of Trademarks. - The Bureau of Trademarks shall have the

following functions:9.1. Search and examination of the applications for the registration of marks,

geographic indications and other marks of ownership and the issuance of the certificates of registration; and

9.2. Conduct studies and researches in the field of trademarks in order to assist the Director General in formulating policies on the administration and examination of trademarks.

3. Bureau of Legal Affairs (Sec. 10)Section 10. The Bureau of Legal Affairs. - The Bureau of Legal Affairs shall have the following functions:10.1. Hear and decide opposition to the application for registration of marks;

cancellation of trademarks; subject to the provisions of Section 64,

1

Page 2: IP Developed Reviewer

cancellation of patents, utility models, and industrial designs; and petitions for compulsory licensing of patents;

10.2. (a) Exercise original jurisdiction in administrative complaints for violations of laws involving intellectual property rights: Provided, That its jurisdiction is limited to complaints where the total damages claimed are not less than P200,000: Provided further, That availment of the provisional remedies may be granted in accordance with the Rules of Court. The Director of Legal Affairs shall have the power to hold and punish for contempt all those who disregard orders or writs issued in the course of the proceedings.

(b) After formal investigation, the Director for Legal Affairs may impose 1 or more of the following administrative penalties:

(i) The issuance of a cease and desist order which shall specify the acts that the respondent shall cease and desist from and shall require him to submit a compliance report within a reasonable time which shall be fixed in the order;

(ii) The acceptance of a voluntary assurance of compliance or discontinuance as may be imposed. Such voluntary assurance may include one or more of the following:(1) An assurance to comply with the provisions of the intellectual

property law violated;(2) An assurance to refrain from engaging in unlawful and unfair acts

and practices subject of the formal investigation;(3) An assurance to recall, replace, repair, or refund the money value of

defective goods distributed in commerce; and(4) An assurance to reimburse the complainant the expenses and costs

incurred in prosecuting the case in the Bureau of Legal Affairs.The Director of Legal Affairs may also require the respondent to submit periodic compliance reports and file a bond to guarantee compliance of his undertaking;

(iii) The condemnation or seizure of products, which are subject of the offense. The goods seized hereunder shall be disposed of in such manner as may be deemed appropriate by the Director of Legal Affairs, such as by sale, donation to distressed local governments or to charitable or relief institutions, exportation, recycling into other goods, or any combination thereof, under such guidelines as he may provide;

(iv) The forfeiture of paraphernalia and all real and personal properties, which have been used in the commission of the offense;

(v) The imposition of administrative fines in such amount as deemed reasonable by the Director of Legal Affairs, which shall in no case be less than P5,000 nor more than P150,000. In addition, an additional fine of not more than P1,000 shall be imposed for each day of continuing violation;

(vi) The cancellation of any permit, license, authority, or registration which may have been granted by the Office, or the suspension of the validity thereof for such period of time as the Director of Legal Affairs may deem reasonable which shall not exceed 1 year;

2

Page 3: IP Developed Reviewer

(vii) The withholding of any permit, license, authority, or registration which is being secured by the respondent from the Office;

(viii) The assessment of damages;(ix) Censure; and (express severe disapproval)(x) Other analogous penalties or sanctions. (Secs. 6, 7, 8, and 9, Executive

Order No. 913 [1983]a)10.3. The Director General may by Regulations establish the procedure to govern

the implementation of this Section.

C. The National Library and Supreme Court Library (Sec. 191) not a mandatory requirement only a system of deposit to make it easier to prove that the right to copyright exists as of

a certain date does not give rise to a right

Section 191. Registration and Deposit with National Library and the Supreme Court Library. - After the first public dissemination of performance by authority of the copyright owner of a work falling under Subsections 172.1, 172.2 and 172.3 (literary & artistic works) of this Act, there shall, for the purpose of completing the records of the National Library and the Supreme Court Library, within 3 weeks, be registered and deposited with it, by personal delivery or by registered mail 2 complete copies or reproductions of the work in such form as the directors of said libraries may prescribe. A certificate of deposit shall be issued for which the prescribed fee shall be collected and the copyright owner shall be exempt from making additional deposit of the works with the National Library and the Supreme Court Library under other laws. If, within 3 weeks after receipt by the copyright owner of a written demand from the directors for such deposit, the required copies or reproductions are not delivered and the fee is not paid, the copyright owner shall be liable to pay a fine equivalent to the required fee per month of delay and to pay to the National Library and the Supreme Court Library the amount of the retail price of the best edition of the work. Only the above mentioned classes of work shall be accepted for deposit by the National Library and the Supreme Court Library. (Sec. 26, P.D. No. 49a)

Kho v. Court of Appeals, GR No. 115758, 19 March 2002Issue: Whether the copyright and patent over the name and container of a beauty cream product

would entitle the registrant to the use and ownership over the same to the exclusion of othersHeld: No. trademark is any visible sign capable of distinguishing the goods (trademark) or services

(service mark) of an enterprise and shall include a stamped or marked container of goods scope of a copyright is confined to literary and artistic works which are original intellectual

creations in the literary and artistic domain protected from the moment of their creation. Patentable inventions, on the other hand, refer to any technical solution of a problem in any

field of human activity which is new, involves an inventive step and is industrially applicablePetitioner has no right to support her claim for the exclusive use of the subject trade name and its container. The name and container of a beauty cream product are proper subjects of a trademark inasmuch as the same falls squarely within its definition. In order to be entitled to exclusively use the same in the sale of the beauty cream product, the user must sufficiently prove that she registered or used it before anybody else did. The petitioner’s copyright and patent registration of

3

Page 4: IP Developed Reviewer

the name and container would not guarantee her the right to the exclusive use of the same for the reason that they are not appropriate subjects of the said intellectual rights.

Ching v. Salinas, GR No. 161295, 29 June 2005Issue: Whether a “Leaf Spring Eye Bushing for Automobile” is a proper subject of a copyright. Held: No.The author’s intellectual creation, regardless of whether it is a creation with utilitarian functions or incorporated in a useful article produced on an industrial scale, is protected by copyright law.  However, the law refers to a “work of applied art which is an artistic creation.”  It bears stressing that there is no copyright protection for works of applied art or industrial design which have aesthetic or artistic features that cannot be identified separately from the utilitarian aspects of the article. Functional components of useful articles, no matter how artistically designed, have generally been denied copyright protection unless they are separable from the useful article.Being plain automotive spare parts that must conform to the original structural design of the components they seek to replace, the Leaf Spring Eye Bushing and Vehicle Bearing Cushion are not ornamental.  They lack the decorative quality or value that must characterize authentic works of applied art.  They are not even artistic creations with incidental utilitarian functions or works incorporated in a useful article.  In actuality, the personal properties described in the search warrants are mechanical works, the principal function of which is utility sans any aesthetic embellishment.

Note:A certificate of registration creates rebuttable presumption of copyright validity where other evidence in the record casts doubt on the question.  In such a case, validity will be presumed.

II. Patents

A. What is a Patent? (Sec. 21)Section 21. Patentable Inventions. - Any technical solution of a problem in any field of human activity which is 2new, involves an 3inventive step and is 4industrially applicable shall be Patentable. It may be, or may relate to, a 1product, or 2process, or an 3improvement of any of the foregoing. (Sec. 7, R.A. No. 165a)

1. Product or Processa) Product- include every output of human ingenuity, every tangible result of

human craftsmanship or partisan (but must possess all the requirements of patentability)

b) Process- “consists of an act, operation, or step, or a series thereof, performed upon a specified subject matter to produce a physical result. xxx Where a process consists of more than a single step, the arrangement, order, or sequence in which the component steps are to be performed may itself be of patentable significance.” A new use for an old substance is patentable provided that the new use

must be recited or reported as a method.

2. A monopoly in exchange for disclosure

4

Page 5: IP Developed Reviewer

3. Rights granted; Product and Process Patents (Sec. 71) territorial

Section 71. Rights Conferred by Patent. – 71.1. A patent shall confer on its owner the following exclusive rights:

(a) Where the subject matter of a patent is a product, to restrain, prohibit and prevent any unauthorized person or entity from making, using, offering for sale, selling OR importing that product ; The defense that the product marketed does not originate from the

process used by the patentee is unavailing. Disjunctive. When one makes a product that is protected by a patent,

there can be infringement even if this product is not sold.

(b) Where the subject matter of a patent is a process, to restrain, prevent or prohibit any unauthorized person or entity from using the process, AND from manufacturing, dealing in, using, selling or offering for sale, or importing any product obtained directly or indirectly from such process . Where the product is identical, it shall be presumed to have been obtained

through the use of the patented process (Sec.78). The onus probandi is then on the defendant to establish that he obtained the product from another process that does not infringe patent rights.

71.2. Patent owners shall also have the right to assign, or transfer by succession the patent, and to conclude licensing contracts for the same. (Sec. 37, R.A. No. 165a)

It is the claim that determines the boundaries of the patent.

4. Term of patent (Sec. 54)Section 54. Term of Patent. - The term of a patent shall be 20 years from the filing date of the application. (Sec. 21, R.A. No. 165a)

After expiration becomes part of public domain

5. Utility Models (Sec. 109, 109.3)Section 109. Special Provisions Relating to Utility Models. – 109.1.

(a) An invention qualifies for registration as a utility model if it is 1new and 2industrially applicable.

(b) Section 21, "Patentable Inventions", shall apply except the reference to inventive step as a condition of protection.

109.2. Sections 43 to 49 shall not apply in the case of applications for registration of a utility model.

109.3. A utility model registration shall expire, without any possibility of renewal, at the end of the 7th year after the date of the filing of the application.

109.4. In proceedings under Sections 61 to 64, the utility model registration shall be canceled on the following grounds:

(a) That the claimed invention does not qualify for registration as a utility model and does not meet the requirements of registrability, in particular having regard to Subsection 109.1 and Sections 22, 23, 24 and 27;

5

Page 6: IP Developed Reviewer

(b) That the description and the claims do not comply with the prescribed requirements;

(c) That any drawing which is necessary for the understanding of the invention has not been furnished;

(d) That the owner of the utility model registration is not the inventor or his successor in title. (Secs. 55, 56, and 57, R.A. No. 165a)

1) useful machine, 2) an implement or tool, 3) a product or composition, 4) an improvement of any of the foregoing (Part II, Rule 201 of the Rules & Regulations on Utility Models & Industrial Designs)

Sec. 110 allows a patent application to be converted into an application for utility model protection provided that the conversion occurs prior to the grant or to the refusal of the patent.

Sec. 111 forbids the filing of parallel applications: the same invention that is submitted for patent protection at the same time as it is submitted for utility model protection.

May an application for utility model be filed for a product that has been denied patent protection? Aquino: Yes.

6. Industrial Designs (Sec. 112, 118)Section 112. Definition of Industrial Design. - An industrial design is any composition

of lines or colors or any three-dimensional form, whether or not associated-with lines or colors: Provided, That such composition or form gives a special appearance to and can serve as pattern for an industrial product or handicraft. (Sec. 55, R.A. No. 165a)

Section 118. The Term of Industrial Design Registration. – 118.1 The registration of an industrial design shall be for a period of 5 years from the

filing date of the application.118.2. The registration of an industrial design may be renewed for not more than 2

consecutive periods of 5 years each, by paying the renewal fee.118.3. The renewal fee shall be paid within 12 months preceding the expiration of the

period of registration. However, a grace period of 6 months shall be granted for payment of the fees after such expiration, upon payment of a surcharge

118.4. The Regulations shall fix the amount of renewal fee, the surcharge and other requirements regarding the recording of renewals of registration.

B. What are not patentable? (Sec. 22) See amendments in RA 9502

Section 22. Non-Patentable Inventions. - The following shall be excluded from patent protection:22.1. Discoveries, scientific theories and mathematical methods; (no longer excluded)22.2. Schemes, rules and methods of performing mental acts, playing games or doing

business, and programs for computers; - but OS, for example, may be copyrightable 22.3. Methods for treatment of the human or animal body by surgery or therapy and diagnostic

methods practiced on the human or animal body. This provision shall not apply to products and composition for use in any of these methods;

6

Page 7: IP Developed Reviewer

22.4. Plant varieties or animal breeds or essentially biological process for the production of plants or animals. This provision shall not apply to micro-organisms and non-biological and microbiological processes. (nature)

Provisions under this subsection shall not preclude Congress to consider the enactment of a law providing sui generis protection of plant varieties and animal breeds and a system of community intellectual rights protection:

22.5. Aesthetic creations; and22.6. Anything which is contrary to public order or morality. (Sec. 8, R.A. No. 165a)

C. Requirements for Patentability

1. Novelty (Sec. 23)Section 23. Novelty. . - An invention shall not be considered new if it forms part of a prior art. (Sec. 9, R.A. No. 165a)

a. What is Prior Art? (Sec. 24)Section 24. Prior Art. - Prior art shall consist of: (synonymous with prejudicial disclosure) (memorize)24.1. Everything which has been made available to the public anywhere in the

world, before the filing date or the priority date of the application claiming the invention; and (worldwide search)

24.2. The whole contents of an application for a patent, utility model, or industrial design registration, published in accordance with this Act, filed or effective in the Philippines, with a filing or priority date that is earlier than the filing or priority date of the application: Provided, That the application which has validly claimed the filing date of an earlier application under Section 31 of this Act, shall be prior art with effect as of the filing date of such earlier application: Provided further, That the applicant or the inventor identified in both applications are not one and the same. (Sec. 9, R.A. No. 165a) (National search; published applications in the Philippines)

b. Searching for Prior Art With the advent of the internet, it is now much easier to search if

something forms part of a prior art.

c. Non-Prejudicial Disclosure (Sec. 25)Section 25. Non-Prejudicial Disclosure. . – 25.1. The disclosure of information contained in the application during the 12

months preceding the filing date or the priority date of the application shall not prejudice the applicant on the ground of lack of novelty if such disclosure was made by: (beyond 12 months, will lapse into the public domain and be prior art already)

(a) The inventor;(b) A patent office and the information was contained

(a) in another application filed by the inventor and should not have been disclosed by the office, or

7

Page 8: IP Developed Reviewer

(b) in an application filed without the knowledge or consent of the inventor by a third party which obtained the information directly or indirectly from the inventor; or

(c) A third party which obtained the information directly or indirectly from the inventor.

25.2. For the purposes of Subsection 25.1, "inventor" also means any person who, at the filing date of application, had the right to the patent.

2. Inventiveness / Non-Obviousness (Sec. 26)Section 26. Inventive Step. - An invention involves an inventive step if, having regard to

prior art, it is not obvious to a person skilled in the art at the time of the filing date or priority date of the application claiming the invention. The inventor is entitled to the obvious & logical consequences of his own

invention. Inferential proofs for non-obviousness (by Goldstein): 1) long-felt demand, 2)

commercial acquiescence in its patentability, 3) commercial success. The mental state of the inventor is of no relevance at all towards the resolution

of the issue of “inventiveness,” and it can very well be the case that that for which the patent is sought was never that which the inventor endeavored to bring about.

3. Industrial Applicability (Sec. 27)Section 27. Industrial Applicability. - An invention that 1can be produced and 2used in any industry shall be industrially applicable.

There is no legal requirement that the ultimate product, process or improvement be that originally intended. But it is required that the invention fulfill the requirement of industrial applicability that the purpose of the product, process or improvement supposedly invented be achieved, at least in part.

D. Ownership of Patent (Sec. 28)Section 28. Right to a Patent. - The right to a patent belongs to the inventor, his heirs,

or assigns. When 2 or more persons have jointly made an invention, the right to a patent shall belong to them jointly. (Sec. 10, R.A. No. 165a)

Inventor Applicant Patentee There is NO international patent

1. First to File Rule (Sec. 29)Section 29. First to File Rule. - If 2 or more persons have made the invention

separately and independently of each other, the right to the patent shall belong to the person who filed an application for such invention, or where two or more applications are filed for the same invention, to the applicant who has the earliest filing date or, the earliest priority date. (3rd sentence, Sec. 10, R.A. No. 165a.)

Right of priority (Section 31) – foreign application

2. Exception to First to File Rule – The True and Actual Inventor (Sec. 67, 68)

8

Page 9: IP Developed Reviewer

Section 67. Patent Application by Persons Not Having the Right to a Patent. . – (Patent is still under application. Contemplates a situation wherein patent was granted to someone other than the first to file… but when will that happen?)67.1. If a person referred to in Section 29 other than the applicant, is declared by final court order or decision as having the right to the patent, such person may, within 3 months after the decision has become final:

(a) Prosecute the application as his own application in place of the applicant;(b) File a new patent application in respect of the same invention;(c) Request that the application be refused; or(d) Seek cancellation of the patent, if one has already been issued.

67.2. The provisions of Subsection 38.2 shall apply mutatis mutandis to a new application filed under Subsection 67. 1(b). (n)

Section 68. Remedies of the True and Actual Inventor. – (patent was already granted) If a person, who was deprived of the patent without his consent or through fraud is declared by final court order or decision to be the true and actual inventor, the court shall order for his substitution as patentee, or at the option of the true inventor, cancel the patent , and award actual and other damages in his favor if warranted by the circumstances. (Sec. 33, R.A. No. 165a)

3. Commissioned Inventions (Sec. 30)Section 30. Inventions Created Pursuant to a Commission. – 30.1. The person who commissions the work shall own the patent, unless otherwise

provided in the contract. No employer-employee relationship30.2. In case the employee made the invention in the course of his employment

contract, the patent shall belong to: employer employee relationship exists(a) The employee, if the inventive activity is not a part of his regular duties even if

the employee uses the time, facilities and materials of the employer.(b) The employer, if the invention is the result of the performance of his regularly-

assigned duties, unless there is an agreement, express or implied, to the contrary.

4. Right of Priority (Sec. 31)Section 31. Right of Priority. . - An application for patent filed by any person who has

previously applied for the same invention in another country which by treaty, convention, or law affords similar privileges to Filipino citizens, shall be considered as filed as of the date of filing the foreign application: Provided, That: (a) the local application expressly claims priority; (b) it is filed within 12 months from the date the earliest foreign application was

filed; and (c) a certified copy of the foreign application together with an English translation is

filed within 6 months from the date of filing in the Philippines. (Sec. 15, R.A. No. 165a)

E. The Patent Application (Sec. 32-37)Section 32. The Application. –

9

Page 10: IP Developed Reviewer

32.1. The patent application shall be in Filipino or English and shall contain the following:(a) A request for the grant of a patent;(b) A description of the invention;(c) Drawings necessary for the understanding of the invention;(d) One or more claims; and(e) An abstract.

32.2. No patent may be granted unless the application identifies the inventor. If the applicant is not the inventor, the Office may require him to submit said authority. (Sec. 13, R.A. No. 165a)

Section 33. Appointment of Agent or Representative. - An applicant who is not a resident of the Philippines must appoint and maintain a resident agent or representative in the Philippines upon whom notice or process for judicial or administrative procedure relating to the application for patent or the patent may be served. (Sec. 11, R.A. No. 165a)

Section 34. The Request. - The request shall contain a petition for the grant of the patent, the name and other data of the applicant, the inventor and the agent and the title of the invention. (n)

Section 35. Disclosure and Description of the Invention. – 35.1. Disclosure. - The application shall disclose the invention in a manner sufficiently clear

and complete for it to be carried out by a person skilled in the art. Where the application concerns a microbiological process or the product thereof and involves the use of a micro-organism which cannot be sufficiently disclosed in the application in such a way as to enable the invention to be carried out by a person skilled in the art, and such material is not available to the public, the application shall be supplemented by a deposit of such material with an international depository institution.

35.2. Description. - The Regulations shall prescribe the contents of the description and the order of presentation. (Sec. 14, R.A. No. 165a)

Section 36. The Claims. – 36.1. The application shall contain 1 or more claims which shall define the matter for which

protection is sought. Each claim shall be clear and concise, and shall be supported by the description.

36.2. The Regulations shall prescribe the manner of the presentation of claims. (n)Section 37. The Abstract. - The abstract shall consist of a concise summary of the

disclosure of the invention as contained in the description, claims and drawings in preferably not more than 150 words. It must be drafted in a way which allows the clear understanding of the technical problem, the gist of the solution of that problem through the invention, and the principal use or uses of the invention. The abstract shall merely serve for technical information. (n)

F. Procedure for Grant of Patent (Sec. 40-50)FilingFormality examinationClassification & SearchPublication – inchoate right of applicantRequest for Substantive ExaminationGrant/ DenialSection 40. Filing Date Requirements. –

10

Page 11: IP Developed Reviewer

40.1. The filing date of a patent application shall be the date of receipt by the Office of at least the following elements:

(a) An express or implicit indication that a Philippine patent is sought;(b) Information identifying the applicant; and(c) Description of the invention and 1 or more claims in Filipino or English.

40.2. If any of these elements is not submitted within the period set by the Regulations, the application shall be considered withdrawn. (n)

Section 41. According a Filing Date. - The Office shall examine whether the patent application satisfies the requirements for the grant of date of filing as provided in Section 40 hereof. If the date of filing cannot be accorded, the applicant shall be given an opportunity to correct the deficiencies in accordance with the implementing Regulations. If the application does not contain all the elements indicated in Section 40, the filing date should be that date when all the elements are received. If the deficiencies are not remedied within the prescribed time limit, the application shall be considered withdrawn. (n)

Section 42. Formality Examination. – 42.1. After the patent application has been accorded a filing date and the required fees

have been paid on time in accordance with the Regulations, the applicant shall comply with the formal requirements specified by Section 32 and the Regulations within the prescribed period, otherwise the application shall be considered withdrawn.

42.2. The Regulations shall determine the procedure for the re-examination and revival of an application as well as the appeal to the Director of Patents from any final action by the examiner. (Sec. 16, R.A. No. 165a)

Section 43. Classification and Search. - An application that has complied with the formal requirements shall be classified and a search conducted to determine the prior art. (n)

Section 44. Publication of Patent Application. – 44.1. The patent application shall be published in the IPO Gazette together with a search

document established by or on behalf of the Office citing any documents that reflect prior art, after the expiration of 18 months from the filing date or priority date.

44.2. After publication of a patent application, any interested party may inspect the application documents filed with the Office.

44.3. The Director General subject to the approval of the Secretary of Trade and Industry, may prohibit or restrict the publication of an application, if in his opinion, to do so would be prejudicial to the national security and interests of the Republic of the Philippines. (n)

Section 45. Confidentiality Before Publication. - A patent application, which has not yet been published, and all related documents, shall not be made available for inspection without the consent of the applicant. (n)

Section 46. Rights Conferred by a Patent Application After Publication. - The applicant shall have all the rights of a patentee under Section 76 against any person who, without his authorization, exercised any of the rights conferred under Section 71 of this Act in relation to the invention claimed in the published patent application, as if a patent had been granted for that invention: Provided, That the said person had:

46.1. Actual knowledge that the invention that he was using was the subject matter of a published application; or

46.2. Received written notice that the invention that he was using was the subject matter of a published application being identified in the said notice by its serial number: Provided, That the action may not be filed until after the grant of a patent on the

11

Page 12: IP Developed Reviewer

published application and within 4 years from the commission of the acts complained of. (n)

Section 47. Observation by Third Parties. - Following the publication of the patent application, any person may present observations in writing concerning the patentability of the invention. Such observations shall be communicated to the applicant who may comment on them. The Office shall acknowledge and put such observations and comment in the file of the application to which it relates. (n)

Section 48. Request for Substantive Examination. – 48.1. The application shall be deemed withdrawn unless within 6 months from the date of

publication under Section 41, a written request to determine whether a patent application meets the requirements of Sections 21 to 27 and Sections 32 to 39 and the fees have been paid on time.

48.2. Withdrawal of the request for examination shall be irrevocable and shall not authorize the refund of any fee. (n)

Section 49. Amendment of Application. - An applicant may amend the patent application during examination: Provided, That such amendment shall not include new matter outside the scope of the disclosure contained in the application as filed. (n)

Doctrine of wrapper estoppel Precludes him from adding that which he had to excise or to excise that which he

had to add by patent laws and procedures to the breadth of his claim

Section 50. Grant of Patent. – 50.1. If the application meets the requirements of this Act, the Office shall grant the patent:

Provided, That all the fees are paid on time.50.2. If the required fees for grant and printing are not paid in due time, the application

shall be deemed to be withdrawn.50.3. A patent shall take effect on the date of the publication of the grant of the patent in

the IPO Gazette. (Sec. 18, R.A. No. 165a)

G. Rights of Patentees and Infringement of Patents

1. Rights conferred (Sec. 71)Section 71. Rights Conferred by Patent. – (infringement if without the authorization of the patentee)71.1. A patent shall confer on its owner the following exclusive rights:(a) Where the subject matter of a patent is a product, to restrain, prohibit and

prevent any unauthorized person or entity from making, using, offering for sale, selling or importing that product;

(b) Where the subject matter of a patent is a process, to restrain, prevent or prohibit any unauthorized person or entity from using the process, and from manufacturing, dealing in, using, selling or offering for sale, or importing any product obtained directly or indirectly from such process.

71.2. Patent owners shall also have the right to assign, or transfer by succession the patent, and to conclude licensing contracts for the same. (Sec. 37, R.A. No. 165a)

12

Page 13: IP Developed Reviewer

2. Assignment of rights (Sec. 103, 104)Section 103. Transmission of Rights. – 103.1. Patents or applications for patents and invention to which they relate, shall

be protected in the same way as the rights of other property under the Civil Code.

103.2. Inventions and any right, title or interest in and to patents and inventions covered thereby, may be assigned or transmitted by inheritance or bequest or may be the subject of a license contract. (Sec. 50, R.A. No. 165a)

Section 104. Assignment of Inventions. - An assignment may be of the entire right, title or interest in and to the patent and the invention covered thereby, or of an undivided share of the entire patent and invention, in which event the parties become joint owners thereof. An assignment may be limited to a specified territory. (Sec. 51, R.A. No. 165)

3. Limitations on Patentee’s Rights (Sec. 72-74)Section 72. Limitations of Patent Rights. - The owner of a patent has no right to

prevent third parties from performing, without his authorization, the acts referred to in Section 71 hereof in the following circumstances:

72.1. Using a patented product which has been put on the market in the Philippines by the owner of the product, or with his express consent, insofar as such use is performed after that product has been so put on the said market;

72.2. Where the act is done privately and on a non-commercial scale or for a non-commercial purpose: Provided, That it does not significantly prejudice the economic interests of the owner of the patent;

72.3. Where the act consists of making or using exclusively for the purpose of experiments that relate to the subject matter of the patented invention;

72.4. Where the act consists of the preparation for individual cases, in a pharmacy or by a medical professional, of a medicine in accordance with a medical prescription or acts concerning the medicine so prepared;

72.5. Where the invention is used in any ship, vessel, aircraft, or land vehicle of any other country entering the territory of the Philippines temporarily or accidentally: Provided, That such invention is used exclusively for the needs of the ship, vessel, aircraft, or land vehicle and not used for the manufacturing of anything to be sold within the Philippines. (Secs. 38 and 39, R.A. No. 165a)

Section 73. Prior User. – 73.1. Notwithstanding Section 72 hereof, any prior user, who, in good faith was

using the invention or has undertaken serious preparations to use the invention in his enterprise or business, before the filing date or priority date of the application on which a patent is granted, shall have the right to continue the use thereof as envisaged in such preparations within the territory where the patent produces its effect.

73.2. The right of the prior user may only be transferred or assigned together with his enterprise or business, or with that part of his enterprise or business in which the use or preparations for use have been made. (Sec. 40, R.A. No. 165a)

Section 74. Use of Invention by Government. – 74.1. A Government agency or third person authorized by the Government may

exploit the invention even without agreement of the patent owner where:

13

Page 14: IP Developed Reviewer

(a) The public interest, in particular, national security, nutrition, health or the development of other sectors, as determined by the appropriate agency of the government, so requires; or

(b) A judicial or administrative body has determined that the manner of exploitation, by the owner of the patent or his licensee is anti-competitive.

74.2. The use by the Government, or third person authorized by the Government shall be subject, mutatis mutandis, to the conditions set forth in Sections 95 to 97 and 100 to 102. (Sec. 41, R.A. No. 165a)

4. Action for Infringement (Sec. 75, 76)Section 75. Extent of Protection and Interpretation of Claims. – 75.1. The extent of protection conferred by the patent shall be determined by the

claims, which are to be interpreted in the light of the description and drawings.75.2. For the purpose of determining the extent of protection conferred by the

patent, due account shall be taken of elements which are equivalent to the elements expressed in the claims, so that a claim shall be considered to cover not only all the elements as expressed therein, but also equivalents. (n)

Doctrine of equivalents Purpose is to protect a patented invention from circumvention by minor

changes or deviations Function-means-result test: There is equivalence if the accused' device

performs substantially the same function in substantially the same way to obtain the same result.’

The patentee has the burden to prove the infringement.

Section 76. Civil Action for Infringement. – 76.1. The making, using, offering for sale, selling, or importing a patented product

or a product obtained directly or indirectly from a patented process, or the use of a patented process without the authorization of the patentee constitutes patent infringement.

76.2. Any patentee, or anyone possessing any right, title or interest in and to the patented invention, whose rights have been infringed, may bring a civil action before a court of competent jurisdiction, to recover from the infringer such damages sustained thereby, plus attorney's fees and other expenses of litigation, and to secure an injunction for the protection of his rights.

76.3. If the damages are inadequate or cannot be readily ascertained with reasonable certainty, the court may award by way of damages a sum equivalent to reasonable royalty.

76.4. The court may, according to the circumstances of the case, award damages in a sum above the amount found as actual damages sustained: Provided, That the award does not exceed 3 times the amount of such actual damages.

76.5. The court may, in its discretion, order that the infringing goods, materials and implements predominantly used in the infringement be disposed of outside the channels of commerce or destroyed, without compensation.

76.6. Anyone who actively induces the infringement of a patent or provides the infringer with a component of a patented product or of a product produced because of a patented process knowing it to be especially adopted for infringing

14

Page 15: IP Developed Reviewer

the patented invention and not suitable for substantial non-infringing use shall be liable as a contributory infringer and shall be jointly and severally liable with the infringer. (Sec. 42, R.A. No. 165a)

a. Civilb. Criminalc. Administrative

Godines v. CA, G.R. No. 97343, Sept. 13, 1993Facts: Letters Patent UM-2236 was issued by Philippine Patent Office to Villaruz covering a utility model

for a hand tractor or power tiller. Patent was acquired by SV-Agro Industries (private respondent) from Villaruz and SV-Agro caused

the publication of the patent in Bulletin Today PR SV-Agro manufactured and sold the patented power tillers with the patent imprinted on them.

They however suffered a decline in sales of more than 50% in Zamboanga del Sur. Upon investigation, it discovered that power tillers similar to those patented by private respondent

were being manufactured and sold by petitioner. PR notified Godines of the existing patent and demanded that Godines stop selling and manufacturing similar power tillers.

RTC held Godines liable for infringement of patent and unfair competition. CA affirmed stating that “Godines is principally a manufacturer of power tillers, not upon specification and design of buyers, but upon his own specification and design; 2) it would be unbelievable that defendant would fabricate power tillers similar to the turtle power tillers of plaintiff upon specifications of buyers without requiring a job order where the specification and designs of those ordered are specified.

Petitioner maintains the defenses which he raised before the trial and appellate courts, to wit: that he was not engaged in the manufacture and sale of the power tillers as he made them only upon the special order of his customers who gave their own specifications; and those made by him were different from those being manufactured and sold by private respondent

 Issue: W/N petitioner’s product infringed upon patent of SV-Agro Held: YES. Tests have been established to determine infringement. These are (a) literal infringement; and (b) the doctrine of equivalents. In using literal infringement as a test, ". . . resort must be had, in the first instance, to the words of the claim. If accused matter clearly falls within the claim, infringement is made out and that is the end of it."  To determine whether the particular item falls within the literal meaning of the patent claims, the court must juxtapose the claims of the patent and the accused product within the overall context of the claims and specifications, to determine whether there is exact identity of all material elements.

In appearance and form, both the floating power tillers of the defendant and the turtle power tiller of the plaintiff are virtually the same. Viewed from any perspective or angle, the power tiller of the defendant is identical and similar to that of the turtle power tiller of plaintiff in form, configuration, design and appearance. The parts or components thereof are virtually the same. In operation, the floating power tiller of the defendant operates also in similar manner as the turtle power tiller of plaintiff. Moreover, it is also observed that petitioner also called his power tiller as a floating power tiller.

Doctrine of equivalents which recognizes that minor modifications in a patented invention are sufficient to put the item beyond the scope of literal infringement. Thus, according to this doctrine,

15

Page 16: IP Developed Reviewer

"(a)n infringement also occurs when a device appropriates a prior invention by incorporating its innovative concept and, albeit with some modification and change, performs substantially the same function in substantially the same way to achieve substantially the same result."  Careful examination between the two power tillers will show that they will operate on the same fundamental principles. And, according to establish jurisprudence, in infringement of patent, similarities or differences are to be determined, not by the names of things, but in the light of what elements do, and substantial, rather than technical, identity in the test. More specifically, it is necessary and sufficient to constitute equivalency that the same function can be performed in substantially the same way or manner, or by the same or substantially the same, principle or mode of operation; but where these tests are satisfied, mere differences of form or name are immaterial. It is not essential to show that the defendant adopted the device or process in every particular; Proof of an adoption of the substance of the thing will be sufficient.

Smith Kline Beckman v. CA, G.R. No. 126627, Aug. 14, 2003

Facts:•    Smith Kline is a US corporation licensed to do business in the Philippines and it filed before the

Bureau of Patents, as assignee, an application for patent over an invention entitled “Methods and Compositions for Producing Biphasic Parasiticide Activity Using Methyl 5 Propylthio-2-Benzimidazole Carbamate.”

•    Letters Patent No. 14561 for the aforesaid invention was issued to petitioner for 17 years and the same provides that patented invention consisted of a new compound named methyl 5 propylthio-2-benzimidazole carbamate and this is an active ingredient in fighting infections caused by gastrointestinal parasites and lungworms in animals.

•    Tryco Pharma distributes and sells vet products, one of which is Impregon, which has Albendazole, an ingredient claimed effective against gastro-intestinal roundworms, lungworms, tapeworms and fluke infestation in animals.

•    Smith Kline sued PR for infringement of patent and unfair competition claiming that its patent covers the substance Albendazole and that by PR’s manufacture, sale and use without its authorization it violated Letters Patent No. 14561.

•    RTC issued TRO and subsequently a preliminary injunction order against Tryco.•    PR claims that

o    nowhere in Letters Patent No. 14561 does it appear that Albendazole is covered for nowhere does that word appear in the Letters Patent

o    even if the patent were to include Albendazole, such substance is unpatentableo    Bureau of Food and Drugs allowed it to manufacture and market Impregon with Albendazole

as its known ingrediento    no proof that it passed off in any way its veterinary products as those of petitionero    Letters Patent No. 14561 is null and void, the application for the issuance thereof having been

filed beyond the one year period from the filing of an application abroad for the same invention covered thereby, in violation of Section 15 of Republic Act No. 165 (The Patent Law)

o    petitioner is not the registered patent holder (but an assignee)•    Tryco filed counterclaim with damages.•    RTC: In favor of PR. Letters Patent 14561 declared null and void for violating Secs. 7,9 and 15 of

Patents Law and ordered Director of Bureau of Patent to cancel the same.•    CA: Upheld the RTC’s finding that private respondent was not liable for any infringement of the

patent of petitioner in light of the latter’s failure to show that Albendazole is the same as the

16

Page 17: IP Developed Reviewer

compound subject of Letters Patent No. 14561. Because Smith Kline was granted a US patent for Albendazole, it considered this as implying that Albendazole is different from methyl 5 propylthio-2-benzimidazole carbamate. It likewise found that private respondent was not guilty of deceiving the public by misrepresenting that Impregon is its product.  BUT that Letters Patent No. 14561 is not void, & was filed well within one year from petitioner’s filing on June 19, 1974 of its Foreign Application Priority Data No. 480,646 in the U.S. covering the same compound subject of Patent Application Serial No. 17280.

Issue: Whether or not Albendazole is included in Smith Kline’s Letters Patent 14561 thus making PR liable for infringement.

Held: NO. Smith Kline’s contention: Under the doctrine of equivalents for determining patent infringement,

Albendazole, the active ingredient it alleges was appropriated by private respondent for its drug Impregon, is substantially the same as methyl 5 propylthio-2-benzimidazole carbamate covered by its patent since both of them are meant to combat worm or parasite infestation in animals. xxx  .  Petitioner adds that the two substances substantially do the same function in substantially the same way to achieve the same results, thereby making them truly identical.  Petitioner thus submits that the appellate court should have gone beyond the literal wordings used in Letters Patent No. 14561, beyond merely applying the literal infringement test, for in spite of the fact that the word Albendazole does not appear in petitioner’s letters patent, it has ably shown by evidence its sameness with methyl 5 propylthio-2-benzimidazole carbamate. xxx   Petitioner concedes

in its Sur-Rejoinder that although methyl 5 propylthio-2-benzimidazole carbamate is not identical with Albendazole, the former is an improvement or improved version of the latter thereby making both substances still substantially the same.

    From a reading of the 9 claims of Letters Patent No. 14561 in relation to the other portions thereof, no mention is made of the compound Albendazole.  All that the claims disclose are:  the covered invention, that is, the compound methyl 5 propylthio-2-benzimidazole carbamate; the compound’s being anthelmintic but nontoxic for animals or its ability to destroy parasites without harming the host animals; and the patented methods, compositions or preparations involving the compound to maximize its efficacy against certain kinds of parasites infecting specified animals.

    When the language of its claims is clear and distinct, the patentee is bound thereby and may not claim anything beyond them. And so are the courts bound which may not add to or detract from the claims matters not expressed or necessarily implied, nor may they enlarge the patent beyond the scope of that which the inventor claimed and the patent office allowed, even if the patentee may have been entitled to something more than the words it had chosen would include

The burden of proof to substantiate a charge for patent infringement rests on the plaintiff. It bears stressing that the mere absence of the word Albendazole in Letters Patent No. 14561 is not determinative of Albendazole’s non-inclusion in the claims of the patent.  While Albendazole is admittedly a chemical compound that exists by a name different from that covered in petitioner’s letters patent, the language of Letter Patent No. 14561 fails to yield anything at all regarding Albendazole.  And no extrinsic evidence had been adduced to prove that Albendazole inheres in petitioner’s patent in spite of its omission therefrom or that the meaning of the claims of the patent embraces the same.

Even using the doctrine of equivalents (see Godines case for definition), a scrutiny of petitioner’s evidence fails to convince this Court of the substantial sameness of petitioner’s patented compound and Albendazole.  While both compounds have the effect of neutralizing parasites in animals, identity

17

Page 18: IP Developed Reviewer

of result does not amount to infringement of patent unless Albendazole operates in substantially the same way or by substantially the same means as the patented compound, even though it performs the same function and achieves the same result. In other words, the principle or mode of operation must be the same or substantially the same.

The doctrine of equivalents thus requires satisfaction of the function-means-and-result test, the patentee having the burden to show that all three components of such equivalency test are met.

While both compounds have the effect of neutralizing parasites in animals, identity of result does not amount to infringement of patent unless Albendazole operates in substantially the same way or by substantially the same means as the patented compound, even though it performs the same function and achieves the same result. In other words, the principle or mode of operation must be the same or substantially the same

5. Who can file action?

Creser v. CA, G.R. No. 118708, Feb. 2, 1998 Facts: Floro International (PR) is a domestic corporation engaged in the manufacture, production,

distribution and sale of military armaments, munitions, airmunitions and other similar materials 1990, PR was granted Letters Patent No. UM-6938 covering an aerial fuze which was published

in the Bureau of Patent’s Official Gazette PR discovered that petitioner submitted samples of its patented aerial fuze to the Armed Forces of

the Philippines (AFP) for testing and was planning to bid and manufacture the same without license or authority from PR. It therefore sent a letter to petitioner advising it of its existing patent and warned of a possible court action

In response to private respondent's demand, petitioner filed a complaint  for injunction and damages arising from the alleged infringement before the RTC claiming to be the first, true and actual inventor of an aerial fuze denominated as "Fuze, PDR 77 CB4" which it developed as early as December 1981 and started supplying the AFP in. Petitioner prayed that a temporary restraining order and/or writ of preliminary injunction be issued enjoining private respondent including any and all persons acting on its behalf from manufacturing, marketing and/or profiting therefrom. Petitioner admits it has no patent.

PR’s contention at RTC: petitioner has no cause of action to file a complaint for infringement against it since it has no patent for the aerial fuze which it claims to have invented; that petitioner's available remedy is to file a petition for cancellation of patent before the Bureau of Patents;

RTC issued TRO stating that plaintiff has amply proven its entitlement to the relief prayed for. It also said that “with regards to the Floro International’s assertion that an action for infringement may only be brought by "anyone possessing right, title or interest to the patented invention," (Section 42, RA 165), this is qualified by Sec. 10, RA 165 to include only "the first true and actual inventor, his heirs, legal representatives or assignees, "this court finds the foregoing to be untenable. Sec. 10 merely enumerates the persons who may have an invention patented which does not necessarily limit to these persons the right to institute an action for infringement

CA reversed RTC and ruled that PR being the patentee has in its favor not only the presumption of validity of its patent, but that of a legal and factual first and true inventor

       Issue: W/N petitioner can file, under Section 42 of the Patent Law (R.A. 165), an action for infringement not as a patentee but as an entity in possession of a right, title or interest in and to the patented invention

18

Page 19: IP Developed Reviewer

 Held: NO. Under Sec. 42 of IPL only the patentee or his successors-in-interest may file an action for infringement. The phrase "anyone possessing any right, title or interest in and to the patented invention" upon which petitioner maintains its present suit, refers only to the patentee's successors-in-interest, assignees or grantees since actions for infringement of patent may be brought in the name of the person or persons interested, whether as patentee, assignees, or as grantees, of the exclusive right.  Moreover, there can be no infringement of a patent until a patent has been issued, since whatever right one has to the invention covered by the patent arises alone from the grant of patent.  In short, a person or entity who has not been granted letters patent over an invention and has not acquired any light or title thereto either as assignee or as licensee, has no cause of action for infringement because the right to maintain an infringement suit depends on the existence of the patent . (no patent, no standing to file suit) 

While petitioner claims to be the first inventor of the aerial fuze, still it has no right of property over the same upon which it can maintain a suit unless it obtains a patent therefor. Under American jurisprudence, an inventor has no common-law right to a monopoly of his invention. He has the right to make, use and vend his own invention, but if he voluntarily discloses it, such as by offering it for sale, the world is free to copy and use it with impunity. A patent, however, gives the inventor the right to exclude all others. As a patentee, he has the exclusive right of making, using or selling the invention. What is petitioner’s remedy?He can, under Section 28 of the aforementioned law, file a petition for cancellation of the patent within 3 years from the publication of said patent with the Director of Patents and raise as ground therefor that the person to whom the patent was issued is not the true and actual inventor. Time barred under old Patent Law. Remedy now lies in Sec. 68 to wit: be declared the true and lawful inventor.

6. Process patents – Burden of Proof (Sec. 78)Section 78. Process Patents; Burden of Proof . - If the subject matter of a patent is a

process for obtaining a product, any identical product shall be presumed to have been obtained through the use of the patented process if the product is 1 new or there is substantial likelihood that the identical product was made by the process and 2the owner of the patent has been unable despite reasonable efforts, to determine the process actually used. In ordering the defendant to prove that the process to obtain the identical product is different from the patented process, the court shall adopt measures to protect, as far as practicable, his manufacturing and business secrets. (n) (note that this is peculiar to a process patent because here there will be a need to go inside the defendant’s working place to prove that there’s infringement of process)

7. Damages (Sec. 79, 80) (read in conjunction with Sec. 46, i.e. inchoate right from moment of publication)

Section 79. Limitation of Action for Damages. - No damages can be recovered for acts of infringement committed more than 4 years before the institution of the action for infringement. (Sec. 43, R.A. No. 165) (if the offense is continuous, damages can be collected only up to this extent)

Section 80. Damages, Requirement of Notice. - Damages cannot be recovered for acts of infringement committed before the infringer had known, or had reasonable

19

Page 20: IP Developed Reviewer

grounds to know of the patent. It is presumed that the infringer had known of the patent if on the patented product, or on the container or package in which the article is supplied to the public, or on the advertising material relating to the patented product or process, are placed the words "Philippine Patent" with the number of the patent. (Sec. 44, R.A. No. 165a)

8. Defense against Infringement (Sec. 81, 61, 82)Infringement suit- invalidity of patent as well as grounds under Sec. 61Petition to cancel a patent- grounds under Sec. 61Section 81. Defenses in Action for Infringement. - In an action for infringement, the

defendant, in addition to other defenses available to him, may show the invalidity of the patent, or any claim thereof, on any of the grounds on which a petition of cancellation can be brought under Section 61 hereof. (Sec. 45, R.A. No. 165)

Section 61. Cancellation of Patents. – 61.1. Any interested person may, upon payment of the required fee, petition to

cancel the patent or any claim thereof, or parts of the claim, on any of the following grounds:(a) That what is claimed as the invention is not new or Patentable;(b) That the patent does not disclose the invention in a manner sufficiently clear

and complete for it to be carried out by any person skilled in the art; or(c) That the patent is contrary to public order or morality.

61.2. Where the grounds for cancellation relate to some of the claims or parts of the claim, cancellation may be effected to such extent only. (Secs. 28 and 29, R.A. No. 165a)

Section 82. Patent Found Invalid May be Cancelled. - In an action for infringement, if the court shall find the patent or any claim to be invalid, it shall cancel the same, and the Director of Legal Affairs upon receipt of the final judgment of cancellation by the court, shall record that fact in the register of the Office and shall publish a notice to that effect in the IPO Gazette. (Sec. 46, R.A. No. 165a)

See Sec. 84- repeated infringer & there’s finality of action - criminally liable

H. Remedies

1. Cancellation Proceedings (Sec. 61) Section 61. Cancellation of Patents. – (patent should be granted & published)61.1. Any interested person may, upon payment of the required fee, petition to

cancel the patent or any claim thereof, or parts of the claim, on any of the following grounds:

(a) That what is claimed as the invention is not new or Patentable;(b) That the patent does not disclose the invention in a manner sufficiently clear

and complete for it to be carried out by any person skilled in the art; or(c) That the patent is contrary to public order or morality.

61.2. Where the grounds for cancellation relate to some of the claims or parts of the claim, cancellation may be effected to such extent only. (Secs. 28 and 29, R.A. No. 165a)

2. Observation by Third Parties (Sec. 47)

20

Page 21: IP Developed Reviewer

Section 47. Observation by Third Parties. - Following the publication of the patent application, any person may present observations in writing concerning the patentability of the invention. Such observations shall be communicated to the applicant who may comment on them. The Office shall acknowledge and put such observations and comment in the file of the application to which it relates. (n)

3. True and Actual Inventor (Sec. 67, 68, 70) (after publication, during the substantive examination, advisory in nature)Section 67. Patent Application by Persons Not Having the Right to a Patent. . –

(patent not yet issued)67.1. If a person referred to in Section 29 (first to file rule) other than the applicant,

is declared by final court order or decision as having the right to the patent, such person may, within 3 months after the decision has become final:(a) Prosecute the application as his own application in place of the applicant;(b) File a new patent application in respect of the same invention;(c) Request that the application be refused; or(d) Seek cancellation of the patent, if one has already been issued.

67.2. The provisions of Subsection 38.2 shall apply mutatis mutandis to a new application filed under Subsection 67. 1(b). (n)

Section 68. Remedies of the True and Actual Inventor. – (patent already issued) If a person, who was deprived of the patent without his consent or through fraud is declared by final court order or decision to be the true and actual inventor, the court shall order for his substitution as patentee, or at the option of the true inventor, cancel the patent, and award actual and other damages in his favor if warranted by the circumstances. (Sec. 33, R.A. No. 165a)

Section 70. Time to File Action in Court. - The actions indicated in Sections 67 and 68 shall be filed within 1 year from the date of publication made in accordance with Sections 44 and 51, respectively. (n)

B. Licensing

1. Voluntary (Sec. 85-92) To promote technology transfer arrangement (TTA) Sec. 4.2 (memorize) 4.2. The term "technology transfer arrangements" refers to contracts or

agreements involving the transfer of systematic knowledge for the manufacture of a product, the application of a process, or rendering of a service including management contracts; and the transfer, assignment or licensing of all forms of intellectual property rights, including licensing of computer software except computer software developed for mass market.

may include franchise agreements that involve a transfer of systematic knowledge/ know-how (does not have to be patented) (eg. Starbucks)

Section 85. Voluntary License Contract. - To encourage the transfer and dissemination of technology, prevent or control practices and conditions that may in particular cases constitute an abuse of intellectual property rights having an adverse effect on

21

Page 22: IP Developed Reviewer

competition and trade, all technology transfer arrangements shall comply with the provisions of this Chapter. (n)

Section 86. Jurisdiction to Settle Disputes on Royalties. - The Director of the Documentation, Information and Technology Transfer Bureau shall exercise quasi-judicial jurisdiction in the settlement of disputes between parties to a technology transfer arrangement arising from technology transfer payments, including the fixing of appropriate amount or rate of royalty. (n)

Section 87. Prohibited Clauses. - Except in cases under Section 91, the following provisions shall be deemed prima facie to have an adverse effect on competition and trade:

87.1. Those which impose upon the licensee the obligation to acquire from a specific source capital goods, intermediate products, raw materials, and other technologies, or of permanently employing personnel indicated by the licensor;

87.2. Those pursuant to which the licensor reserves the right to fix the sale or resale prices of the products manufactured on the basis of the license;

87.3. Those that contain restrictions regarding the volume and structure of production;87.4. Those that prohibit the use of competitive technologies in a non-exclusive

technology transfer agreement;87.5. Those that establish a full or partial purchase option in favor of the licensor;87.6. Those that obligate the licensee to transfer for free to the licensor the inventions

or improvements that may be obtained through the use of the licensed technology;87.7. Those that require payment of royalties to the owners of patents for patents which

are not used;87.8. Those that prohibit the licensee to export the licensed product unless justified for

the protection of the legitimate interest of the licensor such as exports to countries where exclusive licenses to manufacture and/or distribute the licensed product(s) have already been granted;

87.9. Those which restrict the use of the technology supplied after the expiration of the technology transfer arrangement, except in cases of early termination of the technology transfer arrangement due to reason(s) attributable to the licensee;

87.10. Those which require payments for patents and other industrial property rights after their expiration, termination arrangement;

87.11. Those which require that the technology recipient shall not contest the validity of any of the patents of the technology supplier;

87.12. Those which restrict the research and development activities of the licensee designed to absorb and adapt the transferred technology to local conditions or to initiate research and development programs in connection with new products, processes or equipment;

87.13. Those which prevent the licensee from adapting the imported technology to local conditions, or introducing innovation to it, as long as it does not impair the quality standards prescribed by the licensor;

87.14. Those which exempt the licensor for liability for non-fulfilment of his responsibilities under the technology transfer arrangement and/or liability arising from third party suits brought about by the use of the licensed product or the licensed technology; and

87.15. Other clauses with equivalent effects. (Sec. 33-C (2), R.A 165a)Section 88. Mandatory Provisions. - The following provisions shall be included in

22

Page 23: IP Developed Reviewer

voluntary license contracts:88.1. That the laws of the Philippines shall govern the interpretation of the same and in

the event of litigation, the venue shall be the proper court in the place where the licensee has its principal office;

88.2. Continued access to improvements in techniques and processes related to the technology shall be made available during the period of the technology transfer arrangement;

88.3. In the event the technology transfer arrangement shall provide for arbitration, the Procedure of Arbitration of the Arbitration Law of the Philippines or the Arbitration Rules of the United Nations Commission on International Trade Law (UNCITRAL) or the Rules of Conciliation and Arbitration of the International Chamber of Commerce (ICC) shall apply and the venue of arbitration shall be the Philippines or any neutral country; and

88.4. The Philippine taxes on all payments relating to the technology transfer arrangement shall be borne by the licensor. (n)

Section 89. Rights of Licensor. - In the absence of any provision to the contrary in the technology transfer arrangement, the grant of a license shall not prevent the licensor from granting further licenses to third person nor from exploiting the subject matter of the technology transfer arrangement himself. (Sec. 33-B, R.A. 165a)

Section 90. Rights of Licensee. - The licensee shall be entitled to exploit the subject matter of the technology transfer arrangement during the whole term of the technology transfer arrangement. (Sec. 33-C (1), R.A. 165a)

Section 91. Exceptional Cases. - In exceptional or meritorious cases where substantial benefits will accrue to the economy, such as high technology content, increase in foreign exchange earnings, employment generation, regional dispersal of industries and/or substitution with or use of local raw materials, or in the case of Board of Investments, registered companies with pioneer status, exemption from any of the above requirements may be allowed by the Documentation, Information and Technology Transfer Bureau after evaluation thereof on a case by case basis. (n)

Section 92. Non-Registration with the Documentation, Information and Technology Transfer Bureau. - Technology transfer arrangements that conform with the provisions of Sections 86 and 87 need not be registered with the Documentation, Information and Technology Transfer Bureau. Non-conformance with any of the provisions of Sections 87 and 88, however, shall automatically render the technology transfer arrangement unenforceable (capable of ratification?), unless said technology transfer arrangement is approved and registered with the Documentation, Information and Technology Transfer Bureau under the provisions of Section 91 on exceptional cases. (n)

2. Compulsory (Sec. 93-102)Section 93. Grounds for Compulsory Licensing. - The Director of Legal Affairs may

grant a license to exploit a patented invention, even without the agreement of the patent owner, in favor of any person who has shown his capability to exploit the invention, under any of the following circumstances: (read in conjunction with Sec.95)

93.1. National emergency or other circumstances of extreme urgency;93.2. Where the public interest, in particular, national security, nutrition, health or the

23

Page 24: IP Developed Reviewer

development of other vital sectors of the national economy as determined by the appropriate agency of the Government, so requires; or

93.3. Where a judicial or administrative body has determined that the manner of exploitation by the owner of the patent or his licensee is anti-competitive; or

93.4. In case of public non-commercial use of the patent by the patentee, without satisfactory reason;

93.5. If the patented invention is not being worked in the Philippines on a commercial scale, although capable of being worked, without satisfactory reason: Provided, That the importation of the patented article shall constitute working or using the patent. (Secs. 34, 34-A, 34-B, R.A. No. 165a)

added ground by RA 9502: Art.93.6 where demand for patented drugs & meds are not met in adequate extent & reasonable terms as determined by Sec. of DOJ

Section 94. Period for Filing a Petition for a Compulsory License. – 94.1. A compulsory license may not be applied for on the ground stated in Subsection

93.5 (not being worked in the Phil. on commercial scale) before the expiration of a period of 4 years from the date of filing of the application or 3 years from the date of the patent whichever period expires last.

94.2. A compulsory license which is applied for on any of the grounds stated in Subsections 93.2 (public interest) , 93.3 (judicially or administratively declared non- competitive) , and 93.4 (public non-commercial use) and Section 97 (interdependence of patents) may be applied for at any time after the grant of the patent. (Sec. 34(1), R.A. No. 165)

Section 95. Requirement to Obtain a License on Reasonable Commercial Terms. – 95.1. The license will only be granted after the petitioner has made efforts to obtain

authorization from the patent owner on reasonable commercial terms and conditions but such efforts have not been successful within a reasonable period of time.

95.2. The requirement under Subsection 95.1 shall not apply in the following cases:(a) Where the petition for compulsory license seeks to remedy a practice determined

after judicial or administrative process to be anti-competitive;(b) In situations of national emergency or other circumstances of extreme urgency;(c) In cases of public non-commercial use.+ ( (d) RA 9502, Art. 93.6)

95.3. In situations of national emergency or other circumstances of extreme urgency, the right holder shall be notified as soon as reasonably practicable.

95.4. In the case of public non-commercial use, where the government or contractor, without making a patent search, knows or has demonstrable grounds to know that a valid patent is or will be used by or for the government, the right holder shall be informed promptly. (n)

Section 96. Compulsory Licensing of Patents Involving Semi-Conductor Technology. - In the case of compulsory licensing of patents involving semi-conductor technology, the license may only be granted in case of public non-commercial use or to remedy a practice determined after judicial or administrative process to be anti-competitive. (n)

Section 97. Compulsory License Based on Interdependence of Patents. - If the invention protected by a patent, hereafter referred to as the "second patent," within the country cannot be worked without infringing another patent, hereafter referred to

24

Page 25: IP Developed Reviewer

as the "first patent," granted on a prior application or benefiting from an earlier priority, a compulsory license may be granted to the owner of the second patent to the extent necessary for the working of his invention, subject to the following conditions:

97.1. The invention claimed in the second patent involves an important technical advance of considerable economic significance in relation to the first patent;

97.2. The owner of the first patent shall be entitled to a cross-license on reasonable terms to use the invention claimed in the second patent;

97.3. The use authorized in respect of the first patent shall be non-assignable except with the assignment of the second patent; and

97.4. The terms and conditions of Sections 95, 96 and 98 to 100 of this Act. (Sec. 34-C, R.A. No. 165a)

Section 98. Form and Contents of Petition. - The petition for compulsory licensing must be in writing, verified by the petitioner and accompanied by payment of the required filing fee. It shall contain the name and address of the petitioner as well as those of the respondents, the number and date of issue of the patent in connection with which compulsory license is sought, the name of the patentee, the title of the invention, the statutory grounds upon which compulsory license is sought, the ultimate facts constituting the petitioner's cause of action, and the relief prayed for. (Sec. 34-D, R.A. No. 165)

Section 99. Notice of Hearing. – 99.1. Upon filing of a petition, the Director of Legal Affairs shall forthwith serve notice of

the filing thereof upon the patent owner and all persons having grants or licenses, or any other right, title or interest in and to the patent and invention covered thereby as appears of record in the Office, and of notice of the date of hearing thereon, on such persons and petitioner. The resident agent or representative appointed in accordance with Section 33 hereof, shall be bound to accept service of notice of the filing of the petition within the meaning of this Section.

99.2. In every case, the notice shall be published by the said Office in a newspaper of general circulation, once a week for 3 consecutive weeks and once in the IPO Gazette at applicant's expense. (Sec. 34-E, R.A. No. 165)

Section 100. Terms and Conditions of Compulsory License. - The basic terms and conditions including the rate of royalties of a compulsory license shall be fixed by the Director of Legal Affairs subject to the following conditions:

100.1. The scope and duration of such license shall be limited to the purpose for which it was authorized;

100.2. The license shall be non-exclusive;100.3. The license shall be non-assignable, except with that part of the enterprise or

business with which the invention is being exploited;100.4. Use of the subject matter of the license shall be devoted predominantly for the

supply of the Philippine market: Provided, That this limitation shall not apply where the grant of the license is based on the ground that the patentee's manner of exploiting the patent is determined by judicial or administrative process, to be anti-competitive.

100.5. The license may be terminated upon proper showing that circumstances which led to its grant have ceased to exist and are unlikely to recur: Provided, That adequate protection shall be afforded to the legitimate interest of the licensee; and

25

Page 26: IP Developed Reviewer

100.6. The patentee shall be paid adequate remuneration taking into account the economic value of the grant or authorization, except that in cases where the license was granted to remedy a practice which was determined after judicial or administrative process, to be anti-competitive, the need to correct the anti-competitive practice may be taken into account in fixing the amount of remuneration. (Sec. 35-B, R.A. No. 165a)

Section 101. Amendment, Cancellation, Surrender of Compulsory License. – 101.1. Upon the request of the patentee or the licensee, the Director of Legal Affairs

may amend the decision granting the compulsory license, upon proper showing of new facts or circumstances justifying such amendment.

101.2. Upon the request of the patentee, the said Director may cancel the compulsory license:

(a) If the ground for the grant of the compulsory license no longer exists and is unlikely to recur;

(b) If the licensee has neither begun to supply the domestic market nor made serious preparation therefor;

(c) If the licensee has not complied with the prescribed terms of the license;101.3. The licensee may surrender the license by a written declaration submitted to the

Office.101.4. The said Director shall cause the amendment, surrender, or cancellation in the

Register, notify the patentee, and/or the licensee, and cause notice thereof to be published in the IPO Gazette. (Sec. 35-D, R.A. No. 165a)

Section 102. Licensee's Exemption from Liability. - Any person who works a patented product, substance and/or process under a license granted under this Chapter, shall be free from any liability for infringement: Provided however, That in the case of voluntary licensing, no collusion with the licensor is proven. This is without prejudice to the right of the rightful owner of the patent to recover from the licensor whatever he may have received as royalties under the license. (Sec. 35-E, R.A. No. 165a)

III. Copyright Originality - originate from its author, not copied, involve some intellectual effort

+ Expression The object & the copyright are separate Protects the expression & not the idea

A. Definition of Copyright (Sec. 177, Rule 2, Copyright Safeguards and Regulations, Sec. 181)

Copyright is a right granted by a statute to the author or originator of literary, scholarly, scientific or artistic productions, including computer programs. A copyright gives him the legal right to determine how the work is used and to obtain economic benefits from the work. For example, the owner of a copyright for a book or a piece of software has the exclusive right to use, copy, distribute, and sell copies of the work, including later editions or versions of the work. If another person improperly uses material covered by a copyright, the copyright owner can obtain legal relief.Section 177. Copyright or Economic Rights. (memorize)- Subject to the provisions of Chapter

VIII, copyright or economic rights shall consist of the exclusive right to carry out, authorize or prevent the following acts:

26

Page 27: IP Developed Reviewer

177.1. Reproduction of the work or substantial portion of the work;177.2. Dramatization, translation, adaptation, abridgment, arrangement or other transformation

of the work;177.3. The first public distribution of the original and each copy of the work by sale or other

forms of transfer of ownership;177.4. Rental of the original or a copy of an audiovisual or cinematographic work, a work

embodied in a sound recording, a computer program, a compilation of data and other materials or a musical work in graphic form, irrespective of the ownership of the original or the copy which is the subject of the rental; (n)

177.5. Public display of the original or a copy of the work; 177.6. Public performance of the work; and (read in conjunction with Sec. 171.6)177.7. Other communication to the public of the work. (Sec. 5, P. D. No. 49a) (see Sec. 171.3)

Section 181. Copyright and Material Object. - The copyright is distinct from the property in the material object subject to it. Consequently, the transfer or assignment of the copyright shall not itself constitute a transfer of the material object. Nor shall a transfer or assignment of the sole copy or of one or several copies of the work imply transfer or assignment of the copyright. (Sec. 16, P.D. No. 49)

B. Literary and Artistic Works (Sec. 172.1)Section 172. Literary and Artistic Works. – 172.1. Literary and artistic works, hereinafter referred to as "works", are original intellectual

creations in the literary and artistic domain protected from the moment of their creation and shall include in particular:(a) Books, pamphlets, articles and other writings;(b) Periodicals and newspapers;(c) Lectures, sermons, addresses, dissertations prepared for oral delivery, whether or not

reduced in writing or other material form;(d) Letters;(e) Dramatic or dramatico-musical compositions; choreographic works or entertainment in

dumb shows;(f) Musical compositions, with or without words;(g) Works of drawing, painting, architecture, sculpture, engraving, lithography or other

works of art; models or designs for works of art;(h) Original ornamental designs or models for articles of manufacture, whether or not

registrable as an industrial design, and other works of applied art;(i) Illustrations, maps, plans, sketches, charts and three-dimensional works relative to

geography, topography, architecture or science;(j) Drawings or plastic works of a scientific or technical character;(k) Photographic works including works produced by a process analogous to photography;

lantern slides;(l) Audiovisual works and cinematographic works and works produced by a process

analogous to cinematography or any process for making audio-visual recordings;(m) Pictorial illustrations and advertisements;(n) Computer programs; and(o) Other literary, scholarly, scientific and artistic works.

C. Existence of Copyright (Sec. 172.1 and 172.2)

27

Page 28: IP Developed Reviewer

Sec. 172.1 (see above)172.2. Works are protected by the sole fact of their creation, irrespective of their mode or form

of expression, as well as of their content, quality and purpose. (Sec. 2, P.D. No. 49a)

D. Scope of Copyright

1. Derivative Works (Sec. 173,174)Section 173. Derivative Works. – (there is an underlying work)173.1. The following derivative works shall also be protected by copyright:

(a) Dramatizations, translations, adaptations, abridgments, arrangements, and other alterations of literary or artistic works; and

(b) Collections of literary, scholarly or artistic works, and compilations of data and other materials which are original by reason of the selection or coordination or arrangement of their contents. (Sec. 2, [P] and [Q], P.D. No. 49)

173.2. The works referred to in paragraphs (a) and (b) of Subsection 173.1 shall be protected as new works: Provided however, That such new work shall not affect the force of any subsisting copyright upon the original works employed or any part thereof, or be construed to imply any right to such use of the original works, or to secure or extend copyright in such original works. (Sec. 8, P.D. 49; Art. 10, TRIPS)

Section 174. Published Edition of Work. - In addition to the right to publish granted by the author, his heirs, or assigns, the publisher shall have a copyright consisting merely of the right of reproduction of the typographical arrangement of the published edition of the work. (n)

2. Unprotected Subject Matter (Sec. 175, 176)Section 175. Unprotected Subject Matter. - Notwithstanding the provisions of Sections

172 and 173, no protection shall extend, under this law, to 1any idea, procedure, system, method or operation, concept, principle, discovery or mere data as such, even if they are expressed, explained, illustrated or embodied in a work; 2news of the day and other miscellaneous facts having the character of mere items of press information; or any 3official text of a legislative, administrative or legal nature, as well as any official translation thereof (n)

Section 176. Works of the Government. – 176.1. No copyright shall subsist in any work of the Government of the Philippines.

However, prior approval of the government agency or office wherein the work is created shall be necessary for exploitation of such work for profit. Such agency or office may, among other things, impose as a condition the payment of royalties. No prior approval or conditions shall be required for the use of any purpose of statutes, rules and regulations, and speeches, lectures, sermons, addresses, and dissertations, pronounced, read or rendered in courts of justice, before administrative agencies, in deliberative assemblies and in meetings of public character. (Sec. 9, first par., P.D. No. 49)

176.2. The author of speeches, lectures, sermons, addresses, and dissertations mentioned in the preceding paragraphs shall have the exclusive right of making a collection of his works. (n)

176.3. Notwithstanding the foregoing provisions, the Government is not precluded from receiving and holding copyrights transferred to it by assignment, bequest or

28

Page 29: IP Developed Reviewer

otherwise; nor shall publication or republication by the Government in a public document of any work in which copyright is subsisting be taken to cause any abridgment or annulment of the copyright or to authorize any use or appropriation of such work without the consent of the copyright owner. (Sec. 9, third par., P.D. No. 49)

Joaquin v. Drilon, GR No. 108946, Jan. 28, 1999 Facts: Petitioner BJ Productions, Inc. (BJPI) is the holder/grantee of Certificate of Copyright No. M922 of

Rhoda and Me, a dating game show aired from 1970 to 1977. Petitioner BJPI submitted to the National Library an addendum to its certificate of copyright

specifying the show's format and style of presentation. While watching tv, petitioner Joaquin, Jr., president of BJPI, saw on RPN Channel 9 an episode of

It's a Date, which was produced by IXL Productions, Inc. (IXL). Joaquin wrote a letter to private respondent Zosa, president and general manager of IXL, informing

Zosa that BJPI had a copyright to Rhoda and Me and demanding that IXL discontinue airing It's a Date.

In a letter, private respondent Zosa apologized to petitioner Joaquin and requested a meeting to discuss a possible settlement.

IXL, however, continued airing It's a Date, prompting petitioner Joaquin to send a second letter in which he reiterated his demand and warned that, if IXL did not comply, he would endorse the matter to his attorneys for proper legal action.

Meanwhile, private respondent Zosa sought to register IXL's copyright to the first episode of It's a Date for which it was issued by the National Library a certificate of copyright.

Upon complaint of petitioners, an information for violation of PD 49 was filed against private respondent Zosa together with certain officers of RPN Channel 9 in the RTC of QC.

However, private respondent Zosa sought a review of the resolution of the Assistant City Prosecutor before the DOJ.

DOJ Sec. Drilon reversed the Assistant City Prosecutor's findings and directed him to move for the dismissal of the case against private respondents. MR denied.

Both public and private respondents maintain that petitioners failed to establish the existence of probable cause due to their failure to present the copyrighted master videotape of Rhoda and Me. They contend that petitioner BJPI's copyright covers only a specific episode of Rhoda and Me and that the formats or concepts of dating game shows are not covered by copyright protection under PD 49.

Issues/ Held:1) Non-assignment of error – Sec. of Justice not confined to issues submitted to him

Petitioners claim that their failure to submit the copyrighted master videotape of the television show Rhoda and Me was not raised in issue by private respondents during the preliminary investigation and, therefore, it was error for the Secretary of Justice to reverse the investigating prosecutor's finding of probable cause on this ground.

In reviewing resolutions of prosecutors, the Secretary of Justice is not precluded from considering errors, although unassigned, for the purpose of determining whether there is probable cause for filing cases in court. He must make his own finding, of probable cause and is not confined to the issues raised by the parties during preliminary investigation. Moreover, his findings are not subject to review unless shown to have been made with grave abuse.

29

Page 30: IP Developed Reviewer

2) Opinion of Secretary of Justice – Sec. of Justice not precluded from making PIPetitioners contend, however, that the determination of the question whether the format or

mechanics of a show is entitled to copyright protection is for the court, and not the Secretary of Justice, to make.

It is indeed true that the question whether the format or mechanics of petitioners television show is entitled to copyright protection is a legal question for the court to make. This does not, however, preclude respondent Secretary of Justice from making a preliminary determination of this question in resolving whether there is probable cause for filing the case in court. In doing so in this case, he did not commit any grave error.

3) Presentation of Master Tape – should have presented(And is a dating show format copyrightable? NO)

Petitioners claim that respondent Secretary of Justice gravely abused his discretion in ruling that the master videotape should have been presented in order to determine whether there was probable cause for copyright infringement.

Petitioners assert that the format of Rhoda and Me is a product of ingenuity and skill and is thus entitled to copyright protection. It is their position that the presentation of a point-by-point comparison of the formats of the two shows clearly demonstrates the nexus between the shows and hence establishes the existence of probable cause for copyright infringement. Such being the case, they did not have to produce the master tape.

To begin with the format of a show is not copyrightable . Section 2 of PD 49, otherwise known as the DECREE ON INTELLECTUAL PROPERTY, enumerates the classes of work entitled to copyright protection (same as Sec. 172 of IP Code).

PD 49, Sec. 2, in enumerating what are subject to copyright, refers to finished works and not to concepts. The copyright does not extend to an idea, procedure, process, system, method of operation, concept, principle, or discovery, regardless of the form in which it is described, explained, illustrated, or embodied in such work. (see Sec. 175 of IP Code)

What then is the subject matter of petitioners' copyright?This Court is of the opinion that petitioner BJPI's copyright covers audio-visual recordings of

each episode of Rhoda and Me , as falling within the class of works mentioned in P.D. 49, Sec. 2(M),

E. Copyright Ownership (Sec. 178)Section 178. Rules on Copyright Ownership. - Copyright ownership shall be governed by

the following rules:178.1 Subject to the provisions of this section, in the case of original literary and artistic

works, copyright shall belong to the author of the work;

1. Joint authorship178.2. In the case of works of joint authorship, the co-authors shall be the original owners

of the copyright and in the absence of agreement, their rights shall be governed by the rules on co-ownership. If, however, a work of joint authorship consists of parts that can be used separately and the author of each part can be identified, the author of each part shall be the original owner of the copyright in the part that he has created;

2. Employment

30

Page 31: IP Developed Reviewer

178.3. In the case of work created by an author during and in the course of his employment, the copyright shall belong to:(a) The employee, if the creation of the object of copyright is not a part of his regular

duties even if the employee uses the time, facilities and materials of the employer.(b) The employer, if the work is the result of the performance of his regularly-assigned

duties, unless there is an agreement, express or implied, to the contrary.

3. Commissioned Works (works for hire, no ER-EE relationship)178.4. In the case of a work commissioned by a person other than an employer of the

author and who pays for it and the work is made in pursuance of the commission, the person who so commissioned the work shall have ownership of the work, but the copyright thereto shall remain with the creator, unless there is a written stipulation to the contrary;

4. Audio-Visual Works178.5. In the case of audiovisual work, the copyright shall belong to the producer, the

author of the scenario, the composer of the music, the film director, and the author of the work so adapted. However, subject to contrary or other stipulations among the creators, the producer shall exercise the copyright to an extent required for the exhibition of the work in any manner, except for the right to collect performing license fees for the performance of musical compositions, with or without words, which are incorporated into the work; and

178.6. In respect of letters, the copyright shall belong to the writer subject to the provisions of Article 723 of the Civil Code. (Sec. 6, P.D. No. 49a)

F. Term of Copyright Protection (Sec. 213, 214, 215) Section 213. Term of Protection. – 213.1. Subject to the provisions of Subsections 213.2 to 213.5, the copyright in works under

Sections 172 and 173 shall be protected during the life of the author and for 50 years after his death. This rule also applies to posthumous works. (Sec. 21, first sentence, P.D. No. 49a)

213.2. In case of works of joint authorship, the economic rights shall be protected during the life of the last surviving author and for 50 years after his death. (Sec. 21, second sentence, P.D. No. 49)

213.3. In case of anonymous or pseudonymous works, the copyright shall be protected for 50 years from the date on which the work was first lawfully published: Provided, That where, before the expiration of the said period, the author's identity is revealed or is no longer in doubt, the provisions of Subsections 213.1. and 213.2 shall apply, as the case may be: Provided, further, That such works if not published before shall be protected for fifty 50 years counted from the making of the work. (Sec. 23, P.D. No. 49)

213.4. In case of works of applied art (see Sec 172.1 (h)) the protection shall be for a period of 25 years from the date of making. (Sec. 24(B), P.D. No. 49a)

213.5. In case of photographic works, the protection shall be for 50 years from publication of the work and, if unpublished, fifty 50 years from the making. (Sec. 24(C), P.D. 49a)

213.6. In case of audio-visual works including those produced by process analogous to photography or any process for making audio-visual recordings, the term shall be 50 years from date of publication and, if unpublished, from the date of making. (Sec. 24(C), P.D. No.

31

Page 32: IP Developed Reviewer

49a)Section 214. Calculation of Term. - The term of protection subsequent to the death of the

author provided in the preceding Section shall run from the date of his death or of publication, but such terms shall always be deemed to begin on the first day of January of the year following the event which gave rise to them. (Sec. 25, P.D. No. 49)

Section 215. Term of Protection for Performers, Producers and Broadcasting Organizations. – 215.1. The rights granted to performers and producers of sound recordings under this law shall

expire:(a) For performances not incorporated in recordings, 50 years from the end of the year in

which the performance took place; and(b) For sound or image and sound recordings and for performances incorporated therein,

50 years from the end of the year in which the recording took place.215.2. In case of broadcasts, the term shall be 20 years from the date the broadcast took

place. The extended term shall be applied only to old w w0vorks with subsisting protection under the prior law. (Sec. 55, P.D. No. 49a)

G.Limitations on Copyright (Sec. 184-190)Section 184. Limitations on Copyright. – 184.1. Notwithstanding the provisions of Chapter V, the following acts shall not constitute

infringement of copyright:(a) The recitation or performance of a work, once it has been lawfully made accessible to

the public, if 1done privately and free of charge or if 2made strictly for a charitable or religious institution or society; (Sec. 10(1), P.D. No. 49)

(b) The making of quotations from a published work if they are compatible with fair use and only to the extent justified for the purpose , including quotations from newspaper articles and periodicals in the form of press summaries: Provided, That the source and the name of the author, if appearing on the work, are mentioned; (Sec. 11, third par., P.D. No. 49)

(c) The reproduction or communication to the public by mass media of articles on current political, social, economic, scientific or religious topic, lectures, addresses and other works of the same nature, which are delivered in public if such use is for information purposes and has not been expressly reserved: Provided, That the source is clearly indicated; (Sec. 11, P.D. No. 49) (read in relation to (d) )

(d) The reproduction and communication to the public of literary, scientific or artistic works as part of reports of current events by means of photography, cinematography or broadcasting to the extent necessary for the purpose; (Sec. 12, P.D. No. 49)

(e) The inclusion of a work in a publication, broadcast, or other communication to the public, sound recording or film, if such inclusion is made by way of illustration for teaching purposes and is compatible with fair use : Provided, That the source and of the name of the author, if appearing in the work, are mentioned;

(f) The recording made in schools, universities, or educational institutions of a work included in a broadcast for the use of such schools, universities or educational institutions: Provided, That such recording must be deleted within a reasonable period after they were first broadcast: Provided, further, That such recording may not be made from audiovisual works which are part of the general cinema repertoire of feature films except for brief excerpts of the work;

(g) The making of ephemeral recordings by a broadcasting organization by means of its

32

Page 33: IP Developed Reviewer

own facilities and for use in its own broadcast;(h) The use made of a work by or under the direction or control of the Government, by the

National Library or by educational, scientific or professional institutions where such use is in the public interest and is compatible with fair use ;

(i) The public performance or the communication to the public of a work, in a place where no admission fee is charged in respect of such public performance or communication, by a club or institution for charitable or educational purpose only, whose aim is not profit making, subject to such other limitations as may be provided in the Regulations; (n)

(j) Public display of the original or a copy of the work not made by means of a film, slide, television image or otherwise on screen or by means of any other device or process: Provided, That either the work has been published, or, that the original or the copy displayed has been sold, given away or otherwise transferred to another person by the author or his successor in title; and

(k) Any use made of a work for the purpose of any judicial proceedings or for the giving of professional advice by a legal practitioner.

184.2. The provisions of this section shall be interpreted in such a way as to allow the work to be used in a manner which does not conflict with the normal exploitation of the work and does not unreasonably prejudice the right holder's legitimate interests.

Section 185. Fair Use of a Copyrighted Work. – 185.1. The fair use of a copyrighted work for 1 criticism, comment, news reporting, 2 teaching

including multiple copies for classroom use, scholarship, research, and similar purposes is not an infringement of copyright. Decompilation, which is understood here to be the reproduction of the code and translation of the forms of the computer program to achieve the inter-operability of an independently created computer program with other programs may also constitute fair use (see Sega vs. Accolade). In determining whether the use made of a work in any particular case is fair use, the factors to be considered shall include: (memorize: P.A.N.E.)(a) The purpose and character of the use, including whether such use is of a commercial

nature or is for non-profit educational purposes;(b) The nature of the copyrighted work;(c) The amount and substantiality of the portion used in relation to the copyrighted work as

a whole; and(d) The effect of the use upon the potential market for or value of the copyrighted work.

185.2. The fact that a work is unpublished shall not by itself bar a finding of fair use if such finding is made upon consideration of all the above factors.

Section 186. Work of Architecture. - Copyright in a work of architecture shall include the right to control the erection of any building which reproduces the whole or a substantial part of the work either in its original form or in any form recognizably derived from the original: Provided, That the copyright in any such work shall not include the right to control the reconstruction or rehabilitation in the same style as the original of a building to which that copyright relates. (n)

Section 187. Reproduction of Published Work. – 187.1. Notwithstanding the provision of Section 177, and subject to the provisions of

Subsection 187.2, the private reproduction of a published work in a single copy, where the reproduction is made by a natural person exclusively for research and private study, shall be permitted, without the authorization of the owner of copyright in the work.

33

Page 34: IP Developed Reviewer

187.2. The permission granted under Subsection 187.1 shall not extend to the reproduction of:(a) A work of architecture in the form of building or other construction;(b) An entire book, or a substantial part thereof, or of a musical work in graphic form by

reprographic means;(c) A compilation of data and other materials;(d) A computer program except as provided in Section 189; and(e) Any work in cases where reproduction would unreasonably conflict with a normal

exploitation of the work or would otherwise unreasonably prejudice the legitimate interests of the author. (n)

Section 188. Reprographic Reproduction by Libraries. – 188.1. Notwithstanding the provisions of Subsection 177.6, any library or archive whose

activities are not for profit may, without the authorization of the author of copyright owner, make a single copy of the work by reprographic reproduction:(a) Where the work by reason of its fragile character or rarity cannot be lent to user in its

original form;(b) Where the works are isolated articles contained in composite works or brief portions of

other published works and the reproduction is necessary to supply them, when this is considered expedient, to persons requesting their loan for purposes of research or study instead of lending the volumes or booklets which contain them; and

(c) Where the making of such a copy is in order to preserve and, if necessary in the event that it is lost, destroyed or rendered unusable, replace a copy, or to replace, in the permanent collection of another similar library or archive, a copy which has been lost, destroyed or rendered unusable and copies are not available with the publisher.

188.2. Notwithstanding the above provisions, it shall not be permissible to produce a volume of a work published in several volumes or to produce missing tomes or pages of magazines or similar works, unless the volume, tome or part is out of stock: Provided, That every library which, by law, is entitled to receive copies of a printed work, shall be entitled, when special reasons so require, to reproduce a copy of a published work which is considered necessary for the collection of the library but which is out of stock. (Sec. 13, P.D. 49a)

Section 189. Reproduction of Computer Program. – 189.1. Notwithstanding the provisions of Section 177, the reproduction of 1 back-up copy or

adaptation of a computer program shall be permitted, without the authorization of the author of, or other owner of copyright in, a computer program, by the lawful owner of that computer program: Provided, That the copy or adaptation is necessary for:(a) The use of the computer program in conjunction with a computer for the purpose, and

to the extent, for which the computer program has been obtained; and(b) Archival purposes, and, for the replacement of the lawfully owned copy of the computer

program in the event that the lawfully obtained copy of the computer program is lost, destroyed or rendered unusable.

189.2. No copy or adaptation mentioned in this Section shall be used for any purpose other than the ones determined in this Section, and any such copy or adaptation shall be destroyed in the event that continued possession of the copy of the computer program ceases to be lawful.

189.3. This provision shall be without prejudice to the application of Section 185 whenever appropriate. (n)

Section 190. Importation for Personal Purposes. – 190.1. Notwithstanding the provision of Subsection 177.6 (should be 177.3), but subject to the

34

Page 35: IP Developed Reviewer

limitation under the Subsection 185.2 (should be 184.2), the importation of a copy of a work by an individual for his personal purposes shall be permitted without the authorization of the author of, or other owner of copyright in, the work under the following circumstances:

(1. not available in the Philippines; 2. suitcase exemption)

(a) When copies of the work are not available in the Philippines and:(i) Not more than one 1 copy at one time is imported for strictly individual use only; or(ii) The importation is by authority of and for the use of the Philippine Government; or(iii) The importation, consisting of not more than 3 such copies or likenesses in any one

invoice, is not for sale but for the use only of any religious, charitable, or educational society or institution duly incorporated or registered, or is for the encouragement of the fine arts, or for any state school, college, university, or free public library in the Philippines.

(b) When such copies form parts of libraries and personal baggage belonging to persons or families arriving from foreign countries and are not intended for sale: Provided, That such copies do not exceed 3.

190.2. Copies imported as allowed by this Section may not lawfully be used in any way to violate the rights of owner the copyright or annul or limit the protection secured by this Act, and such unlawful use shall be deemed an infringement and shall be punishable as such without prejudice to the proprietor's right of action.

190.3. Subject to the approval of the Secretary of Finance, the Commissioner of Customs is hereby empowered to make rules and regulations for preventing the importation of articles the importation of which is prohibited under this Section and under treaties and conventions to which the Philippines may be a party and for seizing and condemning and disposing of the same in case they are discovered after they have been imported. (Sec. 30, P.D. No. 49)

H. Transfer of Copyright (Sec. 180, 181, 182, 183)Section 180. Rights of Assignee. – 180.1. The copyright may be assigned in whole or in part. Within the scope of the assignment,

the assignee is entitled to all the rights and remedies, which the assignor had with respect to the copyright.

180.2. The copyright is not deemed assigned inter vivos in whole or in part unless there is a written indication of such intention.

180.3. The submission of a literary, photographic or artistic work to a newspaper, magazine or periodical for publication shall constitute only a license to make a single publication unless a greater right is expressly granted. If 2 or more persons jointly own a copyright or any part thereof, neither of the owners shall be entitled to grant licenses without the prior written consent of the other owner or owners. (Sec. 15, P.D. No. 49a)

Section 181. Copyright and Material Object. - The copyright is distinct from the property in the material object subject to it. Consequently, the transfer or assignment of the copyright shall not itself constitute a transfer of the material object. Nor shall a transfer or assignment of the sole copy or of one or several copies of the work imply transfer or assignment of the copyright. (Sec. 16, P.D. No. 49)

Section 182. Filing of Assignment or License. - An assignment or exclusive license may be filed in duplicate with the National Library upon payment of the prescribed fee for registration in books and records kept for the purpose. Upon recording, a copy of the

35

Page 36: IP Developed Reviewer

instrument shall be returned to the sender with a notation of the fact of record. Notice of the record shall be published in the IPO Gazette. (Sec. 19, P.D. No. 49a)

Section 183. Designation of Society. - The copyright owners or their heirs may designate a society of artists, writers or composers to enforce their economic rights and moral rights on their behalf. (Sec. 32, P.D. No. 49a)

I. Deposit with National Library and Notice (Sec. 191, 192, 227, 228, 229)Section 191. Registration and Deposit with National Library and the Supreme Court Library. -

After the first public dissemination of performance by authority of the copyright owner of a work falling under Subsections 172.1, 172.2 and 172.3 (172.1 a, b, c, & d) of this Act, there shall, for the purpose of completing the records of the National Library and the Supreme Court Library, within 3 weeks, be registered and deposited with it, by personal delivery or by registered mail 2 complete copies or reproductions of the work in such form as the directors of said libraries may prescribe. A certificate of deposit shall be issued for which the prescribed fee shall be collected and the copyright owner shall be exempt from making additional deposit of the works with the National Library and the Supreme Court Library under other laws. If, within 3 weeks after receipt by the copyright owner of a written demand from the directors for such deposit, the required copies or reproductions are not delivered and the fee is not paid, the copyright owner shall be liable to pay a fine equivalent to the required fee per month of delay and to pay to the National Library and the Supreme Court Library the amount of the retail price of the best edition of the work. Only the above mentioned classes of work shall be accepted for deposit by the National Library and the Supreme Court Library. (Sec. 26, P.D. No. 49a)

Section 192. Notice of Copyright. - Each copy of a work published or offered for sale may contain a notice bearing the name of the copyright owner, and the year of its first publication, and, in copies produced after the creator's death, the year of such death. (Sec. 27, P.D. No. 49a)

Section 227. Ownership of Deposit and Instruments. - All copies deposited and instruments in writing filed with the National Library and the Supreme Court Library in accordance with the provisions of this Act shall become the property of the Government. (Sec. 60, P.D. No. 49)

Section 228. Public Records. - The section or division of the National Library and the Supreme Court Library charged with receiving copies and instruments deposited and with keeping records required under this Act and everything in it shall be opened to public inspection. The Director of the National Library is empowered to issue such safeguards and regulations as may be necessary to implement this Section and other provisions of this Act. (Sec. 61, P.D. No. 49)

Section 229. Copyright Division; Fees. - The Copyright Section of the National Library shall be classified as a Division upon the effectivity of this Act. The National Library shall have the power to collect, for the discharge of its services under this Act, such fees as may be promulgated by it from time to time subject to the approval of the Department Head. (Sec. 62, P.D. 49a)

J. Infringement

1. Definition

2. Remedies (Sec. 216-220; 225, 226, 231)

36

Page 37: IP Developed Reviewer

Section 216. Remedies for Infringement. – 216.1. Any person infringing a right protected under this law shall be liable:(a) To an injunction restraining such infringement. The court may also order the defendant to

desist from an infringement, among others, to prevent the entry into the channels of commerce of imported goods that involve an infringement, immediately after customs clearance of such goods.

(b) Pay to the copyright proprietor or his assigns or heirs such actual damages, including legal costs and other expenses, as he may have incurred due to the infringement as well as the profits the infringer may have made due to such infringement, and in proving profits the plaintiff shall be required to prove sales only and the defendant shall be required to prove every element of cost which he claims, or, in lieu of actual damages and profits, such damages which to the court shall appear to be just and shall not be regarded as penalty.

(c) Deliver under oath, for impounding during the pendency of the action, upon such terms and conditions as the court may prescribe, sales invoices and other documents evidencing sales, all articles and their packaging alleged to infringe a copyright and implements for making them.

(d) Deliver under oath for destruction without any compensation all infringing copies or devices, as well as all plates, molds, or other means for making such infringing copies as the court may order.

(e) Such other terms and conditions, including the payment of moral and exemplary damages, which the court may deem proper, wise and equitable and the destruction of infringing copies of the work even in the event of acquittal in a criminal case.

216.2. In an infringement action, the court shall also have the power to order the seizure and impounding of any article which may serve as evidence in the court proceedings. (Sec. 28, P.D. No. 49a)

Section 217. Criminal Penalties. – 217.1. Any person infringing any right secured by provisions of Part IV of this Act or aiding or

abetting such infringement shall be guilty of a crime punishable by:(a) Imprisonment of 1 year to 3 years plus a fine ranging from P50,000 to P150,000 for the

first offense.(b) Imprisonment of 3 years and 1 day to 6 years plus a fine ranging from P150,000 to

P500,000 for the second offense.(c) Imprisonment of 6 years and 1 day to 9 years plus a fine ranging from P500,000 to

P1,500,000 for the third and subsequent offenses.(d) In all cases, subsidiary imprisonment in cases of insolvency.

217.2. In determining the number of years of imprisonment and the amount of fine, the court shall consider the value of the infringing materials that the defendant has produced or manufactured and the damage that the copyright owner has suffered by reason of the infringement.

217.3. Any person who at the time when copyright subsists in a work has in his possession an article which he knows, or ought to know, to be an infringing copy of the work for the purpose of:(a) Selling, letting for hire, or by way of trade offering or exposing for sale, or hire, the article;(b) Distributing the article for purpose of trade, or for any other purpose to an extent that will

prejudice the rights of the copyright owner in the work; or(c) Trade exhibit of the article in public, shall be guilty of an offense and shall be liable on

conviction to imprisonment and fine as above mentioned. (Sec. 29, P.D. No. 49a)

37

Page 38: IP Developed Reviewer

Section 218. Affidavit Evidence. – 218.1. In an action under this Chapter, an affidavit made before a notary public by or on behalf

of the owner of the copyright in any work or other subject matter and stating that:(a) At the time specified therein, copyright subsisted in the work or other subject matter;(b) He or the person named therein is the owner of the copyright; and(c) The copy of the work or other subject matter annexed thereto is a true copy thereof, shall

be admitted in evidence in any proceedings for an offense under this Chapter and shall be prima facie proof of the matters therein stated until the contrary is proved, and the court before which such affidavit is produced shall assume that the affidavit was made by or on behalf of the owner of the copyright.

218.2. In an action under this Chapter:(a) Copyright shall be presumed to subsist in the work or other subject matter to which the action

relates if the defendant does not put in issue the question whether copyright subsists in the work or other subject matter; and

(b) Where the subsistence of the copyright is established, the plaintiff shall be presumed to be the owner of the copyright if he claims to be the owner of the copyright and the defendant does not put in issue the question of his ownership .

(c) Where the defendant, without good faith , puts in issue the questions of whether copyright subsists in a work or other subject matter to which the action relates, or the ownership of copyright in such work or subject matter, thereby occasioning unnecessary costs or delay in the proceedings, the court may direct that any costs to the defendant in respect of the action shall not be allowed by him and that any costs occasioned by the defendant to other parties shall be paid by him to such other parties. (n)

Section 219. Presumption of Authorship. – 219.1. The natural person whose name is indicated on a work in the usual manner as the author

shall, in the absence of proof to the contrary, be presumed to be the author of the work. This provision shall be applicable even if the name is a pseudonym, where the pseudonym leaves no doubt as to the identity of the author.

219.2. The person or body corporate whose name appears on a audio-visual work in the usual manner shall, in the absence of proof to the contrary, be presumed to be the maker of said work. (n)

Section 220. International Registration of Works. - A statement concerning a work, recorded in an international register in accordance with an international treaty to which the Philippines is or may become a party, shall be construed as true until the contrary is proved except:

220.1. Where the statement cannot be valid under this Act or any other law concerning intellectual property.

220.2. Where the statement is contradicted by another statement recorded in the international register. (n)

Section 225. Jurisdiction. - Without prejudice to the provisions of Subsection 7.1(c), actions under this Act shall be cognizable by the courts with appropriate jurisdiction under existing law. (Sec. 57, P.D. No. 49a) (Now it is the RTC that has jurisdiction over copyright infringement based on Samson vs. Daway)

Section 226. Damages. - No damages may be recovered under this Act after 4 years from the time the cause of action arose. (Sec. 58, P.D. No. 49)

Section 230. Equitable Principles to Govern Proceedings. - In all inter partes proceedings in the Office under this Act, the equitable principles of laches, estoppel, and acquiescence where applicable, may be considered and applied. (Sec. 9-A, R.A. No. 165)

38

Page 39: IP Developed Reviewer

20th Century Fox v. CA, G.R. No. L-76649-51, Aug. 19, 1988Facts: 20th Century Fox sought the NBI’s assistance in the conduct of searches and seizures in connection

with the latter's anti-film piracy campaign. The letter-complaint alleged that certain videotape outlets all over Metro Manila are engaged in the

unauthorized sale and renting out of copyrighted films in videotape form which constitute a flagrant violation of PD 49 (a.k.a. the Decree on the Protection of Intellectual Property)

NBI conducted surveillance and investigation of the outlets pinpointed by the petitioner and subsequently filed 3 applications for search warrants against the video outlets owned by the private respondents.

The applications were consolidated and heard by Makati RTC. RTC issued the desired search warrants. Armed with the search warrants, the NBI accompanied by the petitioner's agents, raided the video

outlets and seized the items described therein. An inventory of the items seized was made and left with the private respondents.

Acting on a motion to lift search warrants and release seized properties filed by the private respondents, the lower court issued an order lifting the 3 search warrants issued earlier against the private respondents by the court.

Consequently, the articles seized were ordered to be returned to their owners. Petitioner’s MR denied. Petitioner’s petition for certiorari with CA dismissed. Hence, this petition.

Issue: Whether or not the judge properly lifted the search warrants he issued earlier upon the application of the NBI on the basis of the complaint filed by the petitioner.

Re: meaning of "probable cause" within the context of the constitutional provision against illegal searches and seizures (Section 3, Article IV, 1973 Constitution, now, Section 2, Article Ill, 1987 Constitution.)

In the instant case, RTC lifted the 3 questioned search warrants against the private respondents on the ground that it acted on the application for the issuance of the said search warrants and granted it on the misrepresentations of applicant NBI and its witnesses that infringement of copyright or a piracy of a particular film have been committed.

The lower court, therefore, lifted the 3 questioned search warrants in the absence of probable cause that the private respondents violated PD 49. As found out by the court, the NBI agents who acted as witnesses did not have personal knowledge of the subject matter of their testimony which was the alleged commission of the offense by the private respondents. Only the petitioner's counsel who was also a witness during the application for the issuance of the search warrants stated that he had personal knowledge that the confiscated tapes owned by the private respondents were pirated tapes taken from master tapes belonging to the petitioner. However, the lower court did not give much credence to his testimony in view of the fact that the master tapes of the allegedly pirated tapes were not shown to the court during the application. (construction of probable cause: although what is required for the issuance thereof is merely the presence of probable cause, that probable cause must be satisfactory to the Court, for it is a time- honored precept that proceedings to put a man to task as an offender under our laws should be interpreted in strictissimi juris against the government and liberally in favor of the alleged offender.)

39

Page 40: IP Developed Reviewer

The presentation of the master tapes of the copyrighted films from which the pirated films were allegedly copied, was necessary for the validity of search warrants against those who have in their possession the pirated films (‘Cocoon’ is the title of the first videotape). The petitioner's argument to the effect that the presentation of the master tapes at the time of application may not be necessary as these would be merely evidentiary in nature and not determinative of whether or not a probable cause exists to justify the issuance of the search warrants is not meritorious. The court cannot presume that duplicate or copied tapes were necessarily reproduced from master tapes that it owns.

The application for search warrants was directed against videotape outlets which allegedly were engaged in the unauthorized sale and renting out of copyrighted films belonging to the petitioner pursuant to PD 49.

The essence of a copyright infringement is the similarity or at least substantial similarity of the purported pirated works to the copyrighted work. Hence, the applicant must present to the court the copyrighted films to compare them with the purchased evidence of the video tapes allegedly pirated to determine whether the latter is an unauthorized reproduction of the former. This linkage of the copyrighted films to the pirated films must be established to satisfy the requirements of probable cause. Mere allegations as to the existence of the copyrighted films cannot serve as basis for the issuance of a search warrant.

Furthermore, we note that the search warrants described the articles sought to be seized as follows:xxx xxx xxxc) Television sets, Video Cassettes Recorders, rewinders, tape head cleaners, accessories, equipments and other machines used or intended to be used in the unlawful reproduction, sale, rental/lease distribution of the above-mentioned video tapes which she is keeping and concealing in the premises above-described."

Another factor which makes the search warrants under consideration constitutionally objectionable is that they are in the nature of general warrants.

Television sets, video cassette recorders, reminders and tape cleaners are articles which can be found in a video tape store engaged in the legitimate business of lending or renting out betamax tapes. In short, these articles and appliances are generally connected with, or related to a legitimate business not necessarily involving piracy of intellectual property or infringement of copyright laws. Hence, including these articles without specification and/or particularity that they were really instruments in violating an Anti-Piracy law makes the search warrant too general which could result in the confiscation of all items found in any video store.

Columbia Pictures, Inc. v. Court of Appeals, G.R. No. 96597-99, Oct. 6, 1994Facts: NBI filed with the RTC 3 applications for search warrant against private respondents Tube Video

Enterprises and Edward C. Cham (ASW No. 95), the Blooming Rose Tape Center and Ma. Jajorie T. Uy (ASW No. 96), and the VideoChannel and Lydia Nabong (ASW No. 97), charging said respondents with violation of Section 56 of PD 49, otherwise known as the Decree on the Protection of Intellectual Property, as amended by P.D. No. 1988.

In the three applications for search warrant, NBI Agent Reyes stated under oath that the respondents had in their possession and control:

1. (p)irated video tapes of the copyrighted motion pictures/films the titles of which are mentioned in the attached list;

2. (p)osters, advertising leaflets, flyers, brochures, invoices, journals, ledgers, job order slips, delivery

40

Page 41: IP Developed Reviewer

slips, stickers and books of account bearing and/or mentioned the pirated films with titles . . ., or otherwise used in the videogram business or activities of the defendants; sold, leased, distributed or possessed for the purpose of sale, lease, distribution, circulation or public exhibition, journals, ledgers, job order slips, delivery slips, stickers and books of accounts used in the unlawful videogram business or activities of the defendants; (and)

3. (t)elevision sets, video cassette and/or laser disc recorders, dubbing machines, rewinders, film projectors, U-matic machines, image enhancers, dubbing machines, tape head cleaners, converters, accessories, equipment and other machines and paraphernalia, materials or empty/erasable video tapes and master copies used or intended to be used in the unlawful exhibition, showing, reproduction, sale lease or disposition of videograms they are keeping and concealing in the premises abovedescribed.

Acting on the applications, then RTC Judge Austria conducted a joint hearing during which she made a personal examination of the applicant and his witnesses.

Finding just and probable cause for granting the application at the time, Judge Austria issued the corresponding Search Warrants ("SW") numbered 95, 96, and 97.

Private respondents filed their respective motions to quash the three search warrants, citing as grounds therefor the following:

In SW No. 951. There is no probable cause upon which the search warrant is based;2. The NBI has no authority nor the jurisdiction to initiate the filing of suit against the defendants;3. The confiscation of defendants' seized articles based on the questioned search warrant violated

the latter's constitutional right against deprivation of properties without due process.4. The films in question are not protected by PD 1988 in that they were never registered in the

National Library as a condition precedent to the availment of the protection secured by that decree. The complaint has acquired no right under the same.

5. The mere publication by complainant of its alleged ownership over the films in question does not ipso facto vest in the right to proceed under P.D. No. 49 as that law requires official registration. Moreover, the said publication took place only after the application for the questioned search warrant.

In SW No. 961. The complainants, one Domingo and one Baltazar, in representation of the Motion Picture

Association of America, Inc., have not proven nor established their ownership over the films listed in Annex "A" of the search warrant issued by this Honorable Court against the defendants herein.

2. The information provided by the NBI agents and the representatives of the MPAA, Inc. are replete with generalities insofar as the description of the items to be concerned in violation of the provisions of Sec. 3 of Rule 126 of the Rules of Court. Their allegations as to the offense are presumptuous and speculative in violation of the same section of the Rules of Court.

Private respondents in SW No. 97 adopted the motions filed for the quashal of both SW No. 95 and SW No. 96.

Herein petitioners (the private complainants in the three cases), namely, Columbia Pictures Entertainment, Inc., Orion Pictures Corporation, 20th Century Fox Film Corporation, MGM/UA Communications Company, Universal City Studios, Inc., Walt Disney Company and Warner Bros., Inc., submitted their oppositions to the motions to quash.

The movants, herein private respondents, filed their replies to the oppositions and sought, simultaneously, the release of the items seized.

After a rejoinder was filed, the court a quo considered all the incidents submitted for resolution.

41

Page 42: IP Developed Reviewer

Issues in the RTC:1. Whether or not the NBI had authority to file the application for search warrant; whether or not it is

the Videogram Regulatory Board under PD 1987 which has exclusive jurisdiction to file suits against violators of said law. (YES, NBI had authority)

2. Whether or not this Court observed due process of law before issuing the search warrants in question. (YES, due process was observed)

3. Whether or not search warrants Nos. 95, 96 and 97 are general warrants and therefore void. (NO, questioned search warrants were not general in character since the provision of law violated, i.e., Sec. 56 of PD 49, as amended by PD 1988, was clearly specified.)

4. Whether or not there was probable cause in the issuance of the search warrants pursuant to Section 3, Rule 126 of the 1985 Rules on Criminal Procedure and Section 2, Article III of the 1987 Constitution of the Republic of the Philippines.

5. Whether or not private complainants who are members of the Motion Picture Association of America, Inc. (MPAA for brevity) through their counsel, Atty. Rico Domingo, have sufficiently proven their ownership over the alleged pirated video tapes of the copyrighted motion pictures/films.

6. Whether or not the items seized by the NBI agents by virtue of SW Nos. 95, 96 and 97 may be ordered released to defendants.

Anent the first three issues, xxx Judge Austria, nonetheless, reversed her former stand initially finding probable cause for the issuance of the search warrants and ordered the quashal of the search warrants and return of the items giving the following reasons:1. Private complainants were uncertain of their ownership of the titles subject of the seized video

tapes;2. Complainants did not comply with the requirement that the master tapes should be presented

during the application for search warrants; and3. Private complainants cannot seek the protection of Philippine laws as they failed to comply with the

deposit and registration requirements of PD 49 as amended by PD 1988.

Issues in the CA:Petitioners appealed the order of Judge Austria to CA, assigning the following alleged errors:

1. The Court a quo erred in ruling that private complainants were uncertain of their ownership of the titles subject of the pirated video tapes.

2. The Court a quo erred in ordering the quashal of the search warrants on the ground that the requirement of producing the "master tapes" during the application for a search warrant, as enunciated in the 20th Century Fox case, promulgated on 19 August 1988, was applicable to the facts of the instant case which transpired on 07 April 1988, and that the same was not complied with.

3. The Court a quo erred in ruling that appellants do not have a protectable copyright under Philippine laws for their failure to comply with the deposit and registration requirements of Presidential Decree No. 49, as amended by Presidential Decree No. 1988.

CA: Sustained petitioners' first and third assignment of errors but rejecting petitioners' second assignment of error. It, therefore, still affirmed the quashal of the search warrants.

Hence, this petition. Another decision rendered by the CA in another case, involving the same petitioners on substantially identical facts and issues, was also brought before this Court. In a Resolution, this Court consolidated the two petitions.

42

Page 43: IP Developed Reviewer

Issue: Whether or not the presentation of the master tapes was necessary for the validity of the search warrants.

Held: YES. As held in the 20th Centrury Fox v. CA: The presentation of the master tapes of the

copyrighted films from which the pirated films were allegedly copied, was necessary for the validity of search warrants against those who have in their possession the pirated films. The petitioner's argument to the effect that the presentation of the master tapes at the time of application may not be necessary as these would be merely evidentiary in nature and not determinative of whether or not a probable cause exists to justify the issuance of the search warrants is not meritorious. The court cannot presume that duplicate or copied tapes were necessarily reproduced from master tapes that it owns.

The application for search warrants was directed against videotape outlets which allegedly were engaged in the unauthorized sale and renting out of copyrighted films belonging to the petitioner pursuant to PD 49.

The essence of a copyright infringement is the similarity or at least substantial similarity of the purported pirated works to the copyrighted work. Hence, the applicant must present to the court the copyrighted films to compare them with the purchased evidence of the videotapes allegedly pirated to determine whether the latter is an unauthorized reproduction of the former. This linkage of the copyrighted films to the pirated films must be established to satisfy the requirements of probable cause. Mere allegations as to the existence of the copyrighted films cannot serve as basis for the issuance of a search warrant.

In applying for the search warrants the NBI charged violation of the entire provisions of Section 56 of PD 49 as amended by PD 1988. This included not only the sale, lease or distribution of pirated tapes but also the transfer or causing to be transferred of any sound recording or motion picture or other audio visual work.

Columbia Pictures v. Court of Appeals, G.R. No. 110318, Aug. 28, 1996Facts: Complainants lodged a formal complaint with the NBI for violation of PD 49, as amended, and

sought its assistance in their anti-film piracy drive. Agents of the NBI and private researchers made discreet surveillance on various video

establishments in Metro Manila including Sunshine Home Video Inc. (Sunshine for brevity), owned and operated by Danilo A. Pelindario.

NBI Senior Agent Reyes applied for a search warrant with the court a quo against Sunshine seeking the seizure, among others, of pirated video tapes of copyrighted films all of which were enumerated in a list attached to the application; and, tv sets, video cassettes and/or laser disc recordings equipment and other machines and paraphernalia used or intended to be used in the unlawful exhibition, showing, reproduction, sale, lease or disposition of videograms tapes in the premises above described.

Search warrant was issued. Search warrant was served and various videotapes of duly copyrighted motion pictures/films

owned or exclusively distributed by private complainants, and machines, equipment, tv sets, paraphernalia, materials, accessories were seized.

A "Motion To Lift the Order of Search Warrant" was filed but was later denied for lack of merit. MR of Order of Denial was granted because: It is undisputed that the master tapes of the

copyrighted films from which the pirated films were allegedly copies ( sic ), were never presented in

43

Page 44: IP Developed Reviewer

the proceedings for the issuance of the search warrants in question. The orders of the Court granting the search warrants and denying the urgent motion to lift order of search warrants were, therefore, issued in error. Consequently, they must be set aside.

Petitioners appealed to CA but denied. MR also denied. Hence, this petition was brought to SC particularly challenging the validity of respondent court's

retroactive application of the ruling in 20th Century Fox Film Corporation vs. Court of Appeals, et al., in dismissing petitioners' appeal and upholding the quashal of the search warrant by the trial court.

Issues/Held:1) Whether or not petitioners, being unlicensed foreign corporations, cannot file an action here for lack

of personality to sue.

The term "lack of capacity to sue" should not be confused with the term "lack of personality to sue." While the former refers to a plaintiff's general disability to sue, such as on account of minority, insanity, incompetence, lack of juridical personality or any other general disqualifications of a party, the latter refers to the fact that the plaintiff is not the real party in interest. Correspondingly, the first can be a ground for a motion to dismiss based on the ground of lack of legal capacity to sue; whereas the second can be used as a ground for a motion to dismiss based on the fact that the complaint, on the face thereof, evidently states no cause of action.

Applying the above discussion to the instant petition, the ground available for barring recourse to our courts by an unlicensed foreign corporation doing or transacting business in the Philippines should properly be "lack of capacity to sue," not "lack of personality to sue." Certainly, a corporation whose legal rights have been violated is undeniably such, if not the only, real party in interest to bring suit thereon although, for failure to comply with the licensing requirement, it is not capacitated to maintain any suit before our courts.

Lastly, on this point, we reiterate this Court's rejection of the common procedural tactics of erring local companies which, when sued by unlicensed foreign corporations not engaged in business in the Philippines, invoke the latter's supposed lack of capacity to sue. The doctrine of lack of capacity to sue based on failure to first acquire a local license is based on considerations of public policy. It was never intended to favor nor insulate from suit unscrupulous establishments or nationals in case of breach of valid obligations or violation of legal rights of unsuspecting foreign firms or entities simply because they are not licensed to do business in the country.

2) We now proceed to the main issue of the retroactive application to the present controversy of the ruling in 20th Century Fox Film Corporation vs. Court of Appeals, et al., promulgated on August 19, 1988, that for the determination of probable cause to support the issuance of a search warrant in copyright infringement cases involving videograms, the production of the master tape for comparison with the allegedly pirate copies is necessary.

Petitioner’s contention: The lower court could not possibly have been expected to apply, as the basis for a finding of probable cause for the issuance of a search warrant in copyright infringement cases involving videograms, a pronouncement which was not existent at the time of such determination, on December 14, 1987, that is, the doctrine in the 20th Century Fox case that was promulgated only on August 19, 1988, or over eight months later.

Private respondents’ contention: Although the 20th Century Fox case had not yet been decided,

44

Page 45: IP Developed Reviewer

Section 2, Article III of the Constitution and Section 3, Rule 126 of the 1985 Rules on Criminal Procedure embodied the prevailing and governing law on the matter. The ruling in 20th Century Fox was merely an application of the law on probable cause. Hence, they posit that there was no law that was retrospectively applied, since the law had been there all along. To refrain from applying the 20th Century Fox ruling, which had supervened as a doctrine promulgated at the time of the resolution of private respondents' MR seeking the quashal of the search warrant for failure of the trial court to require presentation of the master tapes prior to the issuance of the search warrant, would have constituted grave abuse of discretion.

. . . . But while our decisions form part of the law of the land, they are also subject to Article 4 of the Civil Code which provides that "laws shall have no retroactive effect unless the contrary is provided." This is expressed in the familiar legal maxim lex prospicit, non respicit, the law looks forward not backward. The rationale against retroactivity is easy to perceive. The retroactive application of a law usually divests rights that have already become vested or impairs the obligations of contract and hence, is unconstitutional (Francisco v. Certeza, 3 SCRA 565 [1961]). The same consideration underlies our rulings giving only prospective effect to decisions enunciating new doctrines. . . . .

There is merit in petitioners' impassioned and well-founded argumentation:The case of 20th Century Fox Film Corporation vs. Court of Appeals, et al., 164 SCRA 655

(August 19, 1988) (hereinafter 20th Century Fox) was inexistent in December of 1987 when Search Warrant 87-053 was issued by the lower court. Hence, it boggles the imagination how the lower court could be expected to apply the formulation of 20th Century Fox in finding probable cause when the formulation was yet non-existent.

xxxThe lower court, therefore, lifted the three (3) questioned search warrants in the absence of

probable cause that the private respondents violated P.D. 49. As found out by the court, the NBI agents who acted as witnesses did not have personal knowledge of the subject matter of their testimony which was the alleged commission of the offense by the private respondents. Only the petitioner's counsel who was also a witness during the application for the issuance of the search warrants stated that he had personal knowledge that the confiscated tapes owned by the private respondents were pirated tapes taken from master tapes belonging to the petitioner. However, the lower court did not give much credence to his testimony in view of the fact that the master tapes of the allegedly pirated tapes were not shown to the court during the application (Emphasis ours).

In the case at bar, NBI Senior Agent Reyes who filed the application for search warrant with the lower court following a formal complaint lodged by petitioners, judging from his affidavit and his deposition, did testify on matters within his personal knowledge based on said complaint of petitioners as well as his own investigation and surveillance of the private respondents' video rental shop. Likewise, Atty. Rico V. Domingo, in his capacity as attorney-in-fact, stated in his affidavit and further expounded in his deposition that he personally knew of the fact that private respondents had never been authorized by his clients to reproduce, lease and possess for the purpose of selling any of the copyrighted films.

Both testimonies of Agent Reyes and Atty. Domingo were corroborated by Rene C. Baltazar, a private researcher retained by Motion Pictures Association of America, Inc. (MPAA, Inc.), who was likewise presented as a witness during the search warrant proceedings. The records clearly reflect that the testimonies of the abovenamed witnesses were straightforward and stemmed from matters within their personal knowledge. They displayed none of the ambivalence and uncertainty that the witnesses in the 20th Century Fox case exhibited. This categorical forthrightness in their statements,

45

Page 46: IP Developed Reviewer

among others, was what initially and correctly convinced the trial court to make a finding of the existence of probable cause.

It is evidently incorrect to suggest, as the ruling in 20th Century Fox may appear to do, that in copyright infringement cases, the presentation of master tapes of the copyrighted films is always necessary to meet the requirement of probable cause and that, in the absence thereof, there can be no finding of probable cause for the issuance of a search warrant. It is true that such master tapes are object evidence, with the merit that in this class of evidence the ascertainment of the controverted fact is made through demonstrations involving the direct use of the senses of the presiding magistrate. Such auxiliary procedure, however, does not rule out the use of testimonial or documentary evidence, depositions, admissions or other classes of evidence tending to prove the factum probandum , especially where the production in court of object evidence would result in delay, inconvenience or expenses out of proportion to its evidentiary value.

(Note from Sir: But did not really do away with presentation of master tape. There’s a need to present when there’s doubt as to the content of the master tape & the copies. Also, this case recognized other proofs that may be presented.)

(In this case, the researcher of NBI rented Little Shop of Horror and Robocop from Sunshine. This renting out is clearly without authorization from MPAA.)

As correctly pointed out by petitioners, a blind espousal of the requisite of presentation of the master tapes in copyright infringement cases, as the prime determinant of probable cause, is too exacting and impracticable a requirement to be complied with in a search warrant application which, it must not be overlooked, is only an ancillary proceeding . Further, on realistic considerations, a strict application of said requirement militates against the elements of secrecy and speed which underlie covert investigative and surveillance operations in police enforcement campaigns against all forms of criminality, considering that the master tapes of a motion picture required to be presented before the court consists of several reels contained in circular steel casings which, because of their bulk, will definitely draw attention, unlike diminutive objects like video tapes which can be easily concealed. With hundreds of titles being pirated, this onerous and tedious imposition would be multiplied a hundredfold by judicial fiat, discouraging and preventing legal recourses in foreign jurisdictions.

2) The trial court's finding that private respondents committed acts in blatant transgression of PD 49 all the more bolsters its findings of probable cause, which determination can be reached even in the absence of master tapes by the judge in the exercise of sound discretion. The executive concern and resolve expressed in the foregoing amendments to the decree for the protection of intellectual property rights should be matched by corresponding judicial vigilance and activism, instead of the apathy of submitting to technicalities in the face of ample evidence of guilt.

xxxA copy of a piracy is an infringement of the original, and it is no defense that the pirate, in such

cases, did not know what works he was indirectly copying, or did not know whether or not he was infringing any copyright; he at least knew that what he was copying was not his, and he copied at his peril. In determining the question of infringement, the amount of matter copied from the copyrighted work is an important consideration. To constitute infringement, it is not necessary that the whole or even a large portion of the work shall have been copied. If so much is taken that the value of the original is sensibly diminished, or the labors of the original author are substantially and to an injurious extent appropriated by another, that is sufficient in point of law to constitute apiracy. The question of whether there has been an actionable infringement of a literary, musical, or artistic work in motion

46

Page 47: IP Developed Reviewer

pictures, radio or television being one of fact, it should properly be determined during the trial. That is the stage calling for conclusive or preponderating evidence, and not the summary proceeding for the issuance of a search warrant wherein both lower courts erroneously require the master tapes.

In disregarding private respondent's argument that Search Warrant No. 87-053 is a general warrant, the lower court observed that "it was worded in a manner that the enumerated seizable items bear direct relation to the offense of violation of Sec. 56 of PD 49 as amended. It authorized only the seizur(e) of articles used or intended to be used in the unlawful sale, lease and other unconcerted acts in violation of PD 49 as amended. . . .

xxxThat there were several counts of the offense of copyright infringement and the search warrant

uncovered several contraband items in the form of pirated videotapes is not to be confused with the number of offenses charged. The search warrant herein issued does not violate the one-specific-offense rule.

It is pointless for private respondents to insist on compliance with the registration and deposit requirements under PD 49 as prerequisites for invoking the court's protective mantle in copyright infringement cases. As explained by the court below:

Defendants-movants contend that PD 49 as amended covers only producers who have complied with the requirements of deposit and notice (in other words registration) under Sections 49 and 50 thereof. Absent such registration, as in this case, there was no right created, hence, no infringement under PD 49 as amended. This is not well-taken.

As correctly pointed out by private complainants-oppositors, DOJ has resolved this legal question as far back as December 12, 1978 in its Opinion No. 191 of the then Secretary of Justice Vicente Abad Santos which stated that Sections 26 and 50 do not apply to cinematographic works and PD 49 "had done away with the registration and deposit of cinematographic works" and that "even without prior registration and deposit of a work which may be entitled to protection under the Decree, the creator can file action for infringement of its rights". He cannot demand, however, payment of damages arising from infringement. The same opinion stressed that "the requirements of registration and deposit are thus retained under the Decree, not as conditions for the acquisition of copyright and other rights, but as prerequisites to a suit for damages". The statutory interpretation of the Executive Branch being correct, is entitled (to) weight and respect.

xxx xxx xxxDefendants-movants maintain that complainant and his witnesses led the Court to believe that

a crime existed when in fact there was none. This is wrong. As earlier discussed, PD 49 as amended, does not require registration and deposit for a creator to be able to file an action for infringement of his rights. These conditions are merely pre-requisites to an action for damages. So, as long as the proscribed acts are shown to exist, an action for infringement may be initiated.

Habana v. Robles, G.R. No. 131522, July 19, 1999Facts: Petitioners are authors and copyright owners of duly issued certificates of copyright registration

covering published works entitled College English for Today (CET), Books 1 and 2 and its corresponding workbook.

Respondent Felicidad Robles and Goodwill Trading Co., Inc. are the author/publisher and distributor/seller of another published work entitled Developing English Proficiency (DEP) Books 1 and 2 and its corresponding workbook.

Petitioners learned that DEP was strikingly similar to the contents, scheme of presentation, illustrations and illustrative examples in CET, and that several pages were all together a copy of

47

Page 48: IP Developed Reviewer

their book. Petitioners then made demands for damages and asked that Robles cease and desist from selling

and distributing DEP. Their demands were ignored prompting them to file a complaint for “Infringement and/or unfair competition with damages.” Complaint alleged that Robles being substantially familiar with the contents of petitioners’ works, and without securing their permission, lifted, copied, plagiarized and/or transposed certain portions of their book CET

Respondent Robles was impleaded in the suit because she authored and directly committed the acts of infringement complained of, while respondent Goodwill Trading Co., Inc. was impleaded as the publisher and joint co-owner of the copyright certificates of registration

Robles filed a motion for a bill of particulars which the trial court approved. Goodwill alleged that petitioners had no cause of action against them since it was not privy to the misrepresentation, plagiarism, incorporation and reproduction of the portions of the book and that Robles guaranteed Goodwill that the materials utilized in the manuscript were her own

Robles’ defense: 1.) DEP is the product of her independent researches, studies and experiences, and was not a copy of any existing valid copyrighted book; 2.) DEP followed the scope and sequence or syllabus which are common to all English grammar writers so any similarity between the respondents book and that of the petitioners was due to the orientation of the authors to both works and standards and syllabus;  3.) similarities may be due to the authors’ exercise of the “right to fair use of copyrighted materials, as guides. Robles also counterclaimed for damages alleging bad faith on the part of petitioners because DEP replaced CET as the official textbook of the graduate studies department of FEU.

RTC dismissed complaint and CA rendered judgment in favor of respondents Robles and Goodwill CA’s ratio: similarity of the allegedly infringed work to the author’s or proprietor’s copyrighted work

does not of itself establish copyright infringement, especially if the similarity results from the fact that both works deal with the same subject or have the same common source, as in this case. Robles fully proved that the topics appearing in both books also appeared in earlier books.

 Issues/Held: 1) W/N despite the apparent textual, thematic and sequential similarity between DEP and CET, Robles committed no copyright infringement.  NO, there was infringement.

Though the complaint was filed when PD 49 was in force and that the governing law now is RA 8293, the same principles are reiterated in Sec. 177 (reproduction of the work or a substantial portion of the work) and Sec. 184 Limitations on copyright (making of quotations from a published work if they are compatible with fair use and only to the extent justified for the purpose; inclusion of a work in a publication provided that the source and name of the author if appearing in the work is mentioned).  Robles’ act of lifting from the book of petitioners substantial portions of discussions and examples, and her failure to acknowledge the same in her book is an infringement of petitioners’ copyrights. If so much is taken that the value of the original work is substantially diminished, there is an infringement of copyright and to an injurious extent, the work is appropriated. In determining the question of infringement, the amount of matter copied from the copyrighted work is an important consideration.  To constitute infringement, it is not necessary that the whole or even a large portion of the work shall have been copied.  If so much is taken that the value of the original is sensibly diminished, or the labors of the original author are substantially and to an injurious extent appropriated by another, that is sufficient in point of law to constitute piracy. 2) W/N Robles abused a writer’s right to fair use -YES

48

Page 49: IP Developed Reviewer

The respondents’ claim that the copied portions of the book CET are also found in foreign books and other grammar books, and that the similarity between her style and that of petitioners can not be avoided since they come from the same background and orientation may be true, however the limitations on copyright under Sec. 184 provides that the source and the name of the author must be mentioned. A copy of a piracy is an infringement of the original, and it is no defense that the pirate, in such cases, did not know whether or not he was infringing any copyright; he at least knew that what he was copying was not his, and he copied at his peril. It is not an excuse for them to be identical even in examples contained in their books. 

In cases of infringement, copying alone is not what is prohibited.  The copying must produce an “injurious effect”.  Here, the injury consists in that respondent Robles lifted from petitioners’ book materials that were the result of the latter’s research work and compilation and misrepresented them as her own.   She circulated the book DEP for commercial use and did not acknowledge petitioners as her source.

(But read the brilliant dissent of Justice Davide.)

Joaquin v. Drilon, G.R. No. 108946, Jan. 28, 1999 Facts: Petitioner BJ Productions, Inc. (BJPI) is the holder/grantee of Certificate of Copyright No. M922 of

Rhoda and Me, a dating game show aired from 1970 to 1977. Petitioner BJPI submitted to the National Library an addendum to its certificate of copyright

specifying the show's format and style of presentation. While watching tv, petitioner Joaquin, Jr., president of BJPI, saw on RPN Channel 9 an episode of

It's a Date, which was produced by IXL Productions, Inc. (IXL). Joaquin wrote a letter to private respondent Zosa, president and general manager of IXL, informing

Zosa that BJPI had a copyright to Rhoda and Me and demanding that IXL discontinue airing It's a Date.

In a letter, private respondent Zosa apologized to petitioner Joaquin and requested a meeting to discuss a possible settlement.

IXL, however, continued airing It's a Date, prompting petitioner Joaquin to send a second letter in which he reiterated his demand and warned that, if IXL did not comply, he would endorse the matter to his attorneys for proper legal action.

Meanwhile, private respondent Zosa sought to register IXL's copyright to the first episode of It's a Date for which it was issued by the National Library a certificate of copyright.

Upon complaint of petitioners, an information for violation of PD 49 was filed against private respondent Zosa together with certain officers of RPN Channel 9 in the RTC of QC.

However, private respondent Zosa sought a review of the resolution of the Assistant City Prosecutor before the DOJ.

DOJ Sec. Drilon reversed the Assistant City Prosecutor's findings and directed him to move for the dismissal of the case against private respondents. MR denied.

Both public and private respondents maintain that petitioners failed to establish the existence of probable cause due to their failure to present the copyrighted master videotape of Rhoda and Me. They contend that petitioner BJPI's copyright covers only a specific episode of Rhoda and Me and that the formats or concepts of dating game shows are not covered by copyright protection under PD 49.

Issues/ Held:

49

Page 50: IP Developed Reviewer

1) Non-assignment of error – Sec. of Justice not confined to issues submitted to himPetitioners claim that their failure to submit the copyrighted master videotape of the television

show Rhoda and Me was not raised in issue by private respondents during the preliminary investigation and, therefore, it was error for the Secretary of Justice to reverse the investigating prosecutor's finding of probable cause on this ground.

In reviewing resolutions of prosecutors, the Secretary of Justice is not precluded from considering errors, although unassigned, for the purpose of determining whether there is probable cause for filing cases in court. He must make his own finding, of probable cause and is not confined to the issues raised by the parties during preliminary investigation. Moreover, his findings are not subject to review unless shown to have been made with grave abuse.

2) Opinion of Secretary of Justice – Sec. of Justice not precluded from making PIPetitioners contend, however, that the determination of the question whether the format or

mechanics of a show is entitled to copyright protection is for the court, and not the Secretary of Justice, to make.

It is indeed true that the question whether the format or mechanics of petitioners television show is entitled to copyright protection is a legal question for the court to make. This does not, however, preclude respondent Secretary of Justice from making a preliminary determination of this question in resolving whether there is probable cause for filing the case in court. In doing so in this case, he did not commit any grave error.

3) Presentation of Master Tape – should have presented(And is a dating show format copyrightable? NO)

Petitioners claim that respondent Secretary of Justice gravely abused his discretion in ruling that the master videotape should have been presented in order to determine whether there was probable cause for copyright infringement.

Petitioners assert that the format of Rhoda and Me is a product of ingenuity and skill and is thus entitled to copyright protection. It is their position that the presentation of a point-by-point comparison of the formats of the two shows clearly demonstrates the nexus between the shows and hence establishes the existence of probable cause for copyright infringement. Such being the case, they did not have to produce the master tape.

To begin with the format of a show is not copyrightable . Section 2 of PD 49, otherwise known as the DECREE ON INTELLECTUAL PROPERTY, enumerates the classes of work entitled to copyright protection (same as Sec. 172 of IP Code).

PD 49, Sec. 2, in enumerating what are subject to copyright, refers to finished works and not to concepts. The copyright does not extend to an idea, procedure, process, system, method of operation, concept, principle, or discovery, regardless of the form in which it is described, explained, illustrated, or embodied in such work. (see Sec. 175 of IP Code)

What then is the subject matter of petitioners' copyright?This Court is of the opinion that petitioner BJPI's copyright covers audio-visual recordings of each episode of Rhoda and Me , as falling within the class of works mentioned in P.D. 49, Sec. 2(M),

Pearl & Dean v. Shoemart, G.R. No. 148222, Aug. 15, 2003Facts: Pearl & Dean manufactures advertising display units called light boxes. These units utilize specially

printed posters sandwiched between plastic sheets and illuminated with back lights. These light

50

Page 51: IP Developed Reviewer

boxes were marketed under the trademark “Poster Ads”. They filed an application for registration of trademarks in 1983 but it was approved only in 1988.

Pearl & Dean negotiated with Shoemart Inc. (SMI) for the lease and installation of the light boxes in SM North Edsa but because the same was under construction the last time, SMI offered SM Makati and Cubao as an alternative to which Pearl agreed.

However, only the contract for SM Makati was returned signed and even this was rescinded by SMI due to nonperformance of the terms. To this, Vergara, President of Pearl protested.

2 years later, Metro Industrial Services, the company formerly contracted by Pearl and Dean to fabricate its display units, offered to construct light boxes for Shoemart’s chain of stores.  SMI approved the proposal and 10 light boxes were subsequently fabricated. SMI engaged the services of EYD Rainbow Advertising Corporation to make the light boxes.  Some 300 units were fabricated in 1991.

Pearl discovered that exact copies of its light boxes were installed in SM branches and that North Edsa Marketing Inc. (NEMI) through its marketing arm, Prime Spots Marketing Services, was set up primarily to sell advertising space in lighted display units located in SMI’s different branches. NEMI is a sister company of SMI.

Pearl & Dean sent a letter to both SMI and NEMI enjoining them to cease using the subject light boxes and to remove them from all of SMI’s establishments. SMI suspended the leasing of 224 light boxes and took down advertisements for “Poster Ads”.

SMI’s contention: it independently developed its poster panels using commonly known techniques and available technology, without notice of or reference to Pearl and Dean’s copyright.  SMI noted that the registration of the mark “Poster Ads” was only for stationeries such as letterheads, envelopes, and the like.  Besides, according to SMI, the word “Poster Ads” is a generic term which cannot be appropriated as a trademark, and, as such, registration of such mark is invalid.

NEMI, for its part, denied having manufactured, installed or used any advertising display units, nor having engaged in the business of advertising

 RTC ruling: SMI and NEMI are found jointly and severally liable for infringement of copyright and trademark. It also ordered the impounding in the National Library of all light boxes of SMI, destruction of filler-posters and further ordered SMI to permanently refrain from infringing the copyright on plaintiff’s light boxes and its trademark “Poster Ads”. CA ruling: reversed RTC. Since the light boxes cannot, by any stretch of the imagination, be considered as either prints, pictorial illustrations, advertising copies, labels, tags or box wraps, to be properly classified as a copyrightable class “O” work, we have to agree with SMI when it posited that what was copyrighted were the technical drawings only, and not the light boxes. It held that protection of the drawing does not extend to the unauthorized duplication of the object drawn because copyright extends only to the description or expression of the object and not to the object itself.                As for infringement of trademark, the CA held that “Poster Ads” was registered by Pearl and Dean for specific use in its stationeries, in contrast to defendants-appellants who used the same words in their advertising display units. We are constrained to adopt the view of defendants-appellants that the words “Poster Ads” are a simple contraction of the generic term poster advertising. Issues / Held:1.)     If the engineering or technical drawings of an advertising display unit (light box) are granted copyright protection (copyright certificate of registration) by the National Library, is the light box depicted in such engineering drawings ipso facto also protected by such copyright?

51

Page 52: IP Developed Reviewer

NO. Copyright was limited to the drawings alone and not to the light box itself. Application for copyright certificate clearly stated that it was for class “O” work. Although petitioner’s copyright certificate was entitled “Advertising Display Units” (which depicted the box-type electrical devices), its claim of copyright infringement cannot be sustained. Being a mere statutory grant, the rights are limited to what the statute confers.  It may be obtained and enjoyed only with respect to the subjects and by the persons, and on terms and conditions specified in the statute. Accordingly, it can cover only the works falling within the statutory enumeration or description.  P & D owned a valid copyright but the copyright protection extended only to the technical drawings and not to the light box itself because the latter was not at all in the category of pictorial illustrations. The light box was not a literary or artistic piece which could be copyrighted under the copyright law. Besides, during the trial, P & D himself admitted that the light box was neither a literary nor an artistic work but an engineering or marketing invention.  In the US SC case of Baker vs. Selden , it was held that only the expression of an idea is protected by copyright and not the idea itself. It further held that the description of the art in a book, though entitled to the benefit of copyright, lays no foundation for an exclusive claim to the art itself.  The object of the one is explanation; the object of the other is use.  The former may be secured by copyright.  The latter can only be secured, if it can be secured at all, by letters patent. 2.) If, despite its manufacture and commercial use of the light boxes without license from petitioner, SMI can be held legally liable for infringement of P & D’s copyright over its technical drawings of the said light boxes, should they be liable instead for infringement of patent?

NO. No patent, no protection. P & D never secured a patent for the light boxes.  It therefore acquired no patent rights which could have protected its invention, if in fact it really was. And because it had no patent, P & D could not legally prevent anyone from manufacturing or commercially using the contraption. In Creser Precision Systems, Inc. vs. CA, it was held that “(A)n inventor has no common law right to a monopoly of his invention. He has the right to make use of and vend his invention, but if he voluntarily discloses it, such as by offering it for sale, the world is free to copy and use it with impunity.  A patent, however, gives the inventor the right to exclude all others”.  Even on the assumption that P & D’s advertising units were patentable inventions, petitioner revealed them fully to the public by submitting the engineering drawings thereof to the National Library.  The patent law has a three-fold purpose: “first, patent law seeks to foster and reward invention; second, it promotes disclosures of inventions to stimulate further innovation and to permit the public to practice the invention once the patent expires; third, the stringent requirements for patent protection seek to ensure that ideas in the public domain remain there for the free use of the public”. What petitioner seeks is exclusivity without any opportunity for the patent office (IPO) to scrutinize the light box’s eligibility as a patentable invention. 3.) Can the owner of a registered trademark legally prevent others from using such trademark if it is a mere abbreviation of a term descriptive of his goods, services or business?  NO. Faberge vs. IAC it was held that the certificate of registration issued by the Director of Patents can confer (upon petitioner) the exclusive right to use its own symbol only to those goods specified in the certificate, subject to any conditions and limitations specified in the certificate. One who has adopted and used a trademark on his goods does not prevent the adoption and use of the same trademark by others for products which are of a different description. 

52

Page 53: IP Developed Reviewer

On the issue of unfair competitionIf at all, the cause of action should have been for unfair competition, a situation which was

possible even if P & D had no registration. However, while the petitioner’s complaint in the RTC also cited unfair competition, the trial court did not find private respondents liable therefor. Petitioner did not appeal this particular point; hence, it cannot now revive its claim of unfair competition. If we were to disregard procedural issues however, SMI still NOT GUILT of unfair competition.

By the nature of things, there can be no unfair competition under the law on copyrights although it is applicable to disputes over the use of trademarks. Even a name or phrase incapable of appropriation as a trademark or tradename may, by long and exclusive use by a business (such that the name or phrase becomes associated with the business or product in the mind of the purchasing public), be entitled to protection against unfair competition. However, P & D’s expert witness said that in the mind of the public, the goods and services carrying the trademark “Poster Ads” could not be distinguished from the goods and services of other entities.

Even under “secondary meaning” unfair competition would still fail. . “Secondary meaning” means that a word or phrase originally incapable of exclusive appropriation with reference to an article in the market (because it is geographically or otherwise descriptive) might nevertheless have been used for so long and so exclusively by one producer with reference to his article that, in the trade and to that branch of the purchasing public, the word or phrase has come to mean that the article was his property. The admission by petitioner’s own expert witness that he himself could not associate “Poster Ads” with petitioner P & D because it was “too generic” definitely precluded the application of this exception.

K. Moral Rights (Sec. 193-199)Section 193. Scope of Moral Rights. - The author of a work shall, independently of the

economic rights in Section 177 or the grant of an assignment or license with respect to such right, have the right:

193.1. To require that the authorship of the works be attributed to him, in particular, the right that his name, as far as practicable, be indicated in a prominent way on the copies, and in connection with the public use of his work; (right of attribution)

193.2. To make any alterations of his work prior to, or to withhold it from publication; (up to 193.4 right to integrity)

193.3. To object to any distortion, mutilation or other modification of, or other derogatory action in relation to, his work which would be prejudicial to his honor or reputation; and

193.4. To restrain the use of his name with respect to any work not of his own creation or in a distorted version of his work. (Sec. 34, P.D. No. 49)

Section 194. Breach of Contract. - An author cannot be compelled to perform his contract to create a work or for the publication of his work already in existence. However, he may be held liable for damages for breach of such contract. (Sec. 35, P.D. No. 49)

Section 195. Waiver of Moral Rights. - An author may waive his rights mentioned in Section 193 by a written instrument , but no such waiver shall be valid where its effects is to permit another:

195.1. To use the name of the author, or the title of his work, or otherwise to make use of his reputation with respect to any version or adaptation of his work which, because of alterations therein, would substantially tend to injure the literary or artistic reputation of another author; or

195.2. To use the name of the author with respect to a work he did not create. (Sec. 36, P.D. No. 49)

53

Page 54: IP Developed Reviewer

Section 196. Contribution to Collective Work. - When an author contributes to a collective work, his right to have his contribution attributed to him is deemed waived unless he expressly reserves it. (Sec. 37, P.D. No. 49)

Section 197. Editing, Arranging and Adaptation of Work. - In the absence of a contrary stipulation at the time an author licenses or permits another to use his work, the necessary editing, arranging or adaptation of such work, for publication, broadcast, use in a motion picture, dramatization, or mechanical or electrical reproduction in accordance with the reasonable and customary standards or requirements of the medium in which the work is to be used, shall not be deemed to contravene the author's rights secured by this chapter. Nor shall complete destruction of a work unconditionally transferred by the author be deemed to violate such rights. (Sec. 38, P.D. No. 49)

Section 198. Term of Moral Rights. – 198.1. The rights of an author under this chapter shall last during the lifetime of the author and

for 50 years after his death and shall not be assignable or subject to license. The person or persons to be charged with the posthumous enforcement of these rights shall be named in writing to be filed with the National Library. In default of such person or persons, such enforcement shall devolve upon either the author's heirs, and in default of the heirs, the Director of the National Library.

198.2. For purposes of this Section, "Person" shall mean any individual, partnership, corporation, association, or society. The Director of the National Library may prescribe reasonable fees to be charged for his services in the application of provisions of this Section. (Sec. 39, P.D. No. 49)

Section 199. Enforcement Remedies. - Violation of any of the rights conferred by this Chapter shall entitle those charged with their enforcement to the same rights and remedies available to a copyright owner. In addition, damages which may be availed of under the Civil Code may also be recovered. Any damage recovered after the creator's death shall be held in trust for and remitted to his heirs, and in default of the heirs, shall belong to the government. (Sec. 40, P D No. 49)

L. Right to Proceeds in Subsequent Transfers (Sec. 200, 201)Section 200. Sale or Lease of Work. - In every sale or lease of an original work of painting or

sculpture or of the original manuscript of a writer or composer, subsequent to the first disposition thereof by the author, the author or his heirs shall have an inalienable right to participate in the gross proceeds of the sale or lease to the extent of 5%. This right shall exist during the lifetime of the author and for 50 years after his death. (Sec. 31, P.D. No. 49)

Section 201. Works Not Covered. - The provisions of this Chapter shall not apply to prints, etchings, engravings, works of applied art, or works of similar kind wherein the author primarily derives gain from the proceeds of reproductions. (Sec. 33, P.D. No. 49)

M. Neighboring Rights (Sec. 202, 212)Section 202. Definitions. - For the purpose of this Act, the following terms shall have the

following meanings:202.1. "Performers" are actors, singers, musicians, dancers, and other persons who act, sing,

declaim, play in, interpret, or otherwise perform literary and artistic work;202.2. "Sound recording" means the fixation of the sounds of a performance or of other

sounds, or representation of sound, other than in the form of a fixation incorporated in a

54

Page 55: IP Developed Reviewer

cinematographic or other audiovisual work;202.3. An "audiovisual work or fixation" is a work that consists of a series of related images

which impart the impression of motion, with or without accompanying sounds, susceptible of being made visible and, where accompanied by sounds, susceptible of being made audible;

202.4. "Fixation" means the embodiment of sounds, or of the representations thereof, from which they can be perceived, reproduced or communicated through a device;

202.5. "Producer of a sound recording" means the person, or the legal entity, who or which takes the initiative and has the responsibility for the first fixation of the sounds of a performance or other sounds, or the representation of sounds;

202.6. "Publication of a fixed performance or a sound recording" means the offering of copies of the fixed performance or the sound recording to the public, with the consent of the right holder: Provided, That copies are offered to the public in reasonable quality;

202.7. "Broadcasting" means the transmission by wireless means for the public reception of sounds or of images or of representations thereof; such transmission by satellite is also "broadcasting" where the means for decrypting are provided to the public by the broadcasting organization or with its consent;

202.8. "Broadcasting organization" shall include a natural person or a juridical entity duly authorized to engage in broadcasting; and

202.9 "Communication to the public of a performance or a sound recording" means the transmission to the public, by any medium, otherwise than by broadcasting, of sounds of a performance or the representations of sounds fixed in a sound recording. For purposes of Section 209, "communication to the public" includes making the sounds or representations of sounds fixed in a sound recording audible to the public.

Section 212. Limitations on Rights. - Sections 203 (scope of performer’s right), 208 (scope of right of producers of sound recordings) and 209 (communication to the public of sound recordings) shall not apply where the acts referred to in those Sections are related to:

212.1. The use by a natural person exclusively for his own personal purposes;212.2. Using short excerpts for reporting current events;212.3. Use solely for the purpose of teaching or for scientific research; and212.4. Fair use of the broadcast subject to the conditions under Section 185. (Sec. 44, P.D.

No. 49a)

1. Rights of Performers (Sec. 203-207, 215) (apart from the copyright)Section 203. Scope of Performers' Rights. - Subject to the provisions of Section 212,

performers shall enjoy the following exclusive rights:203.1. As regards their performances, the right of authorizing:

(a) The broadcasting and other communication to the public of their performance; and

(b) The fixation of their unfixed performance. (to control the setting including the taping)

203.2. The right of authorizing the direct or indirect reproduction of their performances fixed in sound recordings, in any manner or form;

203.3. Subject to the provisions of Section 206, the right of authorizing the first public distribution of the original and copies of their performance fixed in the sound recording through sale or rental or other forms of transfer of ownership;

203.4. The right of authorizing the commercial rental to the public of the original and

55

Page 56: IP Developed Reviewer

copies of their performances fixed in sound recordings, even after distribution of them by, or pursuant to the authorization by the performer; and

203.5. The right of authorizing the making available to the public of their performances fixed in sound recordings, by wire or wireless means, in such a way that members of the public may access them from a place and time individually chosen by them. (Sec. 42, P.D. No. 49a)

Section 204. Moral Rights of Performers. – 204.1. Independently of a performer's economic rights, the performer, shall, as

regards his live aural performances or performances fixed in sound recordings, have the right to claim to be identified as the performer of his performances, except where the omission is dictated by the manner of the use of the performance, and to object to any distortion, mutilation or other modification of his performances that would be prejudicial to his reputation.

204.2. The rights granted to a performer in accordance with Subsection 203.1 shall be maintained and exercised 50 years after his death, by his heirs, and in default of heirs, the government, where protection is claimed. (Sec. 43, P.D. No. 49)

Section 205. Limitation on Right. – 205.1. Subject to the provisions of Section 206 (additional remuneration for

subsequent communications or broadcast), once the performer has authorized the broadcasting or fixation of his performance, the provisions of Sections 203 (scope of performer’s right) shall have no further application.

205.2. The provisions of Section 184 (limitations on copyright) and Section 185 (fair use of a copyrighted work) shall apply mutatis mutandis to performers. (n)

Section 206. Additional Remuneration for Subsequent Communications or Broadcasts. - Unless otherwise provided in the contract, in every communication to the public or broadcast of a performance subsequent to the first communication or broadcast thereof by the broadcasting organization, the performer shall be entitled to an additional remuneration equivalent to at least 5% of the original compensation he or she received for the first communication or broadcast. (n)

Section 207. Contract Terms. - Nothing in this Chapter shall be construed to deprive performers of the right to agree by contracts on terms and conditions more favorable for them in respect of any use of their performance. (n)

2. Rights of Producers of Sound Recording (Sec. 208-210, 215)Section 208. Scope of Right. - Subject to the provisions of Section 212 (limitations on

protection), producers of sound recordings shall enjoy the following exclusive rights:

208.1. The right to authorize the direct or indirect reproduction of their sound recordings, in any manner or form; the placing of these reproductions in the market and the right of rental or lending;

208.2. The right to authorize the first public distribution of the original and copies of their sound recordings through sale or rental or other forms of transferring ownership; and

208.3. The right to authorize the commercial rental to the public of the original and copies of their sound recordings, even after distribution by them by or pursuant to authorization by the producer. (Sec. 46, P.D. No. 49a)

Section 209. Communication to the Public. - If a sound recording published for

56

Page 57: IP Developed Reviewer

commercial purposes, or a reproduction of such sound recording, is used directly for broadcasting or for other communication to the public, or is publicly performed with the intention of making and enhancing profit, a single equitable remuneration for the performer or performers, and the producer of the sound recording shall be paid by the user to both the performers and the producer, who, in the absence of any agreement shall share equally. (Sec. 47, P.D. No. 49a)

Section 210. Limitation of Right. - Sections 184 and 185 shall apply mutatis mutandis to the producer of sound recordings. (Sec. 48, P.D. No. 49a)

Section 215. Term of Protection for Performers, Producers and Broadcasting Organizations.

215.1. The rights granted to performers and producers of sound recordings under this law shall expire:

(a) For performances not incorporated in recordings, 50 years from the end of the year in which the performance took place; and

(b) For sound or image and sound recordings and for performances incorporated therein, 50 years from the end of the year in which the recording took place.

215.2. In case of broadcasts, the term shall be 20 years from the date the broadcast took place. The extended term shall be applied only to old works with subsisting protection under the prior law. (Sec. 55, P.D. No. 49a)

3. Rights of Broadcasting Organizations (Sec. 211, 215.2)Section 211. Scope of Right. - Subject to the provisions of Section 212, broadcasting

organizations shall enjoy the exclusive right to carry out, authorize or prevent any of the following acts:

211.1. The rebroadcasting of their broadcasts;211.2. The recording in any manner, including the making of films or the use of video

tape, of their broadcasts for the purpose of communication to the public of television broadcasts of the same; and

211.3. The use of such records for fresh transmissions or for fresh recording. (Sec. 52, P.D. No. 49)

Section 215.2. In case of broadcasts, the term shall be 20 years from the date the broadcast took place. The extended term shall be applied only to old works with subsisting protection under the prior law. (Sec. 55, P.D. No. 49a)

IV. Trademarks

RA 166: 2 registers:

1) principal (ownership, validity & exclusivity presumed) & 2) supplemental (mere notice to the world)

must be owned by the registrant and was in actual use in commerce in the Philippines 2 months prior to filing of registration (not anymore, actual use can be shown by the applicant within 3 years from the date of application)

Sec.2-A: 1) through actual use 2) not appropriated to someone else

no provision on internationally well-known marks; relied on Paris Convention tradenames & trademarks treated similarly in terms of ownership

57

Page 58: IP Developed Reviewer

RA 8293: only 1 register (first to [validly] file rule)

the first to apply can bar the filing of an application of a subsequent identical mark… no requirement of actual use prior to filing of application through registration in accordance with law & Sec. 138, registration is prima facie proof of

ownership, validity & exclusivity of mark provides guidelines to determine whether a mark is internationally well-known (123.1 e & f)

(Sec. 102 of IRR)marks similar to internationally well-known marks are non-registrable

Sec. 165.1 & 165.2 (is the general rule (b) is the specific rule) different treatment for trademarks & tradenames

A. Definition of Trademarks and Functions

Functions:1. origin2. guarantee of quality3. advertise

Definition of Trade Names (Sec. 121.1, 121.2, 121.3, 165)121.1. "Mark" means any visible sign capable of distinguishing the goods (trademark) or

services (service mark) of an enterprise and shall include a stamped or marked container of goods; (Sec. 38, R.A. No. 166a)

121.2. "Collective mark" means any visible sign designated as such in the application for registration and capable of distinguishing the origin or any other common characteristic, including the quality of goods or services of different enterprises which use the sign under the control of the registered owner of the collective mark; (Sec. 40, R.A. No. 166a)

121.3. "Trade name" means the name or designation identifying or distinguishing an enterprise; (Sec. 38, R.A. No. 166a)

Section 165. Trade Names or Business Names. – 165.1. A name or designation may not be used as a trade name if by its nature or the use to

which such name or designation may be put, it is contrary to public order or morals and if, in particular, it is liable to deceive trade circles or the public as to the nature of the enterprise identified by that name.

165.2.(a) Notwithstanding any laws or regulations providing for any obligation to register trade

names, such names shall be protected, even prior to or without registration , against any unlawful act committed by third parties.

(b) In particular, any subsequent use of the trade name by a third party, whether as a trade name or a mark or collective mark, or any such use of a similar trade name or mark, likely to mislead the public, shall be deemed unlawful.

165.3. The remedies provided for in Sections 153 to 156 and Sections 166 and 167 shall apply mutatis mutandis.

165.4. Any change in the ownership of a trade name shall be made with the transfer of the enterprise or part thereof identified by that name. The provisions of Subsections 149.2

58

Page 59: IP Developed Reviewer

to 149.4 shall apply mutatis mutandis.

1. Trademarks and Trade Names

Converse v. Universal Rubber Products, G.R. No. L-27906, January 8, 1987Facts: Respondent Universal Rubber Products , Inc. filed an application with the Philippine Patent office for

registration of the trademark "UNIVERSAL CONVERSE AND DEVICE" used on rubber shoes and rubber slippers.

Petitioner Converse Rubber Corporation filed its opposition to the application for registration on grounds that:

a] The trademark sought to be registered is confusingly similar to the word "CONVERSE" which is part of petitioner's corporate name "CONVERSE RUBBER CORPORATION" as to likely deceive purchasers of products on which it is to be used to an extent that said products may be mistaken by the unwary public to be manufactured by the petitioner; and,

b] The registration of respondent's trademark will cause great and irreparable injury to the business reputation and goodwill of petitioner in the Philippines and would cause damage to said petitioner within the, meaning of Section 8, R.A. No. 166, as amended.

At the trial, petitioner's lone witness, Mrs. Carmen B. Pacquing, a duly licensed private merchant, testified that she had been selling CONVERSE rubber shoes in the local market since 1956. In the invoices issued by her store, the rubber shoes were described as "Converse Chuck Taylor", "Converse All Star," "All Star Converse Chuck Taylor," or "Converse Shoes Chuck Taylor."

Respondent, on the other hand, presented as its lone witness the secretary of said corporation who testified that respondent has been selling on wholesale basis "Universal Converse" sandals since 1962 and "Universal Converse" rubber shoes since 1963. Invoices were submitted as evidence of such sales. The witness also testified that she had no idea why respondent chose "Universal Converse" as a trademark and that she was unaware of the name "Converse" prior to her corporation's sale of "Universal Converse" rubber shoes and rubber sandals.

Eventually, the Director of Patents dismissed the opposition of the petitioner and gave due course to respondent's application because Opposer failed to present proof that the single word "CONVERSE' in its corporate name has become so identified with the corporation that whenever used, it designates to the mind of the public that particular corporation. Furthermore, inasmuch as the Opposer never presented any label herein, or specimen of its shoes, whereon the label may be seen, notwithstanding its witness' testimony touching upon her identification of the rubber shoes sold in her stores, no determination can be made as to whether the word 'CONVERSE' appears thereon. . . .the record is wanting in proof to establish likelihood of confusion so as to cause probable damage to the Opposer.”

MR denied. Hence, petitioner instituted the present petition for review.

Issue: Whether or not the respondent's partial appropriation of petitioner's corporate name is of such character that it is calculated to deceive or confuse the public to the injury of the petitioner to which the name belongs.

Held: Yes. Respondent’s application denied.A trade name is any individual name or surname, firm name, device or word used by

manufacturers, industrialists, merchants and others to identify their businesses, vocations or occupations. As the trade name refers to the business and its goodwill ... the trademark refers to the

59

Page 60: IP Developed Reviewer

goods." The ownership of a trademark or tradename is a property right which the owner is entitled to protect "since there is damage to him from confusion or reputation or goodwill in the mind of the public as well as from confusion of goods. The modern trend is to give emphasis to the unfairness of the acts and to classify and treat the issue as fraud.

From a cursory appreciation of the petitioner's corporate name "CONVERSE RUBBER CORPORATION,' it is evident that the word "CONVERSE" is the dominant word which identifies petitioner from other corporations engaged in similar business. Respondent, in the stipulation of facts, admitted petitioner's existence since 1946 as a duly organized foreign corporation engaged in the manufacture of rubber shoes. This admission necessarily betrays its knowledge of the reputation and business of petitioner even before it applied for registration of the trademark in question. Knowing, therefore, that the word "CONVERSE" belongs to and is being used by petitioner, and is in fact the dominant word in petitioner's corporate name, respondent has no right to appropriate the same for use on its products which are similar to those being produced by petitioner.

A corporation is entitled to the cancellation of a mark that is confusingly similar to its corporate name." "Appropriation by another of the dominant part of a corporate name is an infringement."

Respondent's witness had no idea why respondent chose "UNIVERSAL CONVERSE" as trademark and the record discloses no reasonable explanation for respondent's use of the word "CONVERSE" in its trademark. Such unexplained use by respondent of the dominant word of petitioner's corporate name lends itself open to the suspicion of fraudulent motive to trade upon petitioner's reputation.

The testimony of petitioner's witness, who is a legitimate trader as well as the invoices evidencing sales of petitioner's products in the Philippines, give credence to petitioner's claim that it has earned a business reputation and goodwill in this country. The sales invoices submitted by petitioner's lone witness show that it is the word "CONVERSE" that mainly identifies petitioner's products, i.e. "CONVERSE CHUCK TAYLOR, "CONVERSE ALL STAR," ALL STAR CONVERSE CHUCK TAYLOR," or "CONVERSE SHOES CHUCK and TAYLOR." Thus, contrary to the determination of the respondent Director of Patents, the word "CONVERSE" has grown to be identified with petitioner's products, and in this sense, has acquired a second meaning within the context of trademark and tradename laws.

Furthermore, said sales invoices provide the best proof that there were actual sales of petitioner's products in the country and that there was actual use for a protracted period of petitioner's trademark or part thereof through these sales.

The sales of 12 to 20 pairs a month of petitioner's rubber shoes cannot be considered insignificant, considering that they appear to be of high expensive quality, which not too many basketball players can afford to buy. Such actual sale of goods in the local market establishes trademark use which serves as the basis for any action aimed at trademark pre-exemption. It is a corollary logical deduction that while Converse Rubber Corporation is not licensed to do business in the country and is not actually doing business here, it does not mean that its goods are not being sold here or that it has not earned a reputation or goodwill as regards its products.

The trademark of respondent "UNIVERSAL CONVERSE and DEVICE" is imprinted in a circular manner on the side of its rubber shoes. In the same manner, the trademark of petitioner which reads "CONVERSE CHUCK TAYLOR" is imprinted on a circular base attached to the side of its rubber shoes. The deteminative factor in ascertaining whether or not marks are confusingly similar to each other "is not whether the challenged mark would actually cause confusion or deception of the purchasers but whether the use of such mark would likely cause confusion or mistake on the part of the buying public.

60

Page 61: IP Developed Reviewer

The similarity in the general appearance of respondent's trademark and that of petitioner would evidently create a likelihood of confusion among the purchasing public. But even assuming, arguendo, that the trademark sought to be registered by respondent is distinctively dissimilar from those of the petitioner, the likelihood of confusion would still subsists, not on the purchaser's perception of the goods but on the origins thereof . "The risk of damage is not limited to a possible confusion of goods but also includes confusion of reputation if the public could reasonably assume that the goods of the parties originated from the same source.

Article 8 of the Convention of the Union of Paris for the Protection of Industrial Property, of which the Philippines became a party on Sept. 27, 1965, provides that "a TRADE NAME [corporate name] shall be protected in all the countries of the Union without the obligation of filing or registration, whether or not it forms part of the trademark. " (finds implementation in Sec.37 of RA 166)

Philips Export BV v. CA, GR No. 96161, Feb. 21, 1992Facts: Petitioner Philips Export B.V. (PEBV) , a foreign corporation organized under the laws of the

Netherlands, although not engaged in business here, is the registered owner of the trademarks PHILIPS and PHILIPS SHIELD EMBLEM under Certificates of Registration Nos. R-1641 and R-1674, respectively issued by the Philippine Patents Office. Petitioners Philips Electrical Lamps, Inc. (Philips Electrical, for brevity) and Philips Industrial Developments, Inc. (Philips Industrial, for short), authorized users of the trademarks PHILIPS and PHILIPS SHIELD EMBLEM, were incorporated on 29 August 1956 and 25 May 1956, respectively. All petitioner corporations belong to the PHILIPS Group of Companies.

Respondent Standard Philips Corporation (Standard Philips) , on the other hand, was issued a Certificate of Registration by respondent Commission on 19 May 1982.

On 24 September 1984, Petitioners filed a letter complaint with SEC asking for the cancellation of the word "PHILIPS" from Private Respondent's corporate name in view of the prior registration with the Bureau of Patents of the trademark "PHILIPS" and the logo "PHILIPS SHIELD EMBLEM" in the name of Petitioner, PEBV, and the previous registration of Petitioners Philips Electrical and Philips Industrial with the SEC.

As a result of Private Respondent's refusal to amend its Articles of Incorporation, Petitioners filed with SEC a petition praying for the issuance of a writ of preliminary injunction alleging among others that Private Respondent's use of the word PHILIPS amounts to an infringement and clear violation of Petitioners' exclusive right to use the same considering that both parties engage in the same business.

In its Answer, Private Respondent countered that Petitioner PEBV has no legal capacity to sue; that its use of its corporate name is not at all similar to Petitioners' trademark PHILIPS when considered in its entirety; and that its products consisting of chain rollers, belts, bearings and cutting saw are grossly different from Petitioners' electrical products.

SEC denied issuance of writ because no sufficient ground was shown. Besides, Section 18 of the Corporation Code (infra) is applicable only when the corporate names in question are identical. Here, there is no confusing similarity between Petitioners' and Private Respondent's corporate names as those of the Petitioners contain at least two words different from that of the Respondent.

Petitioner’s MR likewise denied. On appeal, the SEC en banc affirmed. On 30 January 1990, Petitioners sought an extension of time to file a Petition for Review. CA affirmed dismissal and observed that the Converse case is not four-square with the present

61

Page 62: IP Developed Reviewer

case inasmuch as the contending parties in Converse are engaged in a similar business, that is, the manufacture of rubber shoes. Upholding the SEC, the CA concluded that "private respondents' products consisting of chain rollers, belts, bearings and cutting saw are unrelated and non-competing with petitioners' products i.e. electrical lamps such that consumers would not in any probability mistake one as the source or origin of the product of the other."

MR denied.

Issue: Whether or not the tradenames are confusingly similar.

Held: Yes. SEC ordered to amend private respondent's Articles of Incorporation by deleting the word PHILIPS from the corporate name of private respondent.

Our own Corporation Code, in its Section 18, expressly provides that:No corporate name may be allowed by the Securities and Exchange Commission if the

proposed name is identical or deceptively or confusingly similar to that of any existing corporation or to any other name already protected by law or is patently deceptive, confusing or contrary to existing law. Where a change in a corporate name is approved, the commission shall issue an amended certificate of incorporation under the amended name.

The statutory prohibition cannot be any clearer. To come within its scope, two requisites must be proven, namely:(1) that the complainant corporation acquired a prior right over the use of such corporate name; and(2) the proposed name is either:

(a) identical; or(b) deceptively or confusingly similar to that of any existing corporation or to any other name

already protected by law; or(c) patently deceptive, confusing or contrary to existing law.

The right to the exclusive use of a corporate name with freedom from infringement by similarity is determined by priority of adoption . In this regard, there is no doubt with respect to Petitioners' prior adoption of' the name ''PHILIPS" as part of its corporate name. Petitioners Philips Electrical and Philips Industrial were incorporated on 29 August 1956 and 25 May 1956, respectively, while Respondent Standard Philips was issued a Certificate of Registration on 12 April 1982, 26 years later. Petitioner PEBV has also used the trademark "PHILIPS" on electrical lamps of all types and their accessories since 30 September 1922, as evidenced by Certificate of Registration No. 1651.

The second requisite no less exists in this case. In determining the existence of confusing similarity in corporate names, the test is whether the similarity is such as to mislead a person, using ordinary care and discrimination. In so doing, the Court must look to the record as well as the names themselves. While the corporate names of Petitioners and Private Respondent are not identical, a reading of Petitioner's corporate names, to wit: PHILIPS EXPORT B.V., PHILIPS ELECTRICAL LAMPS, INC. and PHILIPS INDUSTRIAL DEVELOPMENT, INC., inevitably leads one to conclude that "PHILIPS" is, indeed, the dominant word in that all the companies affiliated or associated with the principal corporation, PEBV, are known in the Philippines and abroad as the PHILIPS Group of Companies.

Respondents maintain, however, that Petitioners did not present an iota of proof of actual confusion or deception of the public much less a single purchaser of their product who has been deceived or confused or showed any likelihood of confusion. It is settled, however, that proof of actual confusion need not be shown. It suffices that confusion is probably or likely to occur.

Other notes: Private Respondent’s and Petitioner’s products fall under the same category in the

62

Page 63: IP Developed Reviewer

international class. As aptly pointed out by Petitioners, Private Respondent's choice of "PHILIPS" as part of its

corporate name [STANDARD PHILIPS CORPORATION] . . . tends to show said respondent's intention to ride on the popularity and established goodwill of said petitioner's business throughout the world". The subsequent appropriator of the name or one confusingly similar thereto usually seeks an unfair advantage, a free ride of another's goodwill.

What is lost sight of, however, is that PHILIPS is a trademark or trade name, which was registered as far back as 1922. Petitioners, therefore, have the exclusive right to its use, which must be free from any infringement by similarity.

The fact that there are other companies engaged in other lines of business using the word "PHILIPS" as part of their corporate names is no defense and does not warrant the use by Private Respondent of such word which constitutes an essential feature of Petitioners' corporate name previously adopted and registered and-having acquired the status of a well-known mark in the Philippines and internationally as well.

2. Copyright and Trademarks

United Feature Syndicate v. Munsingwear,, G.R. No. 76193, Nov. 9, 1989Facts: Petitioner United Feature Syndicate filed for cancellation of the registration of trademark Charlie

Brown in the name of Munsingwear alleging that petitioner is damaged by the registration of the trademark Charlie Brown of t-shirts in the name of Munsingwear Creation Manufacturing Co., Inc on the following grounds:o CHARLIE BROWN is a character creation or a pictorial illustration, the copyright to which is

exclusively owned worldwide by the petitionero as the owner of the pictorial illustration CHARLIE BROWN, petitioner has since 1950 and

continuously up to the present, used and reproduced the same to the exclusion of otherso Munsingwear has no bona fide use of the trademark in commerce in the Philippines prior to its

application for registration Director of the Phil. Patent Office rendered decision holding that a copyright registration like that of

the name and likeness of Charlie Brown may not provide a cause of action for the cancellation of a trademark registration .

United Feature filed MR but the same did not contain a notice of hearing making it nothing but a piece of scrap paper that did not toll the period for appeal. The law office realizing that its MR didn’t contain a notice of hearing, subsequently filed the same but after the reglementary period for appeal. Decision of Phil. Patent Office became final and executory.

CA agreed with Phil. Patent Office that the decision had already become final and executory at the time the Notice of Appeal was filed.

Petitioner’s contention: it will be damaged by the registration of the trademark CHARLIE BROWN & DEVICE in favor of private respondent and that it has a better right to CHARLIE BROWN & DEVICE since the likeness of CHARLIE BROWN appeared in periodicals having worldwide distribution and covered by copyright registration in its name which antedates the certificate of registration of respondent issued only on September 12, 1979.

Respondent’s defense: It uses the trademark "CHARLIE BROWN" & "DEVICE" on children's wear such as T-shirts, undershirts, sweaters, brief and sandos, in class 25; whereas "CHARLIE BROWN" is used only by petitioner as character, in a pictorial illustration used in a comic

63

Page 64: IP Developed Reviewer

strip appearing in newspapers and magazines. It has no trademark significance and therefore respondent-registrant's use of "CHARLIE BROWN" & "DEVICE" is not in conflict with the petitioner's use of "CHARLIE BROWN".

 Issues / Held:1.)     W/N CA erred when it dismissed petitioner’s appeal on strict technical rules of procedure.

YES. Petitioner's contention that the purpose of a notice of hearing to the adverse party is to afford him an opportunity to resist the motion, more particularly the MR filed by its company is well taken. Said purpose was served when Munsingwear filed its opposition thereto on November 20, 1984 and cured the technical defect of lack of notice of hearing complained of. Otherwise stated such shortcomings of petitioner as to compliance with procedural requirements in taking its appeal cannot be deemed sufficient to deprive it of a chance to secure a review by this court in order to obtain substantial justice; more so where liberality accorded to the petitioner becomes compelling because of the ostensible merit of petitioner's case. the petitioner's delay in filing their record on appeal should not be strictly construed as to deprive them of the right to appeal especially since on its face the appeal appears to be impressed with merit. 2.)     W/N petitioner will sustain damage by the registration of the trademark Charlie Brown and Device in favor of Munsingwear.

YES. Certificate of registration issued to Munsingwear cancelled. Aside from its copyright registration, petitioner is also the owner of several trademark registrations and application for the name and likeness of "CHARLIE BROWN" which is the duly registered trademark and copyright of petitioner United Feature Syndicate Inc. as early as 1957 and additionally also as TV SPECIALS featuring the "PEANUTS" characters "CHARLIE BROWN". Section 2 of Presidential Decree No. 49, otherwise known as the "Decree on Intellectual Property", provides:Section 2. The rights granted by this Decree shall, from the moment of creation, subsist with respect to any of the following classes of works:xxx xxx xxx(O) Prints, pictorial illustrations, advertising copies, labels, tags and box wraps 

It is undeniable from the records that petitioner is the actual owner of said trademark due to its prior registration with the Patent's Office. In upholding the right of the petitioner to maintain the present suit before our courts for unfair competition or infringement of trademarks of a foreign corporation, we are moreover recognizing our duties and the rights of foregoing states under the Paris Convention for the Protection of Industrial Property to which the Philippines and (France) U.S. are parties. We are simply interpreting a solemn international commitment of the Philippines embodied in a multilateral treaty to which we are a party and which we entered into because it is in our national interest to do so.

Wilson Ong Ching Kian Chuan vs. CA, G.R. No. 130360, Aug. 15, 2001Facts: Petitioner Ong imports vermicelli from China National Cereals Oils and Foodstuffs Import and

Export Corporation, based in Beijing , China , under the firm name C.K.C. Trading. He repacks it in cellophane wrappers with a design of two-dragons and the TOWER trademark on the uppermost portion. Ong acquired a certificate of copyright registration for this.

Ong discovered that PR Tan repacked his vermicelli he imports from the same company but based

64

Page 65: IP Developed Reviewer

in Qingdao, China in a "nearly" identical wrapper causing him to file a complaint for infringement with damages and prayer for TRO or writ of preliminary injunction.

RTC issued TRO on the same date complaint was filed. Tan filed an opposition with counter-application for the issuance of a similar writ against Ong. He

alleged that Ong was not entitled to an injunction because he did not have a clear right over the use of the trademark Pagoda and Lungkow vermicelli as these were registered in the name of Ceroilfood Shandong.

Ong countered Tan’s opposition to the issuance of the writ of preliminary injunction by filing a bond and the RTC denied Tan’s motion to dissolve the same.

Tan elevated the case to the CA via a special civil action for certiorari with a prayer for the issuance of a TRO and/or writ of preliminary injunction to which Ong filed an opposition.

CA gave due course to Tan’s petition and set aside RTC orders including the writ of preliminary injunction. Upon Ong’s MR, CA decision was modified and a writ of injunction was issued enjoining Ong and any and all persons acting for and in their behalf from enforcing and/or implementing the Writ of Preliminary Injunction.

Pursuant to the above resolution of the CA, parties submitted their memoranda and a decision was rendered making the injunction against Ong permanent.

 Issue: W/N issuance of a writ of preliminary injunction was proper. Ong’s contention: He is more entitled to injunction. He states that as holder of the Certificate of Copyright Registration of the twin-dragon design, he has the protection of P.D. No. 49. Tan’s contention: Petitioner has no "clear right" over the use of the copyrighted wrapper since the PAGODA trademark and label were first adopted and used and have been duly registered by Ceroilfood Shandong not only in China but in nearly 20 countries and regions worldwide. Petitioner was not the original creator of the label, but merely copied the design of Ceroilfood Shandong. Private respondent alleges, that the trademark PAGODA BRAND was registered in China on October 31, 1979 while the trademark LUNGKOW VERMICELLI WITH TWO-DRAGON DEVICE was registered on August 15, 1985. Held: NO. Injunction not proper because Ong’s right to the same was not clearly demonstrated. To be entitled to an injunctive writ, petitioner must show, inter alia, the existence of a clear and unmistakable right and an urgent and paramount necessity for the writ to prevent serious damage. In Developers Group of Companies, Inc. vs. Court of Appeals, 219 SCRA 715, 722-723 (1993), we held that in the absence of proof of a legal right and the injury sustained by the plaintiff, an order of the trial court granting the issuance of an injunctive writ will be set aside, for having been issued with grave abuse of discretion. Conformably, there was no abuse of discretion by the CA when it issued its own order to restrain the enforcement of the preliminary injunction issued by the trial court. 

However, it was premature for the CA to declare that the design of petitioner's wrapper is a copy of the wrapper allegedly registered by Ceroilfood Shandong. That matter remains for decision after appropriate proceedings at the trial court. The only issue brought before the CA involved the grave abuse of discretion allegedly committed by the trial court in granting the writ of preliminary injunction. The CA in declaring that the wrapper of petitioner is a copy of Ceroilfood Shandong's wrapper went beyond that issue touched on the merits of the infringement case, which remains to be

65

Page 66: IP Developed Reviewer

decided by the trial court.

On the issue of preliminary injunctionA person to be entitled to a copyright must be the original creator of the work. He must

have created it by his own skill, labor and judgment without directly copying or evasively imitating the work of another. The grant of preliminary injunction in a case rests on the sound discretion of the court with the caveat that it should be made with extreme caution. Its grant depends chiefly on the extent of doubt on the validity of the copyright, existence of infringement, and the damages sustained by such infringement. In our view, the copies of the certificates of copyright registered in the name of Ceroilfood Shandong sufficiently raise reasonable doubt. With such a doubt preliminary injunction is unavailing.

Kho v. CA, supra

B. Spectrum Approach: Fanciful or Arbitrary; Suggestive; Descriptive; Generic- fanciful is afforded the greatest protection- - generic should not be granted

C. Actual Use in Commerce Requirement (Sec. 122, 124.2, Rule 204, 205, IRR – Trademarks)

Section 122. How Marks are Acquired. - The rights in a mark shall be acquired through registration made validly in accordance with the provisions of this law. (Sec. 2-A, R A. No. 166a)

Section 124.2. The applicant or the registrant shall file a declaration of actual use of the mark with evidence to that effect, as prescribed by the Regulations within 3 years from the filing date of the application . Otherwise, the application shall be refused or the mark shall be removed from the Register by the Director.

RULE 204. Declaration of Actual Use. - The Office will not require any proof of use in commerce in the processing of trademark applications. However, without need of any notice from the Office, all applicants or registrants shall file a declaration of actual use of the mark with evidence to that effect within 3 years, without possibility of extension, from the filing date of the application. Otherwise, the application shall be refused or the mark shall be removed from the register by the Director motu proprio.

RULE 205. Contents of the Declaration and Evidence of Actual Use. - The declaration shall be under oath, must refer to only one application or registration, must contain the name and address of the applicant or registrant declaring that the mark is in actual use in the Philippines, list of goods where the mark is attached; list the name or names and the exact location or locations of the outlet or outlets where the products are being sold or where the services are being rendered, recite sufficient facts to show that the mark described in the application or registration is being actually used in the Philippines and, specifying the nature of such use. The declarant shall attach five labels as actually used on the goods or the picture of the stamped or marked container visibly and legibly showing the mark as well as proof of payment of the prescribed fee. [As amended by Office Order No. 08 (2000)] (2 declarations, 1 within 3 years from application & another within 1 year from the 5th year AND every 5 years thereafter)

66

Page 67: IP Developed Reviewer

Kabushi Kaisha Isetan v. IAC, G.R. No. 75420, Nov. 15, 1991Facts: Petitioner Kabushi Kaisha Isetan is a foreign corporation and is the owner of the trademark "Isetan"

and the "Young Leaves Design". The petitioner alleges that it first used the trademark Isetan on November 5, 1936. It states that the

trademark is a combination of "Ise" taken from "Iseya" the first name of the rice dealer in Kondo, Tokyo in which the establishment was first located and "Tan" which was taken from "Tanji Kosuge the First". The petitioner claims to have expanded its line of business internationally from 1936 to 1974. The trademark "Isetan" and "Young Leaves Design" were registered in Japan covering more than 34 classes of goods. On October 3, 1983, the petitioner applied for the registration of "Isetan" and "Young Leaves Design" with the Philippine Patent Office under Permanent Serial Nos. 52422 and 52423 respectively.

Private respondent, Isetann Department Store , on the other hand, is a domestic corporation that claims that it used the word "Isetann" as part of its corporated name and on its products particularly on shirts in Joymart Department Store sometime in January 1979. The suffix "Tann" means an altar, the place of offering in Chinese and this was adopted to harmonize the corporate name and the corporate logo of two hands in cup that symbolizes the act of offering to the Supreme Being for business blessing.

On May 30, 1980 and May 20, 1980, the private respondent registered "Isetann Department Store, Inc." and Isetann and Flower Design in the Philippine Patent Office.

On November 28, 1980, the petitioner filed with the Phil. Patent Office 2 petitions for the cancellation of Certificates of Supplemental Registration stating among others that. . . except for the additional letter "N" in the word "Isetan", the mark registered by the registrant is exactly the same as the trademark ISETAN owned by the petitioner and that the young leaves registered by the registrant is exactly the same as the young leaves design owned by the petitioner.

Petitioner further alleged that PR merely wants to ride on the goodwill and popularity of petitioner’s trademark thereby causing great and irreparable injury and damage to it. It argued that both the petitioner's and respondent's goods move in the same channels of trade, and ordinary people will be misled to believe that the products of the private respondent originated or emanated from, are associated with, or are manufactured or sold, or sponsored by the petitioner by reason of the use of the challenged trademark.

The petitioner also invoked the Convention of Paris of March 20, 1883 for the Protection of IP of which the Philippines and Japan are both members. (re: commitment of the government to the protection of trademarks belonging not only to Filipino citizens but also to those belonging to nationals of other member countries who may seek protection in the Philippines)

Meanwhile, the petitioner also filed with SEC a petition to cancel the mark "ISETAN" as part of the registered corporate name of Isetann Department Store, Inc. but petition was denied.

Commission reversed decision of Hearing Officer on appeal but upon MR, initial denial was reinstated. The Commission stated that since the petitioner's trademark and tradename have never been used in commerce on the petitioner's products marketed in the Philippines, the trademark or tradename have not acquired a reputation and goodwill deserving of protection from usurpation by local competitors.

This SEC decision, which denied and dismissed the petition to cancel was submitted to the Director of Patents as part of the evidence for the PR.

The Director of Patents after notice and hearing rendered a joint decision in Inter Partes Cases and ruled that petitioner has not made out a case for cancellation. The corresponding application for registration in the Principal Register of the Trademark and of the tradename aforesaid are hereby

67

Page 68: IP Developed Reviewer

given due course. Petitioner filed MR. Denied. Petitioner appealed to CA. Dismissed for being filed out of time. MR also denied.

Issues/ Held:1) Is the present petition filed out of time?

Yes but on equitable grounds stating that it has a strong and meritorious case, the petition was given due course to enable us to examine more fully any possible denial of substantive justice.

2) On substantial grounds, should petitioner’s petition for cancellation of PR’s registration be given due course?

No. A fundamental principle of Philippine Trademark Law is that actual use in commerce in the Philippines is a pre-requisite to the acquisition of ownership over a trademark or a tradename.

The trademark Law, R.A.166, as amended, under which this case heard and decided provides:

SEC. 2. What are registrable.- Trademark, tradenames and service marks owned by persons, corporation, partnerships or associations domiciled in the Philippines and by persons, corporations, partnerships or associations domiciled in any foreign country may be registered in accordance with the provisions of this Act: Provided, That said trademarks, tradenames, or service marks are actually in use in commerce and services not less than two months in the Philippines before the time the applications for registration are filed: And provided, further, That the country of which the applicant for registration is a citizen grants by law substantially similar privileges to citizens of the Philippines, and such fact is officially certified, with a certified true copy of the foreign law translated into the English language, by the government of the foreign country to the Government of the Republic of the Philippines. (As amended by R.A. No. 865).

SEC. 2-A. Ownership of trademarks, tradenames and service marks; how acquired. - Anyone who lawfully produces or deals in merchandise of any kind or who engages in any lawful business, or who renders any lawful service in commerce, by actual use thereof in manufacture or trade, in business, and in the service rendered, may appropriate to his exclusive use a trademark, a tradename, or a service mark not so appropriated by another, to distinguish his merchandise, business or service from the merchandise, business or service of others. The ownership or possession of a trademark, tradename, service mark, heretofore or hereafter appropriated, as in this section provided, shall be recognized and protected in the same manner and to the same extent as are other property rights known to the law. (As amended by R.A. No. 638)"

These provisions have been interpreted in Sterling Products International, Inc. v. Farbenfabriken Bayer Actiengesellschaft in this way:

A rule widely accepted and firmly entrenched because it has come down through the years is that actual use in commerce or business is a prerequisite to the acquisition of the right of ownership over a trademark.xxx xxx xxx. .. Adoption alone of a trademark would not give exclusive right thereto. Such right grows out of their actual use. Adoption is not use. One way make advertisements, issue circulars, give out price lists on certain goods; but these alone would not give exclusive right of use. For trademark is a creation of use. The underlying reason for all these is that purchasers have come to understand the mark as indicating the origin of the wares. Flowing from this is the trader's right to

68

Page 69: IP Developed Reviewer

protection in the trade he has built up and the goodwill he has accumulated from use of the trademark. ...

In fact, a prior registrant cannot claim exclusive use of the trademark unless it uses it in commerce.

The records show that the petitioner has never conducted any business in the Philippines. It has never promoted its tradename or trademark in the Philippines. It has absolutely no business goodwill in the Philippines. It is unknown to Filipinos except the very few who may have noticed it while travelling abroad. It has never paid a single centavo of tax to the Philippine government. Under the law, it has no right to the remedy it seeks.

The mere origination or adoption of a particular tradename without actual use thereof in the market is insufficient to give any exclusive right to its use, even though such adoption is publicly declared, such as by use of the name in advertisements, circulars, price lists, and on signs and stationery.

The Paris Convention for the Protection of Industrial Property does not automatically exclude all countries of the world, which have signed it from using a tradename which happens to be used in one country. To illustrate - If a taxicab or bus company in a town in the United Kingdom or India happens to use the tradename "Rapid Transportation", it does not necessarily follow that "Rapid" can no longer be registered in Uganda, Fiji, or the Philippines.

Indeed, the Philippines is a signatory to this Treaty and, hence, we must honor our obligation thereunder on matters concerning internationally known or well known marks. However, this Treaty provision clearly indicated the conditions which must exist before any trademark owner can claim and be afforded rights such as the Petitioner herein seeks and those conditions are that:a) the mark must be internationally known or well known;b) the subject of the right must be a trademark, not a patent or copyright or anything else;c) the mark must be for use in the same or similar kinds of goods; andd) the person claiming must be the owner of the mark

The respondent registered its trademark in 1979. It has continuously used that name in commerce. It has established a goodwill through extensive advertising. The people who buy at Isetann Store do so because of Isetann's efforts. There is no showing that the Japanese firm's registration in Japan or HK has any influence whatsoever on the Filipino buying public.

Note: Paris Convention refers to internationally well-known MARKS not tradenames.

Sterling Products v. Farben Fabrieken Bayer, G.R. No. L-19906, April 30, 1969 Facts: In this, a case for trademark infringement and unfair competition, each of the principal suitors,

namely, plaintiff Sterling Products International, Inc. (SPI), and defendant Farbenfabriken Bayer Aktiengesellschaft (FBA), seeks to exclude the other from use in the Philippines of the trademarks BAYER and BAYER CROSS IN CIRCLE.

SPI asks this Court to strike down FBA's registration of BAYER CROSS IN CIRCLE covering industrial and agricultural products, insecticides and other chemicals, not medicines, from the supplemental register.

FBA, for its part, prays for the cancellation from the principal register of SPI's certificates of registration of the trademarks aforesaid for medicines.

Contending parties are doing business in the Philippines. SPI markets Bayer Aspirin, Aspirin for Children and Cafiaspirina. The BAYER and BAYER CROSS

IN CIRCLE are being used by SPI in the Philippines only for said products: Bayer Aspirin,

69

Page 70: IP Developed Reviewer

Cafiaspirina and Bayer Aspirin for Children. On the containers (bottles or printed celophane strips, which, in turn, are placed in cardboard boxes) of Bayer Aspirin, Aspirin for Children and Cafiaspirina, SPI features the trademarks BAYER and BAYER CROSS IN CIRCLE.

FBA thru Allied Manufacturing & Trading Co., Inc. distributes "Folidol" and other industrial and agricultural chemicals. FBA's "Folidol" (in steel or fiber drums or aluminum containers) displays, a replica of SPI's trademark BAYER CROSS IN CIRCLE; on the tin cap and label of the container.

The conflict apparent, suit followed. TC: dismissed complaint & counterclaim and declared itself to be "in favor of the solution that

favors division of the market rather than monopoly." But to avoid confusion, it directed defendants "to add a distinctive word, or words in their mark to indicate that their products come from Germany."         

Both parties appealed: Plaintiff, insofar as the judgment "dismisses plaintiff's complaint and sustains defendants' right to

use the BAYER trademark for their chemicals, insecticides, and other products not medicines"; and defendants, from the portions of the aforementioned decision particularly those which dismiss the

counterclaim of the defendants for the cancellation of the registrations by the plaintiff of the trademarks Bayer and Bayer Cross and which allow the plaintiff "to continue using the Bayer trademarks for medicines."

Issues/Held: 1) Whether or not it is proper to give unqualified protection to plaintiff against the use of said trademarks by all others on goods other than medicine.

NO (Decision of TC affirmed). A rule widely accepted and firmly entrenched because it has come down through the years is that actual use in commerce or business is a prerequisite to the acquisition of the right of ownership over a trademark. This rule is spelled out in our Trademark Law thus:          SEC. 2-A. Ownership of trademarks, trademark names and service-mark; how acquired. Anyone who lawfully produces or deals in merchandise of any kind or who engages in any lawful business, or who renders any lawful service in commerce, by actual use thereof in manufacture or trade, in business, and in the service rendered, may appropriate to his, exclusive use a trademark, a trade-name, or a service-mark not so appropriated by another, to distinguish his merchandise, business, or service from the merchandise, business or service of others. The ownership or possession of a trademark, trade-name, service mark, heretofore or hereafter appropriated, as in this section provided, shall be recognized and protected in the same manner and to the same extent as are other property rights known to the law. (As inserted by Section 1 of Republic Act 638)          It would seem quite clear that adoption alone of a trademark would not give exclusive right thereto. Such right grows out of their actual use." Adoption is not use.

One may make advertisements, issue circulars, give out price lists on certain goods; but these alone would not give exclusive right of use. For trademark is a creation of use. The underlying reason for all these is that Purchasers have come to understand the mark as indicating the origin of the wares. Flowing from this is the trader's right to protection in the trade he has built up and the goodwill he has accumulated from use of the trademark. Registration does not, however, perfect a trademark right.          The BAYER trademarks registered in the Philippines to which plaintiff SPI may lay claim, as correctly stated in the decision below, are those which cover medicines only. For, it was on said goods that the BAYER trademarks were actually used by it in the Philippines. Therefore, the certificates of registration for medicines issued by the Director of Patents upon which the protection is enjoyed are

70

Page 71: IP Developed Reviewer

only for medicines. Nothing in those certificates recited would include chemical or insecticides.xxx                   Really, if the certificate of registration were to be deemed as including goods not specified

therein, then a situation may arise whereby an applicant may be tempted to register a trademark on any and all goods which his mind may conceive even if he had never intended to use the trademark for the said goods. We believe that such omnibus registration is not contemplated by our Trademark Law.          Because of this and of the fact that the Bayer trademarks were never used in the Philippines by plaintiff except for medicines Aspirin, Aspirin for Children and Cafiaspirina we find ourselves unwilling to draw a hard and fast rule which would absolutely and under all circumstances give unqualified protection to plaintiff against the use of said trademarks by all others on goods other than medicines.

Doctrine of Nationality or TerritorialityNeither will the 1927 registration in the US of the BAYER trademark for insecticides serve

plaintiff any. The US is not the Phil. Registration in the US is not registration in the Phil. At the time of the US registration in 1927, we had our own Trademark Law, Act No. 666 of the Phil. Commission, which provided for registration here of trademarks owned by persons domiciled in the US.         Two Types of Confusion

1. confusion of goods- "in which event the ordinarily prudent purchaser would be induced to purchase one product in the belief that he was purchasing the other."

2. confusion of business: "Here though the goods of the parties are different, the defendant's product is such as might reasonably be assumed to originate with the plaintiff, and the public would then be deceived either into that belief or into the belief that there is some connection between the plaintiff and defendant which, in fact, does not exist."

The courts have come to realize that there can be unfair competition or unfair trading even if the goods are noncompeting, and that such unfair trading can cause injury or damage to the first user of a given trademark, first, by prevention of the natural expansion of his business and, second, by having his business reputation confused with and put at the mercy of the second user.

This Court is also impelled to believe that the evidence establishes that among the common people of the Philippines the "Bayer" medicines come from Germany.          There is no whittling away of the identity of plaintiff's trademarks. Plaintiff is not the first user thereof in the Philippines. The trademarks do not necessarily link plaintiff with the public. Plaintiff must show injury; it has not. On the contrary, representations as to the place of manufacture of plaintiff's medicines were untrue, misleading. Plaintiff could still be tagged with the same deception "which (it) complains of in the defendant(s).”

Note: Bad faith on SPL because it rode on the goodwill of FBA’s predecessor. Lack of protest is a sign of acquiescence. Shangri-La Int’l v. Developer’s Group of Companies, G.R. No. 159938, March 31, 2006Facts: DGCI filed on Oct. 1982 with BPTTT an application for registration covering the “Shangri-La” mark

and “S” logo. In 1983 it was issued a certificate of registration. Kuok family on the other hand owns and operates a chain of hotels with interest in hotels and hotel-

related transactions since 1969. As far back as 1962, it adopted the name "Shangri-La" as part of

71

Page 72: IP Developed Reviewer

the corporate names of all companies organized under the Kuok Group. Kuok Group had incorporated in HK and Singapore, among other places, several companies that

form part of the Shangri-La International Hotel Management Ltd. Group of Companies. EDSA Shangri-La Hotel and Resort, Inc., and Makati Shangri-La Hotel and Resort, Inc. were

incorporated in the Philippines beginning 1987 to own and operate the 2 hotels put up by the Kuok Group in Mandaluyong and Makati . All hotels adopted and used the distinctive lettering of the name "Shangri-La" as part of their trade names.

Shangi-La Hotel S’pore commissioned a Singaporean design artist, a certain Mr. William Lee, to conceptualize and design the logo of the hotels. This logo, which is a bold, striking definitive design, embodies both modernity and sophistication in balance and thought.

Since 1975 and up to the present, the "Shangri-La" mark and "S" logo have been used consistently and continuously by all Shangri-La hotels and companies in their paraphernalia, such as stationeries, envelopes, business forms, menus, displays and receipts. The Kuok group registered the mark and logo in patent offices around the world. However it wasn’t until 1987 or 1988 that petitioners operated an establishment in the Philippines albeit they advertised their hotels abroad since 1972 in numerous print media accessible in Philippine magazines and newsstands.

Petitioners filed with the BPTTT a petition for cancellation of the registration of the "Shangri-La" mark and "S" logo issued to respondent DGCI on the ground that the same were illegally and fraudulently obtained and appropriated for the latter's restaurant business.

DGCI filed a complaint for infringement and damages with RTC against SLIHM with injunctive relief alleging that DGCI has for the last 8 years prior exclusive use in the Philippines of the mark and logo in question and the registered owner thereof for its restaurant and allied services. They sought to prohibit petitioner from using the mark and logo for their hotels in the Philippines.

Petitioners accused DGCI of appropriating and illegally using the "Shangri-La" mark and "S" logo , adding that the legal and beneficial ownership thereof pertained to SLIHM and that the Kuok Group and its related companies had been using this mark and logo since March 1962 for all their corporate names and affairs. They invoked the Paris Convention for the Protection of Industrial Property as affording security and protection to SLIHM's exclusive right to said mark and logo, and also claimed that the same were internationally known since 1975.

Pending trial on the merits, RTC issued writ of preliminary injunction and although the parties filed a motion to suspend proceedings on account of the pendency before the BPTTT of Inter Partes Case for the cancellation of DGCI’s certificate of registration, it was the court who ordered the BPTTT to suspend further proceedings in said inter partes case and to await the final outcome of the main case.

Witnesses and Pieces of Evidence Presented by DGCI: Lone witness Syhunliong, Presidient and Chairman of DGCI’s Board of Directors said that the “S-

logo” was made for him by a jeepney signboard artist. Prior to its creation, DGCI was already incorporated with the primary purpose of "owning or operating, or both, of hotels and restaurants.

Oct. 1982, again prior to alleged creation of mark and logo DGCI filed an application for trademark registration of the mark "SHANGRI-LA FINEST CHINESE CUISINE & S Logo" with the BPTTT and amended its articles of incorporation to reflect the same. However it was only in December 1982 (2 ½ months after filing of application with BPTTT) that the restaurant was opened for business.

Syhunliong, as found by CA, had been a guest at the petitioners' hotel (HK) before he caused the registration of the mark and logo.

 

72

Page 73: IP Developed Reviewer

RTC ruling: In favor of DGCI and ordered petitioners to permanently cease and desist from using and/or continuing to use said mark and logo, or any copy, reproduction or colorable imitation.

CA affirmed RTC with the modification of deleting the award of attorney’s fees. CA’s ratio was that:Since the Kuok Group does not have proof of actual use in commerce in the Philippines (in accordance with Section 2 of R.A. No. 166), it cannot claim ownership of the mark and logo in accordance with the holding in Kabushi Kaisha Isetan v. IAC as reiterated in Philip Morris, Inc. v. CA.

The use of the mark or logo in commerce through the bookings made by travel agencies is unavailing since the Kuok Group did not establish any branch or regional office in the Philippines.

While the Paris Convention protects internationally known marks, R.A. 166 still requires use in commerce in the Philippines. Accordingly, and on the premise that international agreements, such as Paris Convention, must yield to a municipal law, the question on the exclusive right over the mark and logo would still depend on actual use in commerce in the Philippines.

Arguments of the parties in SC: Shangri-La and Kuok Group: DGCI had no right to file an application for registration of the

"Shangri-La" mark and "S" logo because it did not have prior actual commercial use thereof.

DGCI: Arguments raised by petitioners were questions of fact that were already resolved by the RTC with concurrence of CA.

 Issue: W/N petitioners were liable for infringement because it was DGCI who first registered “Shangri-la” and “S” logo. Held: NO. What was the law in force at the time DGCI filed an application for registration?

Former trademark law RA 166 and it required that at the time of application for registration of trademark, for the same to be the root of ownership, the trademark must be in actual use in commerce. Trademark is a creation of use and therefore actual use is a pre-requisite to exclusive ownership and its registration with the Philippine Patent Office is a mere administrative confirmation of the existence of such right.  Under Section 2, in order to register a trademark, one must be the owner thereof and must have actually used the mark in commerce in the Philippines for 2 months prior to the application for registration. Since "ownership" of the trademark is required for registration, Section 2-A of the same law sets out to define how one goes about acquiring ownership thereof. Under Section 2-A, it is clear that actual use in commerce is also the test of ownership but the provision went further by saying that the mark must not have been so appropriated by another. Hence, under R.A. 166, as amended, one may be an owner of a mark due to actual use thereof but not yet have the right to register such ownership here due to failure to use it within the Philippines for two months.  It was also only in December 1982 when the respondent's restaurant was opened for business. Respondent cannot now claim before the Court that the certificate of registration itself is proof that the 2-month prior use requirement was complied with, what with the fact that its very own witness testified otherwise in the TC. And because at the time (October 18, 1982) the respondent filed its application for trademark registration of the "Shangri-La" mark and "S" logo, respondent was not using these in the Philippines commercially, the registration is void.

73

Page 74: IP Developed Reviewer

  While the petitioners may not have qualified under Section 2 of R.A. No. 166 as a registrant, neither did respondent DGCI, since the latter also failed to fulfill the 2-month actual use requirement. What is worse, DGCI was not even the owner of the mark. For it to have been the owner, the mark must not have been already appropriated. Was there bad faith on DGCI’s end when it registered the mark?

Yes. It is truly difficult to understand why, of the millions of terms and combination of letters and designs available, the respondent had to choose exactly the same mark and logo as that of the petitioners, if there was no intent to take advantage of the goodwill of petitioners' mark and logo.  President of DGCI Syhunliong was a guest at petitioner’s hotel before he caused the registration of the mark and logo. When a trademark copycat adopts the word portion of another's trademark as his own, there may still be some doubt that the adoption is intentional. But if he copies not only the word but also the word's exact font and lettering style and in addition, he copies also the logo portion of the trademark, the slightest doubt vanishes. It is then replaced by the certainty that the adoption was deliberate, malicious and in bad faith. One who has imitated the trademark of another cannot bring an action for infringement, particularly against the true owner of the mark, because he would be coming to court with unclean hands. Priority is of no avail to the bad faith plaintiff. Good faith is required in order to ensure that a second user may not merely take advantage of the goodwill established by the true owner.  This point is further bolstered by the fact that under either Section 17 of R.A. No. 166, or Section 151 of R.A. No. 8293, or Article 6bis(3) of the Paris Convention, no time limit is fixed for the cancellation of marks registered or used in bad faith.

Conflict between our municipal law and the Paris Convention; Can petitioners rely on Paris Convention?

While the Paris Convention protects internationally known marks, R.A. No. 166 still requires use in commerce in the Philippines. Accordingly, and on the premise that international agreements, such as Paris Convention, must yield to a municipal law, the question on the exclusive right over the mark and logo would still depend on actual use in commerce in the Philippines.

Registration Alone InsufficientRegistration, without more, does not confer upon the registrant an absolute right to the

registered mark. The certificate of registration is merely a prima facie proof that the registrant is the owner of the registered mark or trade name. Evidence of prior and continuous use of the mark or trade name by another can overcome the presumptive ownership of the registrant and may very well entitle the former to be declared owner in an appropriate case.

One in Actual Use PrevailsRegistration in the Principal Register is limited to the actual owner of the trademark and

proceedings therein pass on the issue of ownership, which may be contested through opposition or interference proceedings, or, after registration, in a petition for cancellation. xxx

As between actual use of a mark without registration, and registration of the mark without actual use thereof, the former prevails over the latter. For a rule widely accepted and firmly entrenched, because it has come down through the years, is that actual use in commerce or business is a pre-requisite to the acquisition of the right of ownership. 

74

Page 75: IP Developed Reviewer

Philip Morris, Inc. v. Fortune Tobacco Company, G.R. No. 158589, June 27, 2006Facts: Philip Morris, Inc., is a foreign corporation organized under the laws of Virginia and was issued

certificate of registration by the PPO as the registered owner of the trademark “Mark VII” for its cigarettes.

Similarly, petitioner Benson & Hedges (Canada), Inc., a subsidiary of Philip Morris and is the registered owner of “Mark Ten” according to cert. of registration also issued by PPO.

On the other hand, respondent Fortune Tobacco Corporation, a company organized in the Philippines, manufactures and sells cigarettes using the trademark "MARK."

Petitioners claimed infringement of their respective trademarks against Fortune Tobacco with the RTC. Philip Morris claimed that without their prior consent, Fortune manufactured and sold cigarettes bearing the identical and/or confusingly similar trademark “Mark”. They alleged the following:

They are foreign corporations not doing business in the Philippines and are suing on an isolated transactiono countries in which they are domiciled grant to corporate or juristic persons of the Philippines

the privilege to bring action for infringement, without need of a license to do business in those countries

o registered the aforementioned trademarks in their respective countries of origin and that, by virtue of the long and extensive usage of the same, these trademarks have already gained international fame and acceptance

o Fortune’s “Mark” have caused and is likely to cause confusion or mistake, or would deceive purchasers and the public in general into buying these products under the impression and mistaken belief that they are buying Philip Morris products

o invoked Paris Convention in that under the same upon the request of an interested party, a country of the Union may prohibit the use of a trademark which constitutes a reproduction, imitation, or translation of a mark already belonging to a person entitled to the benefits of the said Convention

Fortune’s answer: "MARK" is a common word, which cannot particularly identify a product to be the product of the petitioners

RTC dismissed the complaint finding that Fortune did not commit trademark infringement because words "MARK", "TEN", "LARK" and the Roman Numerals "VII", either alone or in combination of each other do not by themselves or by association point distinctly to the origin or ownership of the cigarettes to which they refer, such that the buying public could not be deceived. As for Philip Morris’ capacity to sue, it was established that Philip Morris was indeed doing business in the Philippines contrary to its allegations that it is not and it had no license. Also, their failure to present evidence to support their allegation that their respective countries indeed grant Philippine corporations reciprocal or similar privileges by law justifies the dismissal of the complaint.

CA ruled that petitioners had legal capacity to sue but affirmed the ruling of the RTC that there was no infringement. Ratio: "MARK VII", "MARK TEN" and "LARK", do not qualify as well-known marks entitled to protection even without the benefit of actual use in the local market and that the similarities in the trademarks in question are insufficient as to cause deception or confusion tantamount to infringement.

 Issue: W/N Fortune committed trademark infringement against petitioners by its use of the “Mark: for its cigarettes, hence liable for damages 

75

Page 76: IP Developed Reviewer

Held: NO. In determining similarity and likelihood of confusion, jurisprudence has developed two tests: the dominancy test and the holistic test. The dominancy test sets sight on the similarity of the prevalent features of the competing trademarks that might cause confusion and deception, thus constitutes infringement. Under this norm, the question at issue turns on whether the use of the marks involved would be likely to cause confusion or mistake in the mind of the public or deceive purchasers. But, even if the dominancy test were to be used, as urged by the petitioners, but bearing in mind that a trademark serves as a tool to point out distinctly the origin or ownership of the goods to which it is affixed, the likelihood of confusion tantamount to infringement appears to be farfetched. The reason for the origin and/or ownership angle is that unless the words or devices do so point out the origin or ownership, the person who first adopted them cannot be injured by any appropriation or imitation of them by others, nor can the public be deceived.

In contrast, the holistic test entails a consideration of the entirety of the marks as applied to the products, including the labels and packaging, in determining confusing similarity. As it appears on the products of both Philip Morris and Fortune Tobacco, striking dissimilarities are significant enough to warn any purchaser that one is different from the other. Indeed, although the perceived offending word "MARK" is itself prominent in petitioners trademarks "MARK VII" and "MARK TEN," the entire marking system should be considered as a whole and not dissected, because a discerning eye would focus not only on the predominant word but also on the other features appearing in the labels. Since the word "MARK," be it alone or in combination with the word "TEN" and the Roman numeral "VII," does not point to the origin or ownership of the cigarettes to which they apply, the local buying public could not possibly be confused or deceived. On Philip Morris’ legal capacity to sue

It must be emphasized that Philip Morris’ standing to sue in Philippine courts had been recognized, and rightly so, by the CA. However, such right to sue does not necessarily mean protection of their registered marks in the absence of actual use in the Philippines.  While petitioners enjoy the statutory presumptions arising from such registration of their trademarks i.e., as to the validity of the registration, ownership and the exclusive right to use the registered marks, they may not successfully sue on the basis alone of their respective certificates of registration of trademarks. Petitioners are still foreign corporations. As such, they ought, as a condition to availment of the rights and privileges vis-à-vis their trademarks in this country, to show proof that, on top of Philippine registration, their country grants substantially similar rights and privileges to Filipino citizens pursuant to Section 21-A of R.A. No. 166.  Although it is true that the Philippines is obligated to honor and enforce the provisions of the Paris Convention being a signatory thereto, any protection accorded has to be made subject to the limitations of Philippine laws. Foreign nationals must still observe and comply with the conditions imposed by Philippine law on its nationals. Considering that R.A. No. 166, as amended, specifically Sections 2 and 2-A thereof, mandates actual use of the marks and/or emblems in local commerce and trade before they may be registered and ownership thereof acquired, the petitioners cannot, therefore, dispense with the element of actual use. Their being nationals of member-countries of the Paris Union does not alter the legal situation.  In other words, (a foreign corporation) may have the capacity to sue for infringement but the question of whether they have an exclusive right over their symbol as to justify issuance of the controversial writ will depend on actual use of their trademarks in the Philippines in line with Sections 2 and 2-A of the same law. It is thus incongruous for petitioners to claim that when a foreign corporation not licensed to do business in the Philippines files a complaint

76

Page 77: IP Developed Reviewer

for infringement, the entity need not be actually using its trademark in commerce in the Philippines. Such a foreign corporation may have the personality to file a suit for infringement but it may not necessarily be entitled to protection due to absence of actual use of the emblem in the local market. Registration of a trademark unaccompanied by actual use thereof in the country accords the registrant only the standing to sue for infringement in Philippine courts. Entitlement to protection of such trademark in the country is entirely a different matter. What does “ordinary purchaser” mean?

"Ordinarily intelligent buyer" considering the type of product involved. "ordinary purchaser" in this case means "one accustomed to buy, and therefore to some extent familiar with, the goods in question." Products involved are addicting and are purchased mainly by those who are already predisposed to a certain brand. Accordingly, the ordinary buyer thereof would be all too familiar with his brand and discriminating as well

Sufficient to allege that party is suing under Sec. 21-A of Paris ConventionCourt held that where the complainant is a national of a Paris Convention- adhering country,

its allegation that it is suing under said Section 21-A would suffice, because the reciprocal agreement between the two countries is embodied and supplied by the Paris Convention which, being considered part of Philippine municipal laws, can be taken judicial notice of in infringement suits.

Note: Unlike in the Converse case, although also not doing business in the Philippines, where evidence was presented that trademark is being used in commerce.

D. The Trademark Registration Process (Sec. 132 – 136)Filing of Application – accorded filing dateExamination – searches for other similar trademarksPublication – may oppose within 30 days (if opposed, Inter Partes Cases)Issuance

Section 132. Application Number and Filing Date. – 132.1. The Office shall examine whether the application satisfies the requirements for the grant

of a filing date as provided in Section 127 and Regulations relating thereto. If the application does not satisfy the filing requirements, the Office shall notify the applicant who shall within a period fixed by the Regulations complete or correct the application as required, otherwise, the application shall be considered withdrawn.

132.2 Once an application meets the filing requirements of Section 127, it shall be numbered in the sequential order, and the applicant shall be informed of the application number and the filing date of the application will be deemed to have been abandoned. (n)

Section 133. Examination and Publication. – 133.1. Once the application meets the filing requirements of Section 127, the Office shall

examine whether the application meets the requirements of Section 124 and the mark as defined in Section 121 is registrable under Section 123.

133.2. Where the Office finds that the conditions referred to in Subsection 133.1 are fulfilled, it shall upon payment of the prescribed fee, forthwith cause the application, as filed, to be published in the prescribed manner.

133.3. If after the examination, the applicant is not entitled to registration for any reason, the Office shall advise the applicant thereof and the reasons therefor. The applicant shall have a period of 4 months in which to reply or amend his application, which shall then be re-

77

Page 78: IP Developed Reviewer

examined. The Regulations shall determine the procedure for the re-examination or revival of an application as well as the appeal to the Director of Trademarks from any final action by the Examiner.

133.4. An abandoned application may be revived as a pending application within 3 months from the date of abandonment, upon good cause shown and the payment of the required fee.

133.5. The final decision of refusal of the Director of Trademarks shall be appealable to the Director General in accordance with the procedure fixed by the Regulations. (Sec. 7, R.A. No. 166a)

Section 134. Opposition. - Any person who believes that he would be damaged by the registration of a mark may, upon payment of the required fee and within 30 days after the publication referred to in Subsection 133.2, file with the Office an opposition to the application. Such opposition shall be in writing and verified by the oppositor or by any person on his behalf who knows the facts, and shall specify the grounds on which it is based and include a statement of the facts relied upon. Copies of certificates of registration of marks registered in other countries or other supporting documents mentioned in the opposition shall be filed therewith, together with the translation in English, if not in the English language. For good cause shown and upon payment of the required surcharge, the time for filing an opposition may be extended by the Director of Legal Affairs, who shall notify the applicant of such extension. The Regulations shall fix the maximum period of time within which to file the opposition. (Sec. 8, R.A. No. 165a)

Section 135. Notice and Hearing. - Upon the filing of an opposition, the Office shall serve notice of the filing on the applicant, and of the date of the hearing thereof upon the applicant and the oppositor and all other persons having any right, title or interest in the mark covered by the application, as appear of record in the Office. (Sec. 9, R.A. No. 165)

Section 136. Issuance and Publication of Certificate. - When the period for filing the opposition has expired, or when the Director of Legal Affairs shall have denied the opposition, the Office upon payment of the required fee, shall issue the certificate of registration. Upon issuance of a certificate of registration, notice thereof making reference to the publication of the application shall be published in the IPO Gazette. (Sec. 10, R.A. No. 165)

E. Rights Conferred After Registration and Maintenance of Rights (Sec. 138, 145, 146, 147, 148, 149, 150, 151)Section 138. (memorize) Certificates of Registration. - A certificate of registration of a mark

shall be prima facie evidence of the 1 validity of the registration , the 2registrant's ownership of the mark, and of the 3registrant's exclusive right to use the same in connection with the goods or services and those that are related thereto specified in the certificate. (Sec. 20, R.A. No. 165)

Section 145. Duration. - A certificate of registration shall remain in force for 10 years: Provided, That the registrant shall file a declaration of actual use and evidence to that effect, or shall show valid reasons based on the existence of obstacles to such use, as prescribed by the Regulations, within 1 year from the fifth anniversary of the date of the registration of the mark. Otherwise, the mark shall be removed from the Register by the Office. (Sec. 12, R.A. No. 166a)

Section 146. Renewal. – 146.1. A certificate of registration may be renewed for periods of 10 years at its expiration upon

payment of the prescribed fee and upon filing of a request. The request shall contain the

78

Page 79: IP Developed Reviewer

following indications:(a) An indication that renewal is sought;(b) The name and address of the registrant or his successor-in-interest, hereafter referred

to as the "right holder";(c) The registration number of the registration concerned;(d) The filing date of the application, which resulted in the registration concerned to be

renewed;(e) Where the right holder has a representative, the name and address of that

representative;(f) The names of the recorded goods or services for which the renewal is requested or the

names of the recorded goods or services for which the renewal is not requested, grouped according to the classes of the Nice Classification to which that group of goods or services belongs and presented in the order of the classes of the said Classification; and

(g) A signature by the right holder or his representative.146.2. Such request shall be in Filipino or English and may be made at any time within 6

months before the expiration of the period for which the registration was issued or renewed, or it may be made within 6 months after such expiration on payment of the additional fee herein prescribed.

146.3. If the Office refuses to renew the registration, it shall notify the registrant of his refusal and the reasons therefor.

146.4. An applicant for renewal not domiciled in the Philippines shall be subject to and comply with the requirements of this Act. (Sec. 15, R.A. No. 166a)

Section 147. (memorize) Rights Conferred. – 147.1. The owner of a registered mark shall have the exclusive right to prevent all third parties

not having the owner's consent from using in the course of trade identical or similar signs or containers for goods or services which are identical or similar to those in respect of which the trademark is registered where such use would result in a likelihood of confusion. In case of the use of an identical sign for identical goods or services, a likelihood of confusion shall be presumed.

147.2. The exclusive right of the owner of a well-known mark defined in Subsection 123.1(e) which is registered in the Philippines, shall extend to goods and services which are not similar to those in respect of which the mark is registered: Provided, That use of that mark in relation to those goods or services would indicate a connection between those goods or services and the owner of the registered mark: Provided further, That the interests of the owner of the registered mark are likely to be damaged by such use. (n)

Section 148. Use of Indications by Third Parties for Purposes Other than those for which the Mark is Used. - Registration of the mark shall not confer on the registered owner the right to preclude third parties from using bona fide their names, addresses, pseudonyms, a geographical name, or exact indications concerning the kind, quality, quantity, destination, value, place of origin, or time of production or of supply, of their goods or services: Provided, That such use is confined to the purposes of mere identification or information and cannot mislead the public as to the source of the goods or services. (n)

Section 149. Assignment and Transfer of Application and Registration. – 149.1. An application for registration of a mark, or its registration, may be assigned or

transferred with or without the transfer of the business using the mark. (n)149.2. Such assignment or transfer shall, however, be null and void if it is liable to mislead the

79

Page 80: IP Developed Reviewer

public, particularly as regards the nature, source, manufacturing process, characteristics, or suitability for their purpose, of the goods or services to which the mark is applied.

149.3. The assignment of the application for registration of a mark, or of its registration, shall be in writing and require the signatures of the contracting parties. Transfers by mergers or other forms of succession may be made by any document supporting such transfer.

149.4. Assignments and transfers of registrations of marks shall be recorded at the Office on payment of the prescribed fee; assignment and transfers of applications for registration shall, on payment of the same fee, be provisionally recorded, and the mark, when registered, shall be in the name of the assignee or transferee.

149.5. Assignments and transfers shall have no effect against third parties until they are recorded at the Office. (Sec. 31, R.A. No. 166a)

Section 150. License Contracts. – 150.1. Any license contract concerning the registration of a mark, or an application therefor,

shall provide for effective control by the licensor of the quality of the goods or services of the licensee in connection with which the mark is used. If the license contract does not provide for such quality control, or if such quality control is not effectively carried out, the license contract shall not be valid.

150.2. A license contract shall be submitted to the Office which shall keep its contents confidential but shall record it and publish a reference thereto. A license contract shall have no effect against third parties until such recording is effected. The Regulations shall fix the procedure for the recording of the license contract. (n)

Section 151. Cancellation. – 151.1. A petition to cancel a registration of a mark under this Act may be filed with the Bureau

of Legal Affairs by any person who believes that he is or will be damaged by the registration of a mark under this Act as follows:(a) Within 5 years from the date of the registration of the mark under this Act.(b) At any time, if the registered mark 1 becomes the generic name for the goods or

services, or a portion thereof, for which it is registered, or has 2 been abandoned , or its 3 registration was obtained fraudulently or contrary to the provisions of this Act , or if the 4 registered mark is being used by, or with the permission of, the registrant so as to misrepresent the source of the goods or services on or in connection with which the mark is used. If the registered mark becomes the generic name for less than all of the goods or services for which it is registered, a petition to cancel the registration for only those goods or services may be filed. A registered mark shall not be deemed to be the generic name of goods or services solely because such mark is also used as a name of or to identify a unique product or service. The primary significance of the registered mark to the relevant public rather than purchaser motivation shall be the test for determining whether the registered mark has become the generic name of goods or services on or in connection with which it has been used. (n)

(c) At any time, if the registered owner of the mark without legitimate reason fails to use the mark within the Philippines, or to cause it to be used in the Philippines by virtue of a license during an uninterrupted period of 3 years or longer.

151.2. (memorize) Notwithstanding the foregoing provisions, the court or the administrative agency vested with jurisdiction to hear and adjudicate any action to enforce the rights to a registered mark shall likewise exercise jurisdiction to determine whether the registration of said mark may be cancelled in accordance with this Act. The filing of a suit to enforce the registered mark with the proper court or agency shall exclude any other court or

80

Page 81: IP Developed Reviewer

agency from assuming jurisdiction over a subsequently filed petition to cancel the same mark. On the other hand, the earlier filing of petition to cancel the mark with the Bureau of Legal Affairs shall not constitute a prejudicial question that must be resolved before an action to enforce the rights to same registered mark may be decided. (Sec. 17, R.A. No. 166a) (gives life to the presumptions of validity of certificate)

F. What marks cannot be registered? Sec. 123Section 123. Registrability. – 123.1. A mark cannot be registered if it:

(a) Consists of immoral, deceptive or scandalous matter, or matter which may disparage or falsely suggest a connection with persons, living or dead, institutions, beliefs, or national symbols, or bring them into contempt or disrepute;

(b) Consists of the flag or coat of arms or other insignia of the Philippines or any of its political subdivisions, or of any foreign nation, or any simulation thereof;

(c) Consists of a name, portrait or signature identifying a particular living individual except by his written consent, or the name, signature, or portrait of a deceased President of the Philippines, during the life of his widow, if any, except by written consent of the widow;

(d) Is identical with a registered mark belonging to a different proprietor or a mark with an earlier filing or priority date, in respect of:

(i) The same goods or services, or(ii) Closely related goods or services, or(iii) If it nearly resembles such a mark as to be likely to deceive or cause confusion;

(e) Is identical with, or confusingly similar to, or constitutes a translation of a mark which is considered by the competent authority of the Philippines to be well-known internationally and in the Philippines, whether or not it is registered here , as being already the mark of a person other than the applicant for registration, and used for identical or similar goods or services: Provided, That in determining whether a mark is well-known, account shall be taken of the knowledge of the relevant sector of the public, rather than of the public at large, including knowledge in the Philippines which has been obtained as a result of the promotion of the mark;

(f) Is identical with, or confusingly similar to, or constitutes a translation of a mark considered well-known in accordance with the preceding paragraph, which is registered in the Philippines with respect to goods or services which are not similar to those with respect to which registration is applied for: Provided, That use of the mark in relation to those goods or services would indicate a connection between those goods or services, and the owner of the registered mark: Provided further, That the interests of the owner of the registered mark are likely to be damaged by such use;

(g) Is likely to mislead the public, particularly as to the nature, quality, characteristics or geographical origin of the goods or services;

(h) Consists exclusively of signs that are generic for the goods or services that they seek to identify;

(i) Consists exclusively of signs or of indications that have become customary or usual to designate the goods or services in everyday language or in bona fide and established trade practice;

(j) Consists exclusively of signs or of indications that may serve in trade to designate the kind, quality, quantity, intended purpose, value, geographical origin, time or production of the goods or rendering of the services, or other characteristics of the goods or

81

Page 82: IP Developed Reviewer

services;(k) Consists of shapes that may be necessitated by technical factors or by the nature of

the goods themselves or factors that affect their intrinsic value;(l) Consists of color alone, unless defined by a given form; or(m) Is contrary to public order or morality.

123.2. As regards signs or devices mentioned in paragraphs (j), (k), and (l), nothing shall prevent the registration of any such sign or device which has become distinctive in relation to the goods for which registration is requested as a result of the use that have been made of it in commerce in the Philippines. The Office may accept as prima facie evidence that the mark has become distinctive, as used in connection with the applicant's goods or services in commerce, proof of substantially exclusive and continuous use thereof by the applicant in commerce in the Philippines for 5 years before the date on which the claim of distinctiveness is made.

123.3. The nature of the goods to which the mark is applied will not constitute an obstacle to registration. (Sec. 4, R.A. No. 166a)

G. Protection of well-known marks (Sec. 123 (e) and (f), 131.3, Rule 102, Implementing Rules and Regulations for Trademarks)

Section 123. Registrability. – 123.1. A mark cannot be registered if it:(e) Is identical with, or confusingly similar to, or constitutes a translation of a mark which is

considered by the competent authority of the Philippines to be well-known internationally and in the Philippines, whether or not it is registered here, as being already the mark of a person other than the applicant for registration, and used for identical or similar goods or services: Provided, That in determining whether a mark is well-known, account shall be taken of the knowledge of the relevant sector of the public, rather than of the public at large, including knowledge in the Philippines which has been obtained as a result of the promotion of the mark;

(f) Is identical with, or confusingly similar to, or constitutes a translation of a mark considered well-known in accordance with the preceding paragraph, which is registered in the Philippines with respect to goods or services which are not similar to those with respect to which registration is applied for: Provided, That use of the mark in relation to those goods or services would indicate a connection between those goods or services, and the owner of the registered mark: Provided further, That the interests of the owner of the registered mark are likely to be damaged by such use;

Section 131. Priority Right. – 131.1. An application for registration of a mark filed in the Philippines by a person referred to in

Section 3, and who previously duly filed an application for registration of the same mark in one of those countries, shall be considered as filed as of the day the application was first filed in the foreign country.

131.2. No registration of a mark in the Philippines by a person described in this section shall be granted until such mark has been registered in the country of origin of the applicant.

131.3. Nothing in this section shall entitle the owner of a registration granted under this section to sue for acts committed prior to the date on which his mark was registered in this country: Provided, That, notwithstanding the foregoing, the owner of a well-known mark as defined in Section 123.1(e) of this Act, that is not registered in the Philippines, may, against an

82

Page 83: IP Developed Reviewer

identical or confusingly similar mark, oppose its registration, or petition the cancellation of its registration or sue for unfair competition, without prejudice to availing himself of other remedies provided for under the law.

131.4. In like manner and subject to the same conditions and requirements, the right provided in this section may be based upon a subsequent regularly filed application in the same foreign country: Provided, That any foreign application filed prior to such subsequent application has been withdrawn, abandoned, or otherwise disposed of, without having been laid open to public inspection and without leaving any rights outstanding, and has not served, nor thereafter shall serve, as a basis for claiming a right of priority. (Sec. 37, R.A. No. 166a)

(see below)

Mirpuri v. CA, G.R. No. 114508, Nov. 19, 1999Puma v. IAC, G.R. No. 75067 Feb. 26, 1988Sehwani, Inc. v. In-N-Out Burger, Inc. (G.R. No. 171053, October 15, 2007),

Recent Development in Protection of Well-Known Marks in the Philippines

January 16, 2008by: R.G. Pasiliao

In determining a mark as well-known, the fact that the mark is neither registered nor used in the Philippines is of no consequence. This was the ruling of the Supreme Court in Sehwani, Inc. v. In-N-Out Burger, Inc. (G.R. No. 171053, October 15, 2007), where American fast food chain In-N-Out Burger, Inc., was granted exclusive rights over the use of the IN-N-OUT and IN-N-OUT BURGER marks.

The ruling is a marked departure from the doctrines enunciated in Kabushi Kaisha Isetan v. IAC (G.R. No. 75420, November 15, 1991) and Philip Morris v. Court of Appeals (G.R. No. 91332, July 16, 1993), where actual use in commerce in the Philippines was held to be an indispensable element for the acquisition of ownership over a trademark or a tradename.

Under the old Trademark Law, which was the governing law when the controversies in Kabushi Kaisha and Philip Morris took place, actual commercial use in the Philippines was a pre-requisite to the acquisition of ownership over a trademark or a tradename. When the controversy in Sehwani arose, however, the old Trademark Law was no longer in effect. Thus, the Supreme Court cited the rules implementing the Intellectual Property Code, which provide guidelines in determining whether a mark is well-known.

Sehwani also made use of the Joint Recommendation Concerning Provisions on the Protection of Well-Known Marks, adopted by the Assembly of the Paris Union for the Protection of Industrial Property and the General Assembly of the World Intellectual Property Organization (WIPO) in 1999. Rule 102 of the Rules and Regulations on Trademarks, Service Marks, Tradenames and Marked or Stamped Containers, states the criteria for determining whether a mark is well-known. The following criteria or any combination thereof may be taken into account:

(a) the duration, extent and geographical area of any use of the mark, in particular, the duration, extent

83

Page 84: IP Developed Reviewer

and geographical area of any promotion of the mark, including advertising or publicity and the presentation, at fairs or exhibitions, of the goods and/or services to which the mark applies;

(b) the market share, in the Philippines and in other countries, of the goods and/or services to which the mark applies;

(c) the degree of the inherent or acquired distinction of the mark; (d) the quality-image or reputation acquired by the mark; (e) the extent to which the mark has been registered in the world; (f) the exclusivity of registration attained by the mark in the world; (g) the extent to which the mark has been used in the world; (h) the exclusivity of use attained by the mark in the world; (i) the commercial value attributed to the mark in the world; (j) the record of successful protection of the rights in the mark; (k) the outcome of litigations dealing with the issue of whether the mark is a well-known mark; and, (l) the presence or absence of identical or similar marks validly registered for or used on identical or

similar goods or services and owned by persons other than the person claiming that his mark is a well-known mark.

In Sehwani, In-N-Out Burger presented its United States trademark registrations before the Intellectual Property Office as evidence that its mark was well-known. It submitted U.S. Trademark Registration No. 1,514,689 for “IN-N-OUT Burger and Arrow Design” under class 25; U.S. Trademark Registration No. 1,528,456 for “IN-N-OUT Burger and Arrow Design” under Classes 29, 30, 32 and 42; U.S. Trademark Registration No. 1,101,638 for “IN-N-OUT” under Class No. 30; and US Trademark Registration No. 1,085,163 “IN-N-OUT” under Class 42. For the mark “Double-Double”, In-N-Out submitted Certificates of Registration of the mark in several countries.

The Supreme Court took note of these various trademark registrations, observing that under Section (e) Rule 102 of the Implementing Rules, the extent to which a mark has been registered in the world is taken into account in determining whether a mark is indeed well known.

In-N-Out Burger likewise submitted evidence consisting of articles about “IN-N-OUT Burger” appearing in magazines and newspapers, and a print-out of what appeared to be printed representations of its website. In-N-Out also presented videotapes of famous celebrities mentioning IN-N-OUT burgers in the course of their interviews.

Citing Sections (a) and (d) of Rule 102, the Court held that these pieces of evidence would tend to show that the mark was well-known and that the quality, image, and reputation acquired by the IN-N-OUT mark was unmistakable. Thus, on the basis of “registrations in various countries around the world and its comprehensive advertisements therein,” In-N-Out Burger was declared a well-known mark.

The Court observed that the 1999 Joint Recommendation Concerning Provisions on the Protection of Well-Known Marks expanded the scope of protection initially afforded by Article 6bis of the Paris Convention. The Court cited Part I, Article 2(3) of the Joint Recommendation, which enumerates three factors to be eliminated in determining whether or not a mark is well-known. Thus, it shall NOT be required:

a) that the mark has been used in, or that the mark has been registered or that an application for

84

Page 85: IP Developed Reviewer

registration of the mark has been filed in or in respect of, the Member State;

b) that the mark is well known in, or that the mark has been registered or that an application for registration of the mark has been filed in or in respect of, any jurisdiction other than the Member State; or

c) that the mark is well known by the public at large in the Member State.

Accordingly, the Court declared that “the fact that respondent’s marks are neither registered nor used in the Philippines is of no moment.”

Conclusion

In the Philip Morris case, Justice Feliciano wrote a dissenting opinion discussing the advertisement function of a trademark. He stated:

“The third or advertisement function of trademark has become of special importance given the modern technology of communication and transportation and the growth of international trade. Through advertisement in the broadcast and print media, the owner of the trademark is able to establish a nexus between its trademarked products and the public in regions where the owner does not itself manufacture or sell its own products. Through advertisement, a well-established and well-earned reputation may be gained in countries where the trademark owner has itself not established business connection.”

The ruling of the Supreme Court in Sehwani coincides with Justice Feliciano’s opinion. By not requiring actual use in commerce and by giving substantial weight to evidence consisting of magazine and newspaper articles, a print-out of an internet website, and videotapes of celebrities, the High Court now clearly recognizes advertisements in establishing the fame and reputation of a mark.

The Court gives full force and effect to the Intellectual Property Code and to international covenants giving protection to well-known trademarks. The ruling also recognizes the role of modern information and communications technology in raising awareness of trademark ownership.

H. Doctrine of secondary meaning (Sec. 123.2) – refers to the acquired distinctiveness of a mark over time and continued use; only those under (j), (k), (l) can acquire secondary meaning

123.2. As regards signs or devices mentioned in paragraphs (j), (k), and (l), nothing shall prevent the registration of any such sign or device which has become distinctive in relation to the goods for which registration is requested as a result of the use that have been made of it in commerce in the Philippines. The Office may accept as prima facie evidence that the mark has become distinctive, as used in connection with the applicant's goods or services in commerce, proof of substantially exclusive and continuous use thereof by the applicant in commerce in the Philippines for 5 years before the date on which the claim of distinctiveness is made.

123.3. The nature of the goods to which the mark is applied will not constitute an obstacle to registration. (Sec. 4, R.A. No. 166a)

85

Page 86: IP Developed Reviewer

(no need to read cases)Arce Sons & Co. v. Selecta Biscuit Co., G.R. No. L-14761 & 17981, January 28, 1961Ang v. Teodoro, G.R. No. 48226, Dec. 14, 1942Etepha v. Dir. Of Patents, G.R. No. L-20635, March 31, 1966

I. TM Infringement and Unfair Competition – dominancy test; holistic test (Sec. 155, 156, 158, 159, 168, 170)

Section 155. Remedies; Infringement. - Any person who shall, without the consent of the owner of the registered mark:

155.1. Use in commerce any reproduction, counterfeit, copy, or colorable imitation of a registered mark or the same container or a dominant feature thereof in connection with the sale, offering for sale, distribution, advertising of any goods or services including other preparatory steps necessary to carry out the sale of any goods or services on or in connection with which such use is likely to cause confusion, or to cause mistake, or to deceive; or

155.2. Reproduce, counterfeit, copy or colorably imitate a registered mark or a dominant feature thereof and apply such reproduction, counterfeit, copy or colorable imitation to labels, signs, prints, packages, wrappers, receptacles or advertisements intended to be used in commerce upon or in connection with the sale, offering for sale, distribution, or advertising of goods or services on or in connection with which such use is likely to cause confusion, or to cause mistake, or to deceive, shall be liable in a civil action for infringement by the registrant for the remedies hereinafter set forth: Provided, That the infringement takes place at the moment any of the acts stated in Subsection 155.1 or this subsection are committed regardless of whether there is actual sale of goods or services using the infringing material. (Sec. 22, R.A. No 166a) (even if not used in commerce,,infringement already, as opposed to 155.1)

Section 156. Actions, and Damages and Injunction for Infringement. – 156.1. The owner of a registered mark may recover damages from any person who infringes

his rights, and the measure of the damages suffered shall be either the reasonable profit which the complaining party would have made, had the defendant not infringed his rights, or the profit which the defendant actually made out of the infringement, or in the event such measure of damages cannot be readily ascertained with reasonable certainty, then the court may award as damages a reasonable percentage based upon the amount of gross sales of the defendant or the value of the services in connection with which the mark or trade name was used in the infringement of the rights of the complaining party. (Sec. 23, first par., R.A. No. 166a)

156.2. On application of the complainant, the court may impound during the pendency of the action, sales invoices and other documents evidencing sales. (n)

156.3. In cases where actual intent to mislead the public or to defraud the complainant is shown, in the discretion of the court, the damages may be doubled. (Sec. 23, first par., R.A. No. 166)

156.4. The complainant, upon proper showing, may also be granted injunction. (Sec. 23, second par., R.A. No. 166a)

Section 158. Damages; Requirement of Notice. - In any suit for infringement, the owner of the registered mark shall not be entitled to recover profits or damages unless the acts have been committed with knowledge that such imitation is likely to cause confusion, or to cause mistake, or to deceive. Such knowledge is presumed if the registrant gives notice

86

Page 87: IP Developed Reviewer

that his mark is registered by displaying with the mark the words '"Registered Mark" or the letter R within a circle or if the defendant had otherwise actual notice of the registration. (Sec. 21, R.A. No. 166a)

Section 159. Limitations to Actions for Infringement. - Notwithstanding any other provision of this Act, the remedies given to the owner of a right infringed under this Act shall be limited as follows:

159.1. Notwithstanding the provisions of Section 155 hereof, a registered mark shall have no effect against any person who, in good faith, before the filing date or the priority date, was using the mark for the purposes of his business or enterprise: Provided, That his right may only be transferred or assigned together with his enterprise or business or with that part of his enterprise or business in which the mark is used.

159.2. Where an infringer who is engaged solely in the business of printing the mark or other infringing materials for others is an innocent infringer, the owner of the right infringed shall be entitled as against such infringer only to an injunction against future printing.

159.3. Where the infringement complained of is contained in or is part of paid advertisement in a newspaper, magazine, or other similar periodical or in an electronic communication, the remedies of the owner of the right infringed as against the publisher or distributor of such newspaper, magazine, or other similar periodical or electronic communication shall be limited to an injunction against the presentation of such advertising matter in future issues of such newspapers, magazines, or other similar periodicals or in future transmissions of such electronic communications. The limitations of this subparagraph shall apply only to innocent infringers: Provided, That such injunctive relief shall not be available to the owner of the right infringed with respect to an issue of a newspaper, magazine, or other similar periodical or an electronic communication containing infringing matter where restraining the dissemination of such infringing matter in any particular issue of such periodical or in an electronic communication would delay the delivery of such issue or transmission of such electronic communication is customarily conducted in accordance with the sound business practice, and not due to any method or device adopted to evade this section or to prevent or delay the issuance of an injunction or restraining order with respect to such infringing matter. (n)

foreign corporations also protected even if they are not doing business in the Philippines- as long as there is reciprocity

Section 168. Unfair Competition, Rights, Regulation and Remedies. – 168.1. A person who has identified in the mind of the public the goods he manufactures or

deals in, his business or services from those of others, whether or not a registered mark is employed, has a property right in the goodwill of the said goods, business or services so identified, which will be protected in the same manner as other property rights.

168.2. Any person who shall employ deception or any other means contrary to good faith by which he shall pass off the goods manufactured by him or in which he deals, or his business, or services for those of the one having established such goodwill, or who shall commit any acts calculated to produce said result, shall be guilty of unfair competition, and shall be subject to an action therefor. (plaintiff must prove that his trademark has obtained goodwill)

168.3. In particular, and without in any way limiting the scope of protection against unfair competition, the following shall be deemed guilty of unfair competition:

87

Page 88: IP Developed Reviewer

(a) Any person, who is selling his goods and gives them the general appearance of goods of another manufacturer or dealer, either as to the goods themselves or in the wrapping of the packages in which they are contained, or the devices or words thereon, or in any other feature of their appearance, which would be likely to influence purchasers to believe that the goods offered are those of a manufacturer or dealer, other than the actual manufacturer or dealer, or who otherwise clothes the goods with such appearance as shall deceive the public and defraud another of his legitimate trade, or any subsequent vendor of such goods or any agent of any vendor engaged in selling such goods with a like purpose;

(b) Any person who by any artifice, or device, or who employs any other means calculated to induce the false belief that such person is offering the services of another who has identified such services in the mind of the public; or

(c) Any person who shall make any false statement in the course of trade or who shall commit any other act contrary to good faith of a nature calculated to discredit the goods, business or services of another.

168.4. The remedies provided by Sections 156, 157 and 161 shall apply mutatis mutandis. (Sec. 29, R.A. No. 166a)

Section 170. Penalties. - Independent of the civil and administrative sanctions imposed by law, a criminal penalty of imprisonment from 2 years to 5 years and a fine ranging from P50,000 to P200,000, shall be imposed on any person who is found guilty of committing any of the acts mentioned in Section 155, Section 168 and Subsection 169.1. (Arts. 188 and 189, Revised Penal Code) (admin, civil & criminal)

Notes:Extraneous Factors to Consider:

1) profile of purchaser: age, training, education2) immediate consumption?3) luxury or ordinary household products4) conditions under which product is purchased

WON products are competing is a factual finding by the court. (Gallo case)

Del Monte v. CA, G.R. No. 78325, January 25, 1990Facts: Petitioner Del Monte Corporation is a foreign company organized under the laws of the US and not

engaged in business in the Philippines. It granted petitioner Philpack (domestic corp) the right to manufacture, distribute and sell in the Philippines various agricultural products, including catsup, under the Del Monte trademark and logo.

On October 27,1965, Del Monte authorized Philpack to register with the Philippine Patent Office the Del Monte catsup bottle configuration, for which it was granted a Certificate of Trademark Registration number by the PPO under the Supplemental Register.

On November 20, 1972, Del Monte also obtained two registration certificates for its trademark "DEL MONTE" and its logo.

Respondent Sunshine Sauce Manufacturing Industries was issued a Certificate of Registration by the Bureau of Domestic Trade on April 17,1980, to engage in the manufacture, packing, distribution and sale of various kinds of sauce, identified by the logo Sunshine Fruit Catsup. This logo was registered in the Supplemental Register on September 20, 1983. The product itself was contained

88

Page 89: IP Developed Reviewer

in various kinds of bottles, including the Del Monte bottle, which the private respondent bought from the junk shops for recycling.

Having received reports that PR was using its exclusively designed bottles and a logo confusingly similar to Del Monte's, Philpack warned it to desist from doing so on pain of legal action.

Claiming that the demand had been ignored, Philpack and Del Monte filed a complaint against the PR for infringement of trademark and unfair competition, with a prayer for damages and the issuance of a writ of preliminary injunction.

In its answer, Sunshine alleged that it had long ceased to use the Del Monte bottle and that its logo was substantially different from the Del Monte logo and would not confuse the buying public to the detriment of the petitioners.

RTC dismissed the complaint. There were substantial differences between the logos or trademarks of the parties and that even if PR used petitioners’ bottles, PR already became the owner thereof upon its purchase from the junk shops. Furthermore, petitioners had failed to establish the defendant's malice or bad faith, which was an essential element of infringement of trademark or unfair competition.

CA affirmed RTC ruling. Petitioners filed present petition for certiorari under Rule 45.

Section 22 of R.A. No. 166, otherwise known as the Trademark Law, provides in part as follows:Sec. 22. Infringement, what constitutes. Any person who shall use, without the consent of the registrant, any reproduction, counterfeit, copy or colorable imitation of any registered mark or trade-name in connection with the sale, offering for sale, or advertising of any goods, business or services on or in connection with which such use is likely to cause confusion or mistake or to deceive purchasers or others as to the source or origin of such goods or services or identity of such business; or reproduce, counterfeit copy or colorably imitate any such mark or trade name and apply such reproduction, counterfeit copy or colorable imitation to labels, signs, prints, packages, wrappers, receptacles or advertisements intended to be used upon or in connection with such goods, business or services, shall be liable to a civil action by the registrant for any or all of the remedies herein provided.

Sec. 29 of the same law states as follows:Sec. 29. Unfair competition, rights and remedies. A person who has identified in the mind of the public the goods he manufactures or deals in, his business or services from those of others, whether or not a mark or tradename is employed, has a property right in the goodwill of the said goods, business or services so identified, which will be protected in the same manner as other property rights. Such a person shall have the remedies provided in section twenty- three, Chapter V hereof.

Any person who shall employ deception or any other means contrary to good faith by which he shall pass off the goods manufactured by him or in which he deals, or his business, or services for those of the one having established such goodwill, or who shall commit any acts calculated to produce said result, shall be guilty of unfair competition, and shall be subject to an action therefor.

In particular, and without in any way limiting the scope of unfair competition, the following shall be deemed guilty of unfair competition:

(a) Any person, who in selling his goods shall give them the general appearance of goods of another manufacturer or dealer, either as to the goods themselves or in the wrapping of the packages in

89

Page 90: IP Developed Reviewer

which they are contained, or the devices or words thereon, or in any other feature of their appearance, which would likely influence purchasers to believe that the goods offered are those of a manufacturer or dealer other than the actual manufacturer or dealer, or who otherwise clothes the goods with such appearance as shall deceive the public and defraud another of his legitimate trade, or any subsequent vendor of such goods or any agent of any vendor engaged in selling such goods with a like purpose;

(b) Any person who by any artifice, or device, or who employs ally other means calculated to induce the false belief that such person is offering the services of another who has identified such services in the mind of the public; or

(c) Any person who shall make any false statement in the course of trade or who shall commit any other act contrary to good faith of a nature calculated to discredit the goods, business or services of another.

To arrive at a proper resolution of this case, it is important to bear in mind the following distinctions between infringement of trademark and unfair competition.

Infringement of Trademark Unfair Competition

Unauthorized use of a trademark Passing off of one’s goods as those of another

Intent is unnecessary. Fraudulent intent is essential.

Prior registration of the trademark is a prerequisite to the action.

Registration is not necessary.

CA in ruling that there was no colorable imitation of petitioners’ trademark and logo used the holistic test:

In determining whether two trademarks are confusingly similar, the two marks in their entirety as they appear in the respective labels must be considered in relation to the goods to which they are attached; the discerning eye of the observer must focus not only on the predominant words but also on the other features appearing on both labels. (memorize)

Issues/Held: 1) Whether or not there was trademark/logo infringement. YES (holistic test was used)

A number of courts have held that to determine whether a trademark has been infringed, we must consider the mark as a whole and not as dissected. If the buyer is deceived, it is attributable to the marks as a totality, not usually to any part of it. The court therefore should be guided by its first impression, for a buyer acts quickly and is governed by a casual glance, the value of which may be dissipated as soon as the court assumes to analyze carefully the respective features of the mark.

Expensive and valuable items are normally bought only after deliberate, comparative and analytical investigation. But mass products, low priced articles in wide use, and matters of everyday purchase requiring frequent replacement are bought by the casual consumer without great care. In this latter category is catsup.

At that, even if the labels were analyzed together it is not difficult to see that the Sunshine label is a colorable imitation of the Del Monte trademark. The predominant colors used in the Del Monte

90

Page 91: IP Developed Reviewer

label are green and red-orange, the same with Sunshine. The word "catsup" in both bottles is printed in white and the style of the print/letter is the same. Although the logo of Sunshine is not a tomato, the figure nevertheless approximates that of a tomato.

The person who infringes a trade mark does not normally copy out but only makes colorable changes, employing enough points of similarity to confuse the public with enough points of differences to confuse the courts. What is undeniable is the fact that when a manufacturer prepares to package his product, he has before him a boundless choice of words, phrases, colors and symbols sufficient to distinguish his product from the others. When as in this case, Sunshine chose, without a reasonable explanation, to use the same colors and letters as those used by Del Monte though the field of its selection was so broad, the inevitable conclusion is that it was done deliberately to deceive.

2. Whether or not PR is guilty of unfair competition for having used the Del Monte bottle. YESThe reason is that the configuration of the said bottle was merely registered in the

Supplemental Register. As Sunshine's label is an infringement of the Del Monte's trademark, law and equity call for the

cancellation of the private respondent's registration and withdrawal of all its products bearing the questioned label from the market. With regard to the use of Del Monte's bottle, the same constitutes unfair competition; hence, the respondent should be permanently enjoined from the use of such bottles. Emerald Garments v. CA, G.R. No. 100098, December 29, 1995Facts: On 18 September 1981, private respondent H.D. Lee Co., Inc., a foreign corporation organized

under the laws of Delaware, U.S.A., filed with the Bureau of Patents, Trademarks & Technology Transfer (BPTTT) a Petition for Cancellation of Registration No. SR 5054 (Supplemental Register) for the trademark "STYLISTIC MR. LEE" used on skirts, jeans, blouses, socks, briefs, jackets, jogging suits, dresses, shorts, shirts and lingerie under Class 25, issued on 27 October 1980 in the name of petitioner Emerald Garment Manufacturing Corporation, a domestic corporation organized and existing under Philippine laws.

PR, invoking Sec. 37 of RA 166 (Trademark Law) and Art. VIII of the Paris Convention for the Protection of Industrial Property, averred that petitioner's trademark "so closely resembled its own trademark, 'LEE' as previously registered and used in the Philippines, and not abandoned, as to be likely, when applied to or used in connection with petitioner's goods, to cause confusion, mistake and deception on the part of the purchasing public as to the origin of the goods."

In its answer, petitioner contended that its trademark was entirely and unmistakably different from that of private respondent and that its certificate of registration was legally and validly granted.

On 20 February 1984, petitioner caused the publication of its application for registration of the trademark "STYLISTIC MR. LEE" in the Principal Register." On 27 July 1984, private respondent filed a notice of opposition to petitioner's application for registration also on grounds that petitioner's trademark was confusingly similar to its "LEE" trademark. 5 The case was docketed as Inter Partes Case No. 1860.

Director of Patents, on motion filed by PR, issued an order consolidating Inter Partes Cases on grounds that a common question of law was involved.

Director of Patents: Granted PR’s petition for cancellation. Using the dominancy test, Director ruled that petitioner's trademark was confusingly similar to private respondent's mark because "it is the word 'Lee' which draws the attention of the buyer and leads him to conclude that the goods

91

Page 92: IP Developed Reviewer

originated from the same manufacturer. CA affirmed. MR denied.

Issue: Whether or not respondent-registrant's trademark "STYLISTIC MR. LEE" is confusingly similar with the petitioner's trademarks "LEE or LEERIDERS, LEE-LEENS and LEE-SURES.

Held: NO. (holistic test was used)In determining whether a particular name or mark is a "colorable imitation" of another, no all-

embracing rule seems possible in view of the great number of factors which must necessarily be considered in resolving this question of fact, such as the class of product or business to which the article belongs; the product's quality, quantity, or size, including its wrapper or container; the dominant color, style, size, form, meaning of letters, words, designs and emblems used; the nature of the package, wrapper or container; the character of the product's purchasers; location of the business; the likelihood of deception or the mark or name's tendency to confuse;etc.

Proceeding to the task at hand, the essential element of infringement is colorable imitation. This term has been defined as "such a close or ingenious imitation as to be calculated to deceive ordinary purchasers, or such resemblance of the infringing mark to the original as to deceive an ordinary purchaser giving such attention as a purchaser usually gives, and to cause him to purchase the one supposing it to be the other."

2 Tests:1) Test of Dominancy- focuses on the similarity of the prevalent features of the competing

trademarks which might cause confusion or deception and thus constitutes infringement.2) Holistic Test- mandates that the entirety of the marks in question must be considered in

determining confusing similarity.

Applying the foregoing tenets to the present controversy and taking into account the factual circumstances of this case, we considered the trademarks involved as a whole and rule that petitioner's "STYLISTIC MR. LEE" is not confusingly similar to private respondent's "LEE" trademark.

Petitioner's trademark is the whole "STYLISTIC MR. LEE." Although on its label the word "LEE" is prominent, the trademark should be considered as a whole and not piecemeal. The dissimilarities between the two marks become conspicuous, noticeable and substantial enough to matter especially in the light of the following variables that must be factored in.

First, the products involved in the case at bar are, in the main, various kinds of jeans. These are not your ordinary household items like catsup, soysauce or soap which are of minimal cost. Maong pants or jeans are not inexpensive. Accordingly, the casual buyer is predisposed to be more cautious and discriminating in and would prefer to mull over his purchase. Confusion and deception, then, is less likely.

Second, like his beer, the average Filipino consumer generally buys his jeans by brand. He does not ask the sales clerk for generic jeans but for, say, a Levis, Guess, Wrangler or even an Armani. He is, therefore, more or less knowledgeable and familiar with his preference and will not easily be distracted.

Finally, in line with the foregoing discussions, more credit should be given to the "ordinary purchaser." Cast in this particular controversy, the ordinary purchaser is not the "completely unwary consumer" but is the "ordinarily intelligent buyer" considering the type of product involved.

There is no cause for the CA’s apprehension that petitioner's products might be mistaken as

92

Page 93: IP Developed Reviewer

"another variation or line of garments under private respondent's 'LEE' trademark". As one would readily observe, private respondent's variation follows a standard format "LEERIDERS," "LEESURES" and "LEELEENS." It is, therefore, improbable that the public would immediately and naturally conclude that petitioner's "STYLISTIC MR. LEE" is but another variation under private respondent's "LEE" mark.

As we have previously intimated the issue of confusing similarity between trademarks is resolved by considering the distinct characteristics of each case. In the present controversy, taking into account these unique factors, we conclude that the similarities in the trademarks in question are not sufficient as to likely cause deception and confusion tantamount to infringement.

"LEE" is primarily a surname. Private respondent cannot, therefore, acquire exclusive ownership over and singular use of said term.. . . It has been held that a personal name or surname may not be monopolized as a trademark or tradename as against others of the same name or surname. For in the absence of contract, fraud, or estoppel, any man may use his name or surname in all legitimate ways.

In addition to the foregoing, we are constrained to agree with petitioner's contention that private respondent failed to prove prior actual commercial use of its "LEE" trademark in the Philippines before filing its application for registration with the BPTTT and hence, has not acquired ownership over said mark.

On the other hand, petitioner has sufficiently shown that it has been in the business of selling jeans and other garments adopting its "STYLISTIC MR. LEE" trademark since 1975 as evidenced by appropriate sales invoices to various stores and retailers.

Fruit of the Loom v. CA, G.R. No. L-32747, Nov. 29, 1984Facts: Petitioner, a corporation duly organized and existing under the laws the US, is the registrant of a

trademark, FRUIT OF THE LOOM, in the Philippines Patent Office and was issued two Certificates of Registration on November 29, 1957 and July 26, 1958, respectively. The classes of merchandise covered by the first certificate are, among others, men's, women's and children's underwear. The other one covers knitted, netted and textile fabrics.

PR, a domestic corporation, is the registrant of a trademark FRUIT FOR EVE in the Philippine Patent Office and was issued a Certificate of Registration on January 10, 1963 covering garments similar to petitioner's products like women's panties and pajamas.

On March 31, 1965 petitioner filed before the lower court, a complaint for infringement of trademark and unfair competition against PR.

Petitioner principally alleged in the complaint that PR’s trademark FRUIT FOR EVE is confusingly similar to its trademark FRUIT OF THE LOOM used also on women's panties and other textile products. Furthermore, it was also alleged therein that the color get-up and general appearance of PR’s hang tag consisting of a big red apple is a colorable imitation to the hang tag of petitioner.

PR filed an answer invoking the special defense that its registered trademark is not confusingly similar to that of petitioner as the latter alleged. Likewise, PR stated that the trademark FRUIT FOR EVE is being used on ladies' panties and pajamas only whereas petitioner's trademark is used even on men's underwear and pajamas.

RTC: Bureau of Patents is ordered to cancel the registration of the Trademark "Fruit for Eve" and permanently enjoining PR from using the trademark "Fruit for Eve".

CA reversed RTC ruling. MR denied.

Issue: Whether or not PR’s trademark FRUIT FOR EVE and its hang tag are confusingly similar to

93

Page 94: IP Developed Reviewer

petitioner's trademark FRUIT OF THE LOOM and its hang tag so as to constitute an infringement of the latter's trademark rights and justify the cancellation of the former.

Held: NO. (holistic test was used)In cases involving infringement of trademark brought before this Court it has been consistently

held that there is infringement of trademark when the use of the mark involved would be likely to cause confusion or mistake in the mind of the public or to deceive purchasers as to the origin or source of the commodity.

In cases of this nature, there can be no better evidence as to whether there is a confusing similarity in the contesting trademarks than the labels or hang tags themselves. A visual presentation of the labels or hang tags is the best argument for one or the other.

Petitioner asseverates in the third and fourth assignment of errors, which, as We have said, constitute the main argument, that the dominant features of both trademarks is the word FRUIT. In determining whether the trademarks are confusingly similar, a comparison of the words is not the only determinant factor. The trademarks in their entirety as they appear in their respective labels or hang tags must also be considered in relation to the goods to which they are attached. The discerning eye of the observer must focus not only on the predominant words but also on the other features appearing in both labels in order that he may draw his conclusion whether one is confusingly similar to the other.

In the trademarks FRUIT OF THE LOOM and FRUIT FOR EVE, the lone similar word is FRUIT. We agree with the respondent court that by mere pronouncing the two marks, it could hardly be said that it will provoke a confusion, as to mistake one for the other. Standing by itself, FRUIT OF THE LOOM is wholly different from FRUIT FOR EVE. We do not agree with petitioner that the dominant feature of both trademarks is the word FRUIT for even in the printing of the trademark in both hang tags, the word FRUIT is not at all made dominant over the other words.

As to the design and coloring scheme of the hang tags, We believe that while there are similarities in the two marks like the red apple at the center of each mark, We also find differences or dissimilarities which are glaring and striking to the eye.

Hickok v. CA, G.R. No. L-44707, Aug. 31, 1982Facts: Hickok filed a petition to cancel PR’s registration of the trademark of HICKOK for its Marikina shoes

as against petitioner's earlier registration of the same trademark for its other non-competing products

In petitioner-appellee's trademark for handkerchiefs the word 'HICKOK' is in red with white background in the middle of two branches of laurel in light gold. At the lower part thereof is a ribbon on which are the words 'POSITIVELY FINER' in light gold. It carries the same trademark for its underwear except that the words 'POSITIVELY FINER' is in dark gold

In contrast, in respondent-appellant's trademark, the word 'HICKOK' is in white with gold background between the two branches of laurel in red, with the word 'SHOES' also in red below the word 'HICKOK'. The ribbon is in red with the words 'QUALITY AT YOUR FEET,' likewise in red.

Director of Patents ruled in favor of Hickok but the CA reversed this decision. Issue: W/N Hickok trademark for shoes should be cancelled. Held: NO.

It is established doctrine, as held in the above-cited cases, that "emphasis should be on the

94

Page 95: IP Developed Reviewer

similarity of the products involved and not on the arbitrary classification or general description of their properties or characteristics" and that "the mere fact that one person has adopted and used a trademark on his goods does not prevent the adoption and use of the same trademark by others on unrelated articles of a different kind." Taking into account the facts of record that petitioner, a foreign corporation registered the trademark for its diverse articles of men's wear such as wallets, belts and men's briefs which are all manufactured here in the Philippines by a licensee Quality House, Inc. but are so labelled as to give the misimpression that the said goods are of foreign (stateside) manufacture and that respondent secured its trademark registration exclusively for shoes (which neither petitioner nor the licensee ever manufactured or traded in) and which are clearly labelled in block letters as "Made in Marikina, Rizal, Philippines," no error can be attributed to the appellate court in upholding respondent's registration of the same trademark for his unrelated and non-competing product of Marikina shoes.

In the recent case of Acoje Mining Co., Inc. vs. Director of Patents, SC stated:Can it be said then that petitioner's application would be likely to cause confusion or mistake

on the part of the buying public? The answer should be in the negative. It does not defy common sense to assert that a purchaser would be cognizant of the product he is buying. There is quite a difference between soy sauce and edible oil. If one is in the market for the former, he is not likely to purchase the latter just because on the trademark LOTUS. Even on the rare occasion that a mistake does occur, it can easily be rectified. Moreover, there is no denying that the possibility of confusion is remote considering petitioner's trademark being in yellow and red while that of the Philippine Refining Company being in green and yellow, and the much smaller size of petitioner's trademark. When regard is had for the principle that the two trademarks in their entirety as they appear in their respective labels should be considered in relation to the goods advertised before registration could be denied, the conclusion is inescapable that respondent Director ought to have reached a different conclusion. Petitioner has successfully made out a case for registration.

Faberge Inc. v. IAC, G.R. No. 71189, November 4, 1992Facts: Director of Patents authorized herein private respondent Co Beng Kay to register the trademark

"BRUTE" for the briefs manufactured and sold by his Corporation in the domestic market vis-a-vis petitioner's opposition grounded on similarity of said trademark with petitioner's own symbol "BRUT"

“BRUT” enjoyed prior registration for after-shave lotion, shaving cream, deodorant, talcum powder, and toilet soap.

Observation of Director of Patents: the involved trademarks are grossly different in their overall appearance that even at a distance a would-be purchaser could easily distinguish what is BRUTE brief and what is BRUT after-shave lotion, lotion and the like. Opposer's mark BRUT or BRUT 33, as shown in Exhibit "6", is predominantly colored green with a blue and white band at the middle portion of the container. Additionally, the meaning or connotation of the bare word marks of opposer, BRUT, and BRUTE of respondent, are clearly different and not likely to be confused with each other. BRUT simply means "dry", and also, "to browse"; while BRUTE means "ferocious, sensual”.

Director of Patents ruled in favor of Co Beng Key and the ratio is that: Petitioner has never applied for, registered nor used its trademarks for briefs in commerce or trade in the Philippines. PR seeks to register his trademark "BRUTE" only for briefs which is a product non-competitive to and entirely unrelated with petitioner's aforementioned products.

Decision of Director of Patents upheld by CA and the ratio is that controlling ruling to the effect that

95

Page 96: IP Developed Reviewer

the identical trademark can be used by different manufacturers for products that are non-competing and unrelated.

Faberge’s contentions in SC: It is petitioner’s contention that the rulings in the above-cited cases should not have been relied

upon and places emphasis on American jurisprudential doctrines to the effect that sale of cosmetics and wearing apparel under similar marks is likely to cause confusion.

Petitioner also wants it considered that they have an alleged application for registration of the trademark "BRUT 33 DEVICE" for briefs as an explicit proof that petitioner intended to expand its mark "BRUT" to other goods

Petitioner insists that in view of the repeal of RA 166 (which advocated the related goods theory) by RA 666 which deleted the phrase found in the old law that the merchandise must be substantially the same descriptive properties, respondent Court should have heeded the pronouncement in the Ang case that there can be unfair competition even if the goods are noncompeting.

 Issue: W/N PR should be allowed to register “BRUTE” for its briefs Held: YES.

In as much as petitioner has not ventured in the production of briefs, an item which is not listed in its certificate of registration, petitioner can not and should not be allowed to feign that PR had invaded petitioner's exclusive domain. As for its claim that it wanted to expand the mark “BRUT” a mere application by petitioner in this aspect does not suffice and may not vest an exclusive right in its favor that can ordinarily be protected by the Trademark Law. In short, paraphrasing Section 20 of the Trademark Law as applied to the documentary evidence adduced by petitioner, the certificate of registration issued by the Director of Patents can confer upon petitioner the exclusive right to use its own symbol only to those goods specified in the certificate, subject to any conditions and limitations stated therein. One who has adopted and used a trademark on his goods does not prevent the adoption and use of the same trademark by other for products which are of different description.  In the situation before us, the goods are obviously different from each other with "absolutely no iota of similitude" as stressed in respondent court's judgment. They are so foreign to each other as to make it unlikely that purchasers would think that petitioner is the manufacturer of respondent' goods. The mere fact that one person has adopted and used a trademark on his goods does not prevent the adoption and use of the same trademark by others on unrelated articles of a different kind.  The glaring discrepancies between the two products had been amply portrayed to such an extent that indeed, "a purchaser who is out in the market for the purpose of buying respondent's BRUTE brief would definitely be not mistaken or misled into buying BRUT after-shave lotion or deodorant" as categorically opined in the decision of the Director of Patents relative to the inter-partes case.  On the conflict between Section 4(d) relied upon by JBL in Sta. Ana case which does not require that the goods manufactured by the second user be related to the goods produced by the senior user, while Section 20 limits t.he exclusive right of the senior user only to those goods specified in the certificate of registration  Sec. 20 controlling. If the certificate of registration were to be deemed as including goods not specified therein, then a situation may arise whereby an applicant may be tempted to register a trademark on any and all goods which his mind may conceive even if he had never intended to use the trademark for the said goods. We believe that such omnibus registration is not contemplated by

96

Page 97: IP Developed Reviewer

our Trademark Law.

Asia Brewery Inc. v. CA, G.R. No. 103543, July 5, 1993Facts: On September 15, 1988, San Miguel Corporation (SMC) filed a complaint against Asia Brewery Inc.

(ABI) for infringement of trademark and unfair competition on account of the latter's BEER PALE PILSEN or BEER NA BEER product which has been competing with SMC's SAN MIGUEL PALE PILSEN for a share of the local beer market.

On August 27, 1990, a decision was rendered by the RTC, presided over by Judge Bersamira, dismissing SMC's complaint because ABI "has not committed trademark infringement or unfair competition against" SMC.

CA reversed the RTC and found ABI GUILTY of infringement of trademark and unfair competition. CA decision modified its decision in that PR ABI, its officers, agents, servants and employees are

hereby permanently enjoined and restrained from manufacturing, putting up, selling, advertising, offering or announcing for sale, or supplying Beer Pale Pilsen, or any similar preparation, manufacture or beer in bottles and under labels substantially identical with or like the said bottles and labels of petitioner SMC; recall all its products bearing the mark Beer Pale Pilsen from its retailers and deliver these as well as all labels, signs, prints, packages, wrappers, receptacles and advertisements bearing the infringing mark; ordered to pay plaintiff the sum of P2M as moral damages and P500K by way of exemplary damages.

ABI appealed to SC by a petition for certiorari under Rule 45 of the Rules of Court. Issues / Held:1.)     W/N ABI infringes SMC's trademark: San Miguel Pale Pilsen with Rectangular Hops and Malt Design, and thereby commits unfair competition against the latter. NO. (used dominancy test) 

Sec. 22 of Republic Act No. 166, otherwise known as the Trademark Law, defines what constitutes infringement:  Sec. 22. Infringement, what constitutes.  Any person who shall use, without the consent of the registrant, any reproduction, counterfeit, copy or colorable imitation of any registered mark or trade-name in connection with the sale, offering for sale, or advertising of any goods, business or services on or in connection with which such use is likely to cause confusion or mistake or to deceive purchasers or others as to the source or origin of such goods or services, or identity of such business; or reproduce, counterfeit, copy or colorably imitate any such mark or trade-name and apply such reproduction, counterfeit, copy, or colorable imitation to labels, signs, prints, packages, wrappers, receptacles or advertisements intended to be used upon or in connection with such goods, business or services, shall be liable to a civil action by the registrant for any or all of the remedies herein provided.  This definition implies that only registered trade marks, trade names and service marks are protected against infringement or unauthorized use by another or others. The use of someone else's registered trademark, trade name or service mark is unauthorized, hence, actionable, if it is done "without the consent of the registrant."  Infringement is determined by the "test of dominancy" rather than by differences or variations in the details of one trademark and of another. Similarity in size, form and color, while relevant, is not conclusive. If the competing trademark contains the main or essential or dominant features of another, and confusion and deception is likely to result, infringement takes place. Duplication or imitation is not necessary; nor is it necessary that the infringing

97

Page 98: IP Developed Reviewer

label should suggest an effort to imitate. The question at issue in cases of infringement of trademarks is whether the use of the marks involved would be likely to cause confusion or mistakes in the mind of the public or deceive purchasers. What are the dissimilarities in the trade dress or appearance of the 2 competing products:(1) The SAN MIGUEL PALE PILSEN bottle has a slender tapered neck.The BEER PALE PILSEN bottle has a fat, bulging neck.(2) The words "pale pilsen" on SMC's label are printed in bold and laced letters along a diagonal band, whereas the words "pale pilsen" on ABI's bottle are half the size and printed in slender block letters on a straight horizontal band(3) The names of the manufacturers are prominently printed on their respective bottles.SAN MIGUEL PALE PILSEN is "Bottled by the San Miguel Brewery, Philippines ," whereas BEER PALE PILSEN is "Especially brewed and bottled by Asia Brewery Incorporated, Philippines ."(4) On the back of ABI's bottle is printed in big, bold letters, under a row of flower buds and leaves, its copyrighted slogan: "BEER NA BEER!" Whereas SMC's bottle carries no slogan.(5) The back of the SAN MIGUEL PALE PILSEN bottle carries the SMC logo, whereas the BEER PALE PILSEN bottle has no logo.(6) The SAN MIGUEL PALE PILSEN bottle cap is stamped with a coat of arms and the words "San Miguel Brewery Philippines" encircling the same.The BEER PALE PILSEN bottle cap is stamped with the name "BEER" in the center, surrounded by the words "Asia Brewery Incorporated Philippines."(7) Finally, there is a substantial price difference between BEER PALE PILSEN (currently at P4.25 per bottle) and SAN MIGUEL PALE PILSEN (currently at P7.00 per bottle)W/N ABI committed unfair competition 

Will the fact that the words “pale pilsen” are part of ABI’s trademark constitute infringement?NO. MIGUEL PALE PILSEN, for "pale pilsen" are generic words descriptive of the color ("pale"), of a type of beer ("pilsen") , which is a light bohemian beer with a strong hops flavor that originated in the City of Pilsen in Czechoslovakia and became famous in the Middle Ages. No one may appropriate generic or descriptive words. They belong to the public domain. A word or a combination of words which is merely descriptive of an article of trade, or of its composition, characteristics, or qualities, cannot be appropriated and protected as a trademark to the exclusion of its use by others. . . . inasmuch as all persons have an equal right to produce and vend similar articles, they also have the right to describe them properly and to use any appropriate language or words for that purpose, and no person can appropriate to himself exclusively any word or expression, properly descriptive of the article, its qualities, ingredients or characteristics, and thus limit other persons in the use of language appropriate to the description of their manufactures, the right to the use of such language being common to all. This rule excluding descriptive terms has also been held to apply to trade-names.

ABI’s contention that bottle size, shape and color may not be the exclusive property of any one beer manufacturer is well taken. SMC's being the first to use the steinie bottle does not give SMC a vested right to use it to the exclusion of everyone else. Being of functional or common use, and not the exclusive invention of any one, it is available to all who might need to use it within the industry. Nobody can acquire any exclusive right to market articles supplying simple human needs in containers or wrappers of the general form, size and character commonly and immediately used in marketing such articles. Protection against imitation should be properly confined to nonfunctional features. Even if purely functional elements are slavishly copied, the resemblance will

98

Page 99: IP Developed Reviewer

not support an action for unfair competition, and the first user cannot claim secondary meaning protection. 2.)     W/N ABI committed unfair competition. NO.

In order that there may be deception of the buying public in the sense necessary to constitute unfair competition, it is necessary to suppose a public accustomed to buy, and therefore to some extent familiar with, the goods in question. The test of fraudulent simulation is to be found in the likelihood of the deception of persons in some measure acquainted with an established design and desirous of purchasing the commodity with which that design has been associated. The test is not found in the deception, or possibility of the deception, of the person who knows nothing about the design which has been counterfeited, and who must be indifferent as between that and the other. The simulation, in order to be objectionable, must be such as appears likely to mislead the ordinarily intelligent buyer who has a need to supply and is familiar with the article that he seeks to purchase.

SMC claims that the "trade dress" of BEER PALE PILSEN is "confusingly similar" to its SAN MIGUEL PALE PILSEN because both are bottled in 320 ml. steinie type, amber-colored bottles with white rectangular labels.

However, when as in this case, the names of the competing products are clearly different and their respective sources are prominently printed on the label and on other parts of the bottle, mere similarity in the shape and size of the container and label, does not constitute unfair competition. The steinie bottle is a standard bottle for beer and is universally used. SMC did not invent it nor patent it. The fact that SMC's bottle is registered under R.A. No. 623 (as amended by RA 5700, An Act to Regulate the Use of Duly Stamped or Marked Bottles, Boxes, Casks, Kegs, Barrels and Other Similar Containers) simply prohibits manufacturers of other foodstuffs from the unauthorized use of SMC's bottles by refilling these with their products. It was not uncommon then for products such as patis (fish sauce) and toyo (soy sauce) to be sold in recycled SAN MIGUEL PALE PILSEN bottles. Registration of SMC's beer bottles did not give SMC a patent on the steinie or on bottles of similar size, shape or color.

Considering further that SAN MIGUEL PALE PILSEN has virtually monopolized the domestic beer market for the past hundred years, those who have been drinking no other beer but SAN MIGUEL PALE PILSEN these many years certainly know their beer too well to be deceived by a newcomer in the market. If they gravitate to ABI's cheaper beer, it will not be because they are confused or deceived, but because they find the competing product to their taste. 3.)     W/N Del Monte ruling applies to all products. NO.

The Del Monte ruling states “to determine whether a trademark has been infringed, we must consider the mark as a whole and not as dissected. If the buyer is deceived, it is attributable to the marks as a totality, not usually to any part of it” xxx  That ruling may not apply to all kinds of products. The Court itself cautioned that in resolving cases of infringement and unfair competition, the courts should "take into consideration several factors which would affect its conclusion, to wit: 1the age, training and education of the usual purchaser, 2the nature and cost of the article, 3 whether the article is bought for immediate consumption and also the 4 conditions under which it is usually purchased ." Our ruling in Del Monte would not apply to beer which is not usually picked from a store shelf but ordered by brand by the beer drinker himself from the storekeeper or waiter in a pub or restaurant. Dissenting Opinion

99

Page 100: IP Developed Reviewer

Cruz, J.While the Court does recognize these distinctions, it does not agree with the conclusion that

there was no infringement or unfair competition. It seems to us that the lower courts have been so preoccupied with the details that they have not seen the total picture.  The question is not whether the two articles are distinguishable by their labels when set aside by side but whether the general confusion made by the article upon the eye of the casual purchaser who is unsuspicious and off his guard, is such as to likely result in his confounding it with the original. As observed in several cases, the general impression of the ordinary purchaser, buying under the normally prevalent conditions in trade and giving the attention such purchasers usually give in buying that class of goods, is the touchstone.  It has been held that in making purchases, the consumer must depend upon his recollection of the appearance of the product which he intends to purchase. The buyer having in mind the mark/label of the respondent must rely upon his memory of the petitioner's mark. Unlike the judge who has ample time to minutely examine the labels in question in the comfort of his sala, the ordinary shopper does not enjoy the same opportunity.  The court therefore should be guided by its first impression, for a buyer acts quickly and is governed by a casual glance, the value of which may be dissipated as soon as the court assumes to analyze carefully the respective features of the mark.  It has also been held that it is not the function of the court in cases of infringement and unfair competition to educate purchasers but rather to take their carelessness for granted, and to be ever conscious of the fact that marks need not be identical. A confusing similarity will justify the intervention of equity. The judge must also be aware of the fact that usually a defendant in cases of infringement does not normally copy but makes only colorable changes. Well has it been said that the most successful form of copying is to employ enough points of similarity to confuse the public with enough points of difference to confuse the courts.

McDonald’s v. L.C. Big Mak Burger, Inc., G.R. No. 143993, Aug. 18, 2004Facts: Petitioner McDonald's Corporation ("McDonald's") is a corporation organized under the laws of

Delaware, United States. McDonald's owns a family of marks including the "Big Mac" mark for its "double-decker” hamburger sandwich.

McDonald's registered this trademark with the United States Trademark Registry on 16 October 1979. Based on this Home Registration, McDonald's applied for the registration of the same mark in the Principal Register of the then Philippine Bureau of Patents, Trademarks and Technology

McDonald's introduced its "Big Mac" hamburger sandwiches in the Philippine market in September 1981.

On 18 July 1985, the PBPTT allowed registration of the "Big Mac" mark in the Principal Register based on its Home Registration in the United States.

Petitioner McGeorge Food Industries ("petitioner McGeorge"), a domestic corporation, is McDonald's Philippine franchisee.

Respondent L.C. Big Mak Burger, Inc. ("respondent corporation") is a domestic corporation which operates fast-food outlets and snack vans in Metro Manila and nearby provinces.

On 21 October 1988, respondent corporation applied with the PBPTT for the registration of the "Big Mak" mark for its hamburger sandwiches. McDonald's opposed respondent corporation's application on the ground that "Big Mak" was a colorable imitation of its registered "Big Mac" mark for the same food products.

100

Page 101: IP Developed Reviewer

Having received no reply from respondent, petitioner sued respondent for trademark infringement and unfair competition.

RTC- TRO against respondent. Respondent admitted that they have been using the name “Big Mak” but claimed that petitioner

does not have exclusive right to the “Big Mac” mark since another corporation registered it in 1979. Petitioner denied respondent’s claim and that the corporation mentioned by respondent registered

the word “Big Mac” only in a Supplemental Register. RTC – respondents liable for trademark infringement and unfair competition. CA – reversed. Vast difference in the appearance of the product and the manner that the

tradename “Big Mak” is being used and presented to the public (Petitioner- air-con resto/ Respondent- snack vans)

Issue: Whether respondent is liable for (1) trademark infringement and (2) unfair competition?

SC Ruling:

1. On trademark infringement:- YES, respondent is liable.- Section 22 of RA 166 - To establish trademark infringement, the following elements must be shown: (1) the validity of

plaintiff's mark; (2) the plaintiff's ownership of the mark; and (3) the use of the mark or its colorable imitation by the alleged infringer results in "likelihood of confusion." Of these, it is the element of likelihood of confusion that is the gravamen of trademark infringement.

Respondent’s contention with regard to the validity of Petitioner’s mark (1st element – validity of mark)

- The two words in the "Big Mac" mark, it is only the word "Mac" that is valid because the word

"Big" is generic and descriptive (proscribed under Section 4[e]), and thus "incapable of exclusive appropriation."

SC Ruling on the validity of mark: The contention has no merit. The "Big Mac" mark, which should be treated in its entirety and not dissected word for word, is neither generic nor descriptive. "Big Mac" falls under the class of fanciful or arbitrary marks as it bears no logical relation to the actual characteristics of the product it represents. As such, it is highly distinctive and thus valid.

A mark which is not registered in the Principal Register, and thus not distinctive, has no real protection. Indeed, we have held that registration in the Supplemental Register is not even a prima facie evidence of the validity of the registrant's exclusive right to use the mark on the goods specified in the certificate.

Types of Confusion1. Confusion of Goods- exists when the infringing mark is used on goods of a similar kind2. Confusion of Service – exists when the products are non-competing but related enough to

produce confusion of affiliation (what was found present in this case)

101

Page 102: IP Developed Reviewer

SC Ruling on the type of confusion: Defendants' unauthorized acts are likely, and calculated, to confuse, mislead or deceive the public into believing that the products and services offered by defendant Big Mak Burger, and the business it is engaged in, are approved and sponsored by, or affiliated with, plaintiffs. Respondents assert that their "Big Mak" hamburgers cater mainly to the low-income group while petitioners' "Big Mac" hamburgers cater to the middle and upper income groups. Even if this is true, the likelihood of confusion of business remains, since the low-income group might be led to believe that the "Big Mak" hamburgers are the low-end hamburgers marketed by petitioners. After all, petitioners have the exclusive right to use the "Big Mac" mark.

3rd element – Likelihood of confusion

- In determining likelihood of confusion, jurisprudence has developed two tests, the dominancy test and the holistic test. The dominancy test focuses on the similarity of the prevalent features of the competing trademarks that might cause confusion. In contrast, the holistic test requires the court to consider the entirety of the marks as applied to the products, including the labels and packaging, in determining confusing similarity

CA used the Holistic test in their decision.

SC Ruling on the Likelihood of Confusion: Used the Dominancy test. Applying the dominancy test, the Court finds that respondents' use of the "Big Mak" mark results in likelihood of confusion. First, "Big Mak" sounds exactly the same as "Big Mac." Second, the first word in "Big Mak" is exactly the same as the first word in "Big Mac." Third, the first two letters in "Mak" are the same as the first two letters in "Mac." Fourth, the last letter in "Mak" while a "k" sounds the same as "c" when the word "Mak" is pronounced. Fifth, in Filipino, the letter "k" replaces "c" in spelling, thus "Caloocan" is spelled "Kalookan."

In short, aurally the two marks are the same, with the first word of both marks phonetically the same, and the second word of both marks also phonetically the same. Visually, the two marks have both two words and six letters, with the first word of both marks having the same letters and the second word having the same first two letters. In spelling, considering the Filipino language, even the last letters of both marks are the same.

Clearly, respondents have adopted in "Big Mak" not only the dominant but also almost all the features of "Big Mac." Applied to the same food product of hamburgers, the two marks will likely result in confusion in the public mind.

Respondents' inability to explain sufficiently how and why they came to choose "Big Mak" for their hamburger sandwiches indicates their intent to imitate petitioners' "Big Mac" mark. Absent proof that respondents' adoption of the "Big Mak" mark was due to honest mistake or was fortuitous, the inescapable conclusion is that respondents adopted the "Big Mak" mark to "ride on the coattails" of the more established "Big Mac" mark.

Petitioners' failure to present proof of actual confusion does not negate their claim of trademark infringement. As noted in American Wire & Cable Co. v. Director of Patents, Section 22 requires

102

Page 103: IP Developed Reviewer

the less stringent standard of "likelihood of confusion" only. While proof of actual confusion is the best evidence of infringement, its absence is inconsequential

2. On the issue Unfair Competition

- Yes, respondent is liable.

- Section 29 ("Section 29") of RA 166 defines unfair competition, thus:

xxxx

Any person who will employ deception or any other means contrary to good faith by which he shall pass off the goods manufactured by him or in which he deals, or his business, or services for those of the one having established such goodwill, or who shall commit any acts calculated to produce said result, shall be guilty of unfair competition, and shall be subject to an action therefor.

In particular, and without in any way limiting the scope of unfair competition, the following shall be deemed guilty of unfair competition:

(a) Any person, who in selling his goods shall give them the general appearance of goods of another manufacturer or dealer, either as to the goods themselves or in the wrapping of the packages in which they are contained, or the devices or words thereon, or in any feature of their appearance, which would be likely to influence purchasers to believe that the goods offered are those of a manufacturer or dealer, other than the actual manufacturer or dealer, or who otherwise clothes the goods with such appearance as shall deceive the public and defraud another of his legitimate trade, or any subsequent vendor of such goods or any agent of any vendor engaged in selling such goods with a like purpose;

(b) Any person who by any artifice, or device, or who employs any other means calculated to induce the false belief that such person is offering the services of another who has identified such services in the mind of the public; or

(c) Any person who shall make any false statement in the course of trade or who shall commit any other act contrary to good faith of a nature calculated to discredit the goods, business or services of another. (Emphasis supplied)

- The essential elements of an action for unfair competition are (1) confusing similarity in the general appearance of the goods, and (2) intent to deceive the public and defraud a competitor.74 The confusing similarity may or may not result from similarity in the marks, but may result from other external factors in the packaging or presentation of the goods. The intent to deceive and defraud may be inferred from the similarity of the appearance of the goods as offered for sale to the public.75 Actual fraudulent intent need not be show

- Unfair competition is broader than trademark infringement and includes passing off goods with or without trademark infringement. Trademark infringement is a form of unfair competition.77

103

Page 104: IP Developed Reviewer

Trademark infringement constitutes unfair competition when there is not merely likelihood of confusion, but also actual or probable deception on the public because of the general appearance of the goods.

- If respondents sold egg sandwiches only instead of hamburger sandwiches, their use of the "Big Mak" mark would not give their goods the general appearance of petitioners' "Big Mac" hamburgers. In such case, there is only trademark infringement but no unfair competition. However, since respondents chose to apply the "Big Mak" mark on hamburgers, just like petitioner's use of the "Big Mac" mark on hamburgers, respondents have obviously clothed their goods with the general appearance of petitioners' goods.

Mighty Corp. v. E.J. Gallo Winery, et al. G.R. No. 154342, July 14, 2004Facts: Respondent Gallo Winery is a foreign corporation not doing business in the Philippines but

organized and existing under the laws of the CA, USA. Gallo Winery produces different kinds of wines and brandy products and sells them in many countries under different registered trademarks, including the GALLO and ERNEST & JULIO GALLO wine trademarks.

Respondent domestic corporation, Andresons, has been Gallo Winery’s exclusive wine importer and distributor in the Philippines since 1991, selling these products in its own name and for its own account.

Petitioners Mighty Corporation and La Campana and their sister company, Tobacco Industries of the Philippines, are engaged in the cultivation, manufacture, distribution and sale of tobacco products

1971 : Gallo Winery registered its GALLO wine trademark in the principal register of the PPO 1973 : Tobacco started using the trademark GALLO filter cigarette mark. The BIR also approved

Tobacco’s use of the GALLO 100’s cigarette mark in this year, both for the manufacture and sale of its cigarette products.

1974: Tobacco applied for, but eventually did not pursue, the registration of the GALLO cigarette trademark in the principal register of the PPO.

1976 : BIR approved Tobacco’s use of the GALLO filter cigarette mark, both for the manufacture and sale of its cigarette products.

1984: Tobacco assigned the GALLO cigarette trademark to La Campana, which in 1985, applied for trademark registration in the Philippine Patent Office.

1985: the National Library issued Certificate of Copyright Registration for La Campana’s lifetime copyright claim over GALLO cigarette labels. Subsequently, La Campana authorized Mighty Corporation to manufacture and sell cigarettes bearing the GALLO trademark.

1988 : BIR approved Mighty’ use of GALLO 100’s cigarette brand, under licensing agreement with Tobacco

1989 : BIR approved Mighty’ use of GALLO SPECIAL MENTHOL 100’s cigarette brand 1990 : Gallo Winery applied for registration its ERNEST & JULIO GALLO wine trademark , the

records however do not disclose if it was ever approved by the Director of Patents. 1991 : Gallo Winery renewed its GALLO wine trademark for another 20 years.

Despite the 1971 registration, respondents claim that they first introduced and sold the GALLO and ERNEST & JULIO GALLO wines in the Philippines circa 1974 within the then U.S. military facilities only. By 1979, they had expanded their Philippine market through authorized distributors and independent outlets.

104

Page 105: IP Developed Reviewer

Respondents claim that they first learned about the existence of GALLO cigarettes in the latter part of 1992 when an Andresons employee saw such cigarettes on display with GALLO wines in a Davao supermarket wine cellar section. Forthwith, respondents sent a demand letter to petitioners asking them to stop using the GALLO trademark, to no avail.

Respondents sued petitioners for trademark and tradename infringement and unfair competition, with a prayer for damages and preliminary injunction. Respondents charged petitioners with violating Article 6 of the Paris Convention, and RA 166 (Trademark Law)

They claimed that petitioners adopted the GALLO trademark to ride on Gallo Winery’s GALLO and ERNEST & JULIO GALLO trademarks established reputation and popularity, thus causing confusion, deception and mistake on the part of the purchasing public who had always associated GALLO and ERNEST & JULIO GALLO trademarks with Gallo Winery’s wines.

Petitioner’s arguments petitioner’s GALLO cigarettes and Gallo Winery’s wines were totally unrelated products; Gallo Winery’s GALLO trademark registration certificate covered wines only, not cigarettes; GALLO cigarettes and GALLO wines were sold through different channels of trade; GALLO cigarettes, sold at P4.60 for GALLO filters and P3 for GALLO menthols, were low-cost

items compared to Gallo Winery’s high-priced luxury wines which cost between P98 to P242.50; the target market of Gallo Winery’s wines was the middle or high-income bracket with at least

P10,000 monthly income while GALLO cigarette buyers were farmers, fishermen, laborers and other low-income workers;

the dominant feature of the GALLO cigarette mark was the rooster device with the manufacturer’s name clearly indicated as MIGHTY CORPORATION while, in the case of Gallo Winery’s wines, it was the full names of the founders-owners ERNEST & JULIO GALLO or just their surname GALLO;

by their inaction and conduct, respondents were guilty of laches and estoppel; and petitioners acted with honesty, justice and good faith in the exercise of their right to manufacture

and sell GALLO cigarettes.

RTC denied respondent’s prayer for the issuance of a writ of PI, holding that respondent’s GALLO trademark registration certificate covered wines only, that respondents’ wines and petitioners’ cigarettes were not related goods and respondents failed to prove material damage or great irreparable injury. It also denied MR. CA likewise dismissed respondents’ petition for review on certiorari.

After trial on the merits, however, RTC, held petitioners liable for, and permanently enjoined them from, committing trademark infringement and unfair competition with respect to the GALLO trademark.

On appeal, the CA affirmed the Makati RTC decision and subsequently denied petitioner’s MR.

Pertinent Issue 1: Whether or not petitioners committed trademark infringement.

Sub-Issue1: Whether or not there is LIKELIHOOD OF CONFUSION, MISTAKE, OR DECEIT as to the identity or source of goods or business. Ruling: None. Identical mark does not, by itself, lead to a legal conclusion that there is trademark

infringement.

105

Page 106: IP Developed Reviewer

Whether a trademark causes confusion and is likely to deceive the public hinges on "colorable imitation" which has been defined as "such similarity in form, content, words, sound, meaning, special arrangement or general appearance of the trademark or tradename in their overall presentation or in their essential and substantive and distinctive parts as would likely mislead or confuse persons in the ordinary course of purchasing the genuine article."

Jurisprudence has developed TWO TESTS in determining similarity and likelihood of confusion in trademark resemblance:a. the DOMINANCY TEST focuses on the similarity of the prevalent features of the competing

trademarks which might cause confusion or deception, and thus infringement. If the competing trademark contains the main, essential or dominant features of another, and confusion or deception is likely to result, infringement takes place. Duplication or imitation is not necessary; nor is it necessary that the infringing label should suggest an effort to imitate. The question is whether the use of the marks involved is likely to cause confusion or mistake in the mind of the public or deceive purchasers.

b. the HOLISTIC TEST requires that the entirety of the marks in question be considered in resolving confusing similarity. Comparison of words is not the only determining factor. The trademarks in their entirety as they appear in their respective labels or hang tags must also be considered in relation to the goods to which they are attached. The discerning eye of the observer must focus not only on the predominant words but also on the other features appearing in both labels in order that he may draw his conclusion whether one is confusingly similar to the other.

Applying the Dominancy and Holistic Tests, there are many different features like color schemes, art works and other markings of both products drown out the similarity between them: the use of the word ‘GALLO’ ― a family surname for the Gallo Winery’s wines and a Spanish word for rooster for petitioners’ cigarettes.

the dominant feature of the GALLO CIGARETTE TRADEMARK is the device of a large rooster facing left, outlined in black against a gold background. The rooster’s color is either green or red-green for GALLO menthols and red for GALLO filters. Directly below the large rooster device is the word GALLO. The rooster device is given prominence in the GALLO cigarette packs in terms of size and location on the labels.

The GALLO mark appears to be a fanciful and arbitrary mark for the cigarettes as it has no relation at all to the product but was chosen merely as a trademark due to the fondness for fighting cocks of the son of petitioners’ president. Furthermore, petitioners adopted GALLO, the Spanish word for rooster, as a cigarette trademark to appeal to one of their target markets, the sabungeros (cockfight aficionados).

Also, as admitted by respondents themselves, on the side of the GALLO cigarette packs are the words "MADE BY MIGHTY CORPORATION," thus clearly informing the public as to the identity of the manufacturer of the cigarettes.

On the other hand, GALLO WINERY’S WINE AND BRANDY LABELS are diverse. In many of them, the labels are embellished with sketches of buildings and trees, vineyards or a bunch of grapes while in a few, one or two small roosters facing right or facing each other (atop the EJG crest, surrounded by leaves or ribbons), with additional designs in green, red and yellow colors, appear as minor features thereof.

Directly below or above these sketches is the entire printed name of the founder-owners, "ERNEST & JULIO GALLO" or just their surname "GALLO," which appears in different fonts, sizes, styles and labels, unlike petitioners’ uniform casque-font bold-lettered GALLO mark.

106

Page 107: IP Developed Reviewer

Moreover, on the labels of Gallo Winery’s wines are printed the words "VINTED AND BOTTLED BY ERNEST & JULIO GALLO, MODESTO, CALIFORNIA.”

Sub-Issue2: Whether respondents GALLO and Adresons’ ACTUAL COMMERCIAL USE in the Philippines of GALLO CIGARETTE TRADEMARK preceded that of the petitioners. Ruling: By respondents’ own judicial admission, the GALLO wine trademark was registered in the

Philippines in 1971 but the wine itself was first marketed and sold in the country only in 1974 and only within the former U.S. military facilities, and outside thereof, only in 1979.

Nothing at all, however, was presented to evidence the alleged sales of GALLO wines in the Philippines in 1974 or, for that matter, prior to 1981.

On the other hand, petitioners and its predecessor-in-interest, Tobacco, have indeed been using and selling GALLO cigarettes in the Philippines since 1973 or before 1981.

Actual use in commerce in the Philippines is an essential prerequisite for the acquisition of ownership over a trademark pursuant to Sec. 2 and 2-A of the Philippine Trademark Law (R.A. No. 166).

Following universal acquiescence and comity, our municipal law on trademarks regarding the requirement of actual use in the Philippines must subordinate an international agreement inasmuch as the apparent clash is being decided by a municipal tribunal

In other words, (a foreign corporation) may have the capacity to sue for infringement irrespective of lack of business activity in the Philippines on account of Section 21-A of the Trademark Law but the question of whether they have an exclusive right over their symbol as to justify issuance of the controversial writ will depend on actual use of their trademarks in the Philippines in line with Sections 2 and 2-A of the same law.

However, registration alone will not suffice. Actual use in commerce or business is a prerequisite in the acquisition of the right of ownership over a trademark. A registration certificate serves merely as prima facie evidence. It is not conclusive but can and may be rebutted by controverting evidence.

Sub-Issue 3: Whether Respondents’ GALLO TRADEMARK REGISTRATION IS LIMITED TO WINES ONLYRuling: YES. We also note that the GALLO trademark registration certificates in the Philippines and in

other countries expressly state that they cover wines only, without any evidence or indication that registrant Gallo Winery expanded or intended to expand its business to cigarettes. Thus, by strict application of Section 20 of the Trademark Law, Gallo Winery’s exclusive right to use the GALLO trademark should be limited to wines, the only product indicated in its registration certificates.

The certificate of registration issued by the Director of Patents can confer upon petitioner the exclusive right to use its own symbol only to those goods specified in the certificate. That one who has adopted and used a trademark on his goods does not prevent the adoption and use of the same trademark by others for products which are of a different description.

Although the dissimilarity of goods will not preclude relief if the junior user's goods are not remote from any other product which the first user would be likely to make or sell, it is equally true that as aforesaid, the protective mantle of the Trademark Law extends only to the goods used by the first user as specified in the certificate of registration following the clear message conveyed by Section 20.

107

Page 108: IP Developed Reviewer

Really, if the certificate of registration were to be deemed as including goods not specified therein, then a situation may arise whereby an applicant may be tempted to register a trademark on any and all goods which his mind may conceive even if he had never intended to use the trademark for the said goods. We believe that such omnibus registration is not contemplated by our Trademark Law.

Sub-Issue 4: Whether or not wines and cigarettes are IDENTICAL, SIMILAR, COMPETING, OR RELATED GOODS. Ruling: No. Tobacco and alcohol products may be considered related only in cases involving special

circumstances which exist only if a famous mark is involved and there is a demonstrated intent to capitalize on it. Both of these are absent in the present case.

Although confusion of goods can arise either where litigants are competing or non-competing, Petitioners’ use of the GALLO cigarette trademark is not likely to cause confusion or mistake, or to deceive the "ordinarily intelligent buyer" of either wines or cigarettes or both as to the identity of the goods, their source and origin, or identity of the business of petitioners and respondents.

Obviously, wines and cigarettes are not identical or competing products. Neither do they belong to the same class of goods.

Both products are also not related. First, not only wines and cigarettes can be used for pleasure and relaxation and can be harmful when taken in excess. Indeed, it would be a grave abuse of discretion to treat wines and cigarettes as similar or related products likely to cause confusion just because they are pleasure-giving, relaxing or potentially harmful. Second, it is common knowledge that supermarkets sell an infinite variety of wholly unrelated products and the goods here involved, wines and cigarettes, have nothing whatsoever in common with respect to their essential characteristics, quality, quantity, size, including the nature of their packages, wrappers or containers.

They are also non-competing products. There is a distinct price segmentation based on vastly different social classes of purchasers. GALLO cigarettes are inexpensive items while GALLO wines are not. Also, GALLO cigarettes and GALLO wines are not sold through the same channels of trade. GALLO cigarettes are distributed, marketed and sold through ambulant and sidewalk vendors, small local sari-sari stores and grocery stores in Philippine rural areas. On the other hand, GALLO wines are sold in hotels, expensive bars and restaurants, and high-end grocery stores and supermarkets, not through sari-sari stores or ambulant vendors.

Sub-Issue 5: Whether or not the GALLO WINE TRADEMARK is a well-known mark in the context of the Paris Convention. Ruling: No, the GALLO trademark cannot be considered a strong and distinct mark in the Philippines. Documentary evidence that aside from Gallo Winery’s GALLO trademark registration, a certificate

of registration was also issued in 1992 under the Principal Register approving Productos Alimenticios Gallo, S.A.’s application for GALLO trademark registration and use for its "noodles, prepared food or canned noodles, ready or canned sauces for noodles, semolina, wheat flour and bread crumbs, pastry, confectionery, ice cream, honey, molasses syrup, yeast, baking powder, salt, mustard, vinegar, species and ice.

Most importantly, "GALLO" cannot be considered a "well-known" mark within the contemplation and protection of the Paris Convention in this case since wines and cigarettes are not identical or similar goods. Similarity is a third necessary element for the Paris Convention to apply.

108

Page 109: IP Developed Reviewer

Regarding the applicability of Article 8 of the Paris Convention, there is no automatic protection afforded an entity whose tradename is alleged to have been infringed through the use of that name as a trademark by a local entity.

Pertinent Issue 2: Whether or not petitioners are liable for unfair competition. No. Actual or probable deception and confusion on the part of customers by reason of

defendant’s practices must always appear. On this score, we find that petitioners never attempted to pass off their cigarettes as those of respondents. There is no evidence of bad faith or fraud imputable to petitioners in using their GALLO cigarette mark.

What are the two types of confusion in trademark infringement? Likelihood of confusion is admittedly a relative term, to be determined rigidly according to the

particular (and sometimes peculiar) circumstances of each case. The first is "confusion of goods" when an otherwise prudent purchaser is induced to purchase

one product in the belief that he is purchasing another, in which case defendant’s goods are then bought as the plaintiff’s and its poor quality reflects badly on the plaintiff’s reputation.

The other is "confusion of business" wherein the goods of the parties are different but the defendant’s product can reasonably (though mistakenly) be assumed to originate from the plaintiff, thus deceiving the public into believing that there is some connection between the plaintiff and defendant which, in fact, does not exist.

What are the distinctions between Trademark Infringement and Unfair Competition? Law on unfair competition is broader and more inclusive. The law on trademark infringement

is more limited but it recognizes a more exclusive right derived from the trademark adoption and registration by the person whose goods or business is first associated with it. It is a specialized subject distinct from the law on unfair competition, although the two subjects are entwined with each other and are dealt with together in the Trademark Law. Hence, even if one fails to establish his exclusive property right to a trademark, he may still obtain relief on the ground of his competitor’s unfairness or fraud. Conduct constitutes unfair competition if the effect is to pass off on the public the goods of one man as the goods of another. It is not necessary that any particular means should be used to this end.

In Del Monte Corporation vs. Court of Appeals, we distinguished trademark infringement from unfair competition:1. Infringement of trademark is the unauthorized use of a trademark, whereas unfair

competition is the passing off of one's goods as those of another.2. In infringement of trademark fraudulent intent is unnecessary, whereas in unfair

competition fraudulent intent is essential.3. In infringement of trademark the prior registration of the trademark is a prerequisite to the

action, whereas in unfair competition registration is not necessary.

What are the Pertinent Provisions on Trademark Infringement under the Paris Convention and the Trademark Law? Paris Convention protects well-known trademarks only (to be determined by domestic

authorities), while the Trademark Law protects all trademarks, whether well-known or not, provided that they have been registered and are in actual commercial use in the Philippines.

109

Page 110: IP Developed Reviewer

Following universal acquiescence and comity, in case of domestic legal disputes on any conflicting provisions between the Paris Convention (which is an international agreement) and the Trademark law (which is a municipal law) the latter will prevail.

Under both the Paris Convention and the Trademark Law, the protection of a registered trademark is limited only to goods identical or similar to those in respect of which such trademark is registered and only when there is likelihood of confusion. Under both laws, the time element in commencing infringement cases is material in ascertaining the registrant’s express or implied consent to another’s use of its trademark or a colorable imitation thereof. This is why acquiescence, estoppel or laches may defeat the registrant’s otherwise valid cause of action.

Hence, proof of all the elements of trademark infringement is a condition precedent to any finding of liability.

J. Rights of foreigners to sue (Sec. 160)Section 160. Right of Foreign Corporation to Sue in Trademark or Service Mark Enforcement

Action. - Any foreign national or juridical person who meets the requirements of Section 3 of this Act and does not engage in business in the Philippines may bring a civil or administrative action hereunder for opposition, cancellation, infringement, unfair competition, or false designation of origin and false description, whether or not it is licensed to do business in the Philippines under existing laws. (Sec. 21-A, R.A. No. 166a)

La Chemise Lacoste v. Fernandez, G.R. No. 63796-97, May 21, 1984Converse v. Universal Rubber Products, supra

110